Maternity Final

¡Supera tus tareas y exámenes ahora con Quizwiz!

14. Oogenesis, the process of egg formation, begins during fetal life in the female. Which statement related to ovum formation is correct? a. Two million primary oocytes will mature. b. At birth, all ova are contained in the female's ovaries. c. The oocytes complete their division during fetal life. d. Monthly, at least two oocytes mature.

ANS: B Feedback B All of the cells that may undergo meiosis in a woman's lifetime are contained in the ovaries at birth.

1. The breastfeeding mother should be taught a safe method to remove the breast from the baby's mouth. Which suggestion by the nurse is most appropriate? a. Slowly remove the breast from the baby's mouth when the infant has fallen asleep and the jaws are relaxed. b. Break the suction by inserting your finger into the corner of the infant's mouth. c. A popping sound occurs when the breast is correctly removed from the infant's mouth. d. Elicit the Moro reflex to wake the baby and remove the breast when the baby cries.

ANS: B Inserting a finger into the corner of the baby's mouth between the gums to break the suction avoids trauma to the breast. The infant who is sleeping may lose grasp on the nipple and areola, resulting in "chewing" on the nipple, making it sore. A popping sound indicates improper removal of the breast from the baby's mouth and may cause cracks or fissures in the breast. Most mothers prefer the infant to continue to sleep after the feeding. Gentle wake-up techniques are recommended.

During which phase of maternal adjustment will the mother relinquish the baby of her fantasies and accept the real baby? a. Letting go b. Taking hold c. Taking in d. Taking on

ANS: A Accepting the real infant and relinquishing the fantasy infant occurs during the letting-go phase of maternal adjustment. During the taking-hold phase the mother assumes responsibility for her own care and shifts her attention to the infant. In the taking-in phase the mother is primarily focused on her own needs. There is no taking-on phase of maternal adjustment.

A nurse is teaching a prenatal class. The nurse teaches that during weeks 25 to 28, which fetal development occurs? a. Eyes reopen. b. Vernix caseosa covers the skin. c. Lanugo may develop. d. Brown fat is deposited.

ANS: A During this time frame the eyes reopen, and the fetus becomes plumper with smoother skin. The other changes occur during weeks 17 to 20.

Which woman is most likely to have severe afterbirth pains and request a narcotic analgesic? a. Gravida 5, para 5 b. Woman who is bottle-feeding her first child c. Primipara who delivered a 7-lb boy d. Woman who has started to breastfeed

ANS: A The discomfort of after pains is more acute for multiparas because repeated stretching of muscle fibers leads to loss of uterine muscle tone. After pains are particularly severe during breastfeeding, not bottle-feeding. The uterus of a primipara tends to remain contracted. The breastfeeding woman may have increased pain due to engorgement, but the multipara probably will have the most severe afterbirth pains.

Which finding 12 hours after birth requires further assessment? a. The fundus is palpable two fingerbreadths above the umbilicus. b. The fundus is palpable at the level of the umbilicus. c. The fundus is palpable one fingerbreadth below the umbilicus. d. The fundus is palpable two fingerbreadths below the umbilicus.

ANS: A The fundus rises to the umbilicus after delivery and remains there for about 24 hours. A fundus that is above the umbilicus may indicate uterine atony or urinary retention. The nurse needs to make further assessments. The other findings are within normal limits for the time period.

A 36-year-old divorcee with a successful modeling career finds out that her 18-year-old married daughter is expecting her first child. What is a major factor in determining how the woman will respond to becoming a grandmother? a. Her career b. Being divorced c. Her age d. Age of the daughter

ANS: C Feedback A Career responsibilities may have demands that make the grandparents not as accessible, but it is not a major factor in determining the woman's response to becoming a grandmother. B Being divorced is not a major factor that determines adaptation of grandparents. C Age is a major factor in determining the emotional response of prospective grandparents. Young grandparents may not be happy with the stereotype of grandparents as being old. D The age of the daughter is not a major factor that determines adaptation of grandparents. The age of the grandparent is a major factor.

The nurse assesses a woman's episiotomy or perineal laceration using the acronym REEDA. What factors does this include? (Select all that apply.) a. Redness b. Edema c. Approximation d. Depth e. Discharge

ANS: A, B, C, E The acronym REEDA indicates redness, edema, ecchymosis or bruising, discharge, and approximation. Depth is not a consideration with this acronym.

A patient who is 7 months pregnant states, "I'm worried that something will happen to my baby." The nurse's best response is a. "There is nothing to worry about." b. "The doctor is taking good care of you and your baby." c. "Tell me about your concerns." d. "Your baby is doing fine."

ANS: C Feedback A This statement is belittling the patient's concerns. B This statement is belittling the patient's concerns by telling her she should not worry. C Encouraging the client to discuss her feelings is the best approach. Women during their third trimester need reassurance that such fears are not unusual in pregnancy. D This statement disregards the patient's feelings and treats them as unimportant.

3. The student learns about shunts that support fetal circulation. Which of the following are included in this support system? (Select all that apply.) a. Ductus venosus b. Foramen ovale c. Ductus arteriosus d. Foramen magnum e. Ductus deferens

ANS: A,B,C The ductus venosus, foramen ovale, and ductus arteriosus are part of fetal circulation. The foramen magnum is located at the base of the skull. The ductus (or vas) deferens is part of the male reproductive system.

A woman's last menstrual period was June 10. Her estimated date of delivery (EDD) is a. April 7 b. March 17 c. March 27 d. April 17

ANS: B Feedback A April 7 would be subtracting 2 months instead of 3 months and then subtracting 3 days instead of adding 7 days. B To determine the EDD, the nurse uses the first day of the last menstrual period (June 10), subtracts 3 months (March 10), and adds 7 days (March 17). C March is the correct month, but instead of adding 7 days, 17 days were added. D April 17 is subtracting 2 months instead of 3 months.

Which statement related to changes in the breasts during pregnancy is the most accurate? a. During the early weeks of pregnancy there is decreased sensitivity. b. Nipples and areolae become more pigmented. c. Montgomery tubercles are no longer visible around the nipples. d. Venous congestion of the breasts is more visible in the multiparous woman.

ANS: B Feedback A Fullness, heightened sensitivity, tingling and heaviness of the breasts occur in the early weeks of gestation in response to increased levels of estrogen and progesterone. B Nipples and areolae become more pigmented, and the nipples become more erectile and may express colostrum. C Montgomery tubercles may be seen around the nipples. These sebaceous glands may have a protective role in that they keep the nipples lubricated for breastfeeding. D Venous congestion in the breasts is more obvious in primigravidas.

29. The nurse caring for a woman hospitalized for hyperemesis gravidarum should expect that initial treatment involves a. Corticosteroids to reduce inflammation b. IV therapy to correct fluid and electrolyte imbalances c. An antiemetic, such as pyridoxine, to control nausea and vomiting d. Enteral nutrition to correct nutritional deficits

ANS: B Feedback A Corticosteroids have been used successfully to treat refractory hyperemesis gravidarum, but they are not the expected initial treatment for this disorder. B Initially, the woman who is unable to down clear liquids by mouth requires IV therapy for correction of fluid and electrolyte imbalances. C Pyridoxine is vitamin B6, not an antiemetic. Promethazine, a common antiemetic, may be prescribed. D In severe cases of hyperemesis gravidarum, enteral nutrition via a feeding tube may be necessary to correct maternal nutritional deprivation. This is not an initial treatment for this patient. PTS: 1 DIF: Cognitive Level: Comprehension REF: p. 590 OBJ: Nursing Process: Implementation MSC: Client Needs: Physiologic Integrity

16. A patient with pregnancy-induced hypertension is admitted complaining of pounding headache, visual changes, and epigastric pain. Nursing care is based on the knowledge that these signs indicate a. Anxiety due to hospitalization b. Worsening disease and impending convulsion c. Effects of magnesium sulfate d. Gastrointestinal upset

ANS: B Feedback A These are danger signs and should be treated. B Headache and visual disturbances are due to increased cerebral edema. Epigastric pain indicates distention of the hepatic capsules and often warns that a convulsion is imminent. C She has not been started on magnesium sulfate as a treatment yet. Also, these are not expected effects of the medication. D These are danger signs showing increased cerebral edema and impending convulsion. PTS: 1 DIF: Cognitive Level: Analysis REF: pp. 599-600 OBJ: Nursing Process: Assessment MSC: Client Needs: Physiologic Integrity

28. Methotrexate is recommended as part of the treatment plan for which obstetric complication? a. Complete hydatidiform mole b. Missed abortion c. Unruptured ectopic pregnancy d. Abruptio placentae

ANS: C Feedback A Methotrexate is not indicated or recommended as a treatment option for a complete hydatidiform mole. B Methotrexate is not indicated or recommended as a treatment option for missed abortions. C Methotrexate is an effective, nonsurgical treatment option for a hemodynamically stable woman whose ectopic pregnancy is unruptured and less than 4 cm in diameter. D Methotrexate is not indicated or recommended as a treatment option for abruptio placentae. PTS: 1 DIF: Cognitive Level: Knowledge REF: p. 580 OBJ: Nursing Process: Planning MSC: Client Needs: Physiologic Integrity

A nurse is teaching a group of prenatal clients about the importance of exercise during pregnancy. Which client would be the best candidate to continue with her exercise regime? a. A client with placenta previa. b. A client with a diagnosis of diabetes. c. A client with a diagnosis of pre-eclampsia. d. A client with an incompetent cervix and cerclage

A client with a diagnosis of diabetes. Rationale: Exercise would be therapeutic in helping a client with a diagnosis of diabetes to control her glucose utilization. However, it would be contraindicated in a client with an incompetent cervix and cerclage, a client with a diagnosis of pre-eclampsia, or a client with placenta previa.

14. When a nonreassuring pattern of the fetal heart rate is noted and the mother is lying on her left side, what nursing action is indicated? a. Lower the head of the bed. b. Place the mother in a Trendelenburg position. c. Change her position to the right side. d. Place a wedge under the left hip.

ANS: C Feedback A Repositioning to the opposite side is the first intervention. If unsuccessful with improving the FHR pattern, further changes in position can be attempted. B The Trendelenburg position is not appropriate for early interventions. If unsuccessful with improving the FHR pattern with other types of position changes, Trendelenburg may be the choice. C Repositioning on the opposite side may relieve compression on the umbilical cord and improve blood flow to the placenta. D The woman is already on her left side, so a wedge on that side is not an appropriate choice.

Which hormone remains elevated in the immediate postpartum period of the breastfeeding woman? a. Estrogen b. Progesterone c. Prolactin d. Human placental lactogen

ANS: C Prolactin levels in the blood increase progressively throughout pregnancy. In women who breastfeed, prolactin levels remain elevated into the sixth week after birth. Estrogen and progesterone levels decrease markedly after expulsion of the placenta, reaching their lowest levels 1 week into the postpartum period. Human placental lactogen levels dramatically decrease after expulsion of the placenta.

32. A woman had an epidural place an hour ago and is now complaining of severe itching. What action by the nurse is most appropriate? a. Discontinue the epidural infusion at once. b. Notify the anesthesia provider. c. Prepare to administer diphenhydramine (Benedryl). d. Prepare to administer promethazine (Phenergan).

ANS: C Pruritis (itching) is a common side effect of epidural medications. The nurse should be prepared to administer diphenhydramine. There is no need to discontinue the epidural infusion or notify the anesthesia provider. Promethazine is used for nausea. PTS: 1 DIF: Cognitive Level: Application/Applying REF: Table 18.1 OBJ: Nursing Process: Implementation MSC: Client Needs: Physiologic Integrity

6. While examining a newborn, the nurse notes uneven skin folds on the buttocks and a click when performing the Ortolani maneuver. The nurse recognizes these findings as a sign that the newborn probably has: a. Polydactyly. c. Hip dysplasia. b. Clubfoot. d. Webbing

ANS: C The Ortolani maneuver is used to detect the presence of hip dysplasia. Polydactyly is the presence of extra digits. Clubfoot (talipes equinovarus) is a deformity in which the foot turns inward and is fixed in a plantar-flexion position. Webbing, or syndactyly, is a fusing of the fingers or toes.

What documentation on a woman's chart on postpartum day 14 indicates a normal involution process? a. Moderate bright red lochial flow b. Breasts firm and tender c. Fundus below the symphysis and not palpable d. Episiotomy slightly red and puffy

ANS: C The fundus descends 1 cm/day, so by postpartum day 14 it is no longer palpable. The lochia should be changed by this day to serosa. Breasts are not part of the involution process. The episiotomy should not be red or puffy at this stage.

8. The nurse teaches a pregnant woman that which diagnostic test evaluates the effect of fetal movement on fetal heart activity? a. Contraction stress test (CST) b. Sonography c. Biophysical profile d. Nonstress test (NST)

ANS: D An NST evaluates the ability of the fetal heart to accelerate either spontaneously or in association with fetal movement. CST evaluates the fetal reaction to contractions. Sonographic examinations visualize the fetus and are done for various other reasons. The biophysical profile evaluates fetal status using many variables. PTS: 1 DIF: Cognitive Level: Knowledge/Remembering REF: p. 281 OBJ: Integrated Process: Teaching-Learning MSC: Client Needs: Physiologic Integrity

12. The difference between the aseptic and terminal methods of sterilization is that the a. aseptic method does not require boiling of the bottles. b. terminal method requires boiling water to be added to the formula. c. aseptic method requires a longer preparation time. d. terminal method sterilizes the prepared formula at the same time it sterilizes the equipment.

ANS: D In the terminal sterilization method, the formula is prepared in the bottles, which are loosely capped, and then the bottles are placed in the sterilizer, where they are boiled for 25 minutes. With the aseptic method, the bottles are boiled separate from the formula. This process takes about 5 minutes.

9. A woman received 50 mcg of fentanyl intravenously 1 hour before delivery. What drug should the nurse have readily available? a. Promethazine (Phenergan) b. Nalbuphine (Nubain) c. Butorphanol (Stadol) d. Naloxone (Narcan)

ANS: D Naloxone reverses narcotic-induced respiratory depression, which may occur with administration of narcotic analgesia. Phenergan is normally given for nausea. Nubain and Stadol are analgesics that can be given to women in labor. PTS: 1 DIF: Cognitive Level: Knowledge/Remembering REF: p. 372 | Table 18.2 OBJ: Nursing Process: Planning MSC: Client Needs: Physiologic Integrity

8. The nurse assessing a newborn knows that the most critical physiologic change required of the newborn is: a. Closure of fetal shunts in the circulatory system. b. Full function of the immune defense system at birth. c. Maintenance of a stable temperature. d. Initiation and maintenance of respirations.

ANS: D The most critical adjustment of a newborn at birth is the establishment of respirations. The cardiovascular system changes markedly after birth as a result of fetal respiration, which reduces pulmonary vascular resistance to the pulmonary blood flow and initiates a chain of cardiac changes that support the cardiovascular system. The infant relies on passive immunity received from the mother for the first 3 months of life. After the establishment of respirations, heat regulation is critical to newborn survival.

Which nursing action is most appropriate to correct a boggy uterus that is displaced above and to the right of the umbilicus? a. Notify the provider of an impending hemorrhage. b. Assess the blood pressure and pulse. c. Evaluate the lochia. d. Assist the patient in emptying her bladder.

ANS: D Urinary retention can cause overdistention of the urinary bladder, which lifts and displaces the uterus. Nursing actions need to be implemented before notifying the provider. Blood pressure, pulse, and lochia are important to assess, but first the nurse assesses the bladder so corrective action can be taken if needed.

2. Infant mortality for late preterm infants (34 to 36 weeks) is three times the rate of mortality for term infants. Is this statement true or false?

ANS: T This statement is correct. LPI infants may appear full term at birth; however, the appearance of the infant is deceiving. LPI infants have a mortality risk three times that of term infants for death from all causes. PTS: 1 DIF: Cognitive Level: Comprehension REF: p. 646 OBJ: Nursing Process: Assessment MSC: Client Needs: Physiologic Integrity

One technique to measure a preterm infants output is to A. Attach a plastic bag to the perineum to collect the urine B. Lay a diaper under the infant and weigh the diaper when changing C. Weigh the diaper when changed D. Insert a Foley catheter

Answer: C Rationale: A. Attaching a bag to the perineum can cause skin breakdown and the potential for infection. B. Laying the diaper under the infant may not collect all of the urine. C. The diaper is weighed prior to putting it on the infant and when it is taken off. The fluid loss is 1 mL = 1 g of weight. D. Foley catheters will increase the risk of infections.

Infants born before surfactant production are at risk for _________.

Answer: Respiratory distress syndrome

Meconium-stained amniotic fluid occurs in 10% to 15% of births. Meconium aspiration syndrome (MAS) is a condition in which there is obstruction, air trapping, and chemical pneumonitis caused by meconium in the infant's lungs. MAS develops in what percentage of those infants? A. 1% B. 5% C. 25% D. 50%

B. 5% MAS occurs in 5% of cases. It most often occurs when hypoxia causes increased peristalsis of the intestines and relaxation of the anal sphincter before or during labor. MAS develops when meconium in the amniotic fluid enters the lungs during fetal life or at birth.

Which factor is known to increase the risk of gestational diabetes mellitus? a. Previous birth of large infant b. Maternal age younger than 25 years c. Underweight prior to pregnancy d. Previous diagnosis of type 2 diabetes mellitus.

a. Previous birth of large infant

Examination of a newborn in the birth room reveals bilateral cataracts. Which disease process in the maternal history would likely cause this abnormality? a. Rubella b. Cytomegalovirus (CMV) c. Syphilis d. HIV

a. Rubella

Which factor is most important in diminishing maternal, fetal, and neonatal complications in a pregnant client with diabetes? a. Evaluation of retinopathy by an ophthalmologist b. The client's stable emotional and psychological status c. Degree of glycemic control before and during the pregnancy d. Total protein excretion and creatinine clearance within normal limits

c. Degree of glycemic control before and during the pregnancy

Which client teaching instructions are necessary for a pregnant client who is to undergo a glucose challenge test (GCT) as part of a routine pregnancy treatment plan? a. Consume a low-fat diet for 48 hours prior to testing. b. Fast for 12 hours prior to testing. c. There are no dietary restrictions prior to testing. d. Consume a consistent carbohydrate diet (60 g) prior

c. There are no dietary restrictions prior to testing.

16. At approximately _____ weeks of gestation, lecithin is forming on the alveolar surfaces, the eyelids open, and the fetus measures approximately 27 cm crown to rump and weighs approximately 1110 g. a. 20 b. 24 c. 28 d. 30

ANS: C Feedback C These are all milestones that occur at 28 weeks.

7. To initiate the milk ejection reflex, the mother should a. wear a firm-fitting bra. b. drink plenty of fluids. c. place the infant to the breast d. apply cool packs to her breast.

ANS: C Oxytocin, which causes the milk let-down reflex, increases in response to nipple stimulation. A firm bra is important to support the breast but will not initiate the let-down reflex. Drinking plenty of fluids is necessary for adequate milk production but will not initiate the let-down reflex. Cool packs to the breast will decrease the let-down reflex.

13. How many ounces will a formula-fed infant who is on a 4-hour feeding schedule need to consume at each feeding to meet daily caloric needs? a. 0.5 to 1 b. 1 to 2 c. 2 to 3 d. 4

ANS: C The newborn requires approximately 2 to 3 ounces per feeding within 1 week after birth.

As the nurse is assessing a 2-day-old newborn, jaundice is noted on the face only. The nurse can anticipate a bilirubin level of about A. 5 mg/dL. B. 10 mg/dL. C. 15 mg/dL. D. 20 mg/dL.

A When the bilirubin level reaches 5 to 7 mg/dL, jaundice is visible in the newborn's face. It moves down the body as bilirubin levels continue to rise.

Calculate the body mass index (BMI) for a woman who is 65 cm tall and weighs 115 lb.

ANS: BMI 16 A commonly used method of evaluating the appropriateness of weight for height is the BMI, which is calculated by the following formula. BMI = Weight in kg divided by height in meters squared

While providing education to a primiparous woman regarding the normal changes of pregnancy, it is important for the nurse to explain that the uterus undergoes irregular contractions. These are known as _____________ contractions.

ANS: Braxton Hicks Irregular painless contractions occur throughout pregnancy, although many women do not notice them until the third trimester. Women who are unsure, who have 5 or 6 regular contractions within one hour, or who demonstrate other signs of labor should contact their provider.

2. The condition in which the placenta is implanted in the lower uterine segment near or over the internal cervical os is _____________.

ANS: placenta previa In placenta previa, the placenta is implanted in the lower uterine segment such that it completely or partially covers the cervix or is close enough to the cervix to cause bleeding when the cervix dilates or the lower uterine segment effaces. PTS: 1 DIF: Cognitive Level: Comprehension REF: p. 583 OBJ: Nursing Process: Assessment MSC: Client Needs: Physiologic Integrity

Prenatal testing for the human immunodeficiency virus (HIV) is recommended for which women? a. All women, regardless of risk factors b. A woman who has had more than one sexual partner c. A woman who has had a sexually transmitted infection d. A woman who is monogamous with her partner

ANS: A Feedback A An HIV test is recommended for all women, regardless of risk factors. The incidence of perinatal transmission from an HIV-positive mother to her fetus ranges from 25% to 35%. Women who test positive for HIV can then be treated. B All women should be tested for HIV, although this patient is at increased risk of contracting the disease. C Regardless of past sexual history, all women should have an HIV test completed prenatally. D Although this patient is apparently monogamous, an HIV test is still recommended.

17. Rh incompatibility can occur if the woman is Rh negative and her a. Fetus is Rh positive b. Husband is Rh positive c. Fetus is Rh negative d. Husband and fetus are both Rh negative

ANS: A Feedback A For Rh incompatibility to occur, the mother must be Rh negative and her fetus Rh positive. B The husband's Rh factor is a concern only as it relates to the possible Rh factor of the fetus. C If the fetus is Rh negative, the blood types are compatible and no problems should occur. D If the fetus is Rh negative, the blood type with the mother is compatible. The husband's blood type does not enter into the problem. PTS: 1 DIF: Cognitive Level: Knowledge REF: p. 601 OBJ: Nursing Process: Assessment MSC: Client Needs: Physiologic Integrity

Immediately after the forceps-assisted birth of an infant, the nurse should a. Assess the infant for signs of trauma. b. Give the infant prophylactic antibiotics. c. Apply a cold pack to the infant's scalp. d. Measure the circumference of the infant's head.

ANS: A Feedback A Forceps delivery can result in local irritation, bruising, or lacerations of the fetal scalp. B Prophylactic antibiotics are not necessary with a forceps delivery. C This would put the infant at risk for cold stress and would be contraindicated. D Measuring the circumference of the head is part of the initial nursing assessment.

What is an essential part of nursing care for the laboring woman? a. Helping the woman manage the pain. b. Eliminating the pain associated with labor. c. Sharing personal experiences regarding labor and delivery to decrease her anxiety. d. Feeling comfortable with the predictable nature of intrapartal care.

ANS: A Feedback A Helping a woman manage the pain is an essential part of nursing care, because pain is an expected part of normal labor and cannot be fully relieved. B Labor pain cannot be fully relieved. C Decreasing anxiety is important, but managing pain is a top priority. D The labor nurse should always be assessing for unpredictable occurrences.

Leopold's maneuvers are used by practitioners to determine a. The best location to assess the fetal heart rate (FHR) b. Cervical dilation and effacement c. Whether the fetus is in the posterior position d. The status of the membranes

ANS: A Feedback A Leopold's maneuvers are often performed before assessing the FHR. These maneuvers help identify the best location to obtain the FHR. B Dilation and effacement are best determined by vaginal examination. C Assessment of fetal position is more accurate with vaginal examination. D A Nitrazine or ferning test can be performed to determine the status of the fetal membranes.

20. The nurse caring for the woman in labor should understand that absent or minimal variability is classified as either abnormal or indeterminate. Which condition related to decreased variability is considered benign? a. A periodic fetal sleep state b. Extreme prematurity c. Fetal hypoxemia d. Pre-existing neurologic injury

ANS: A Feedback A When the fetus is temporarily in a sleep state there is minimal variability present. Periodic fetal sleep states usually last no longer than 30 minutes. B A woman who presents in labor with extreme prematurity may display a FHR pattern of minimal or absent variability. C Abnormal variability may also be related to fetal hypoxemia and metabolic acidemia. D Congenital anomalies or pre-existing neurologic injury may also present as absent or minimal variability. Other possible causes might be CNS depressant medications, narcotics or general anesthesia.

18. A breastfeeding mother who was discharged yesterday calls to ask about a tender, hard area on her right breast. The nurse's first response should be a. "Try massaging the area and apply heat, as this is probably a plugged duct." b. "Stop breastfeeding because you probably have an infection." c. "Notify your doctor so he can start you on antibiotics." d. "This is a normal response in breastfeeding mothers."

ANS: A A plugged lactiferous duct results in localized edema, tenderness, and a palpable hard area. Massage of the area followed by heat will cause the duct to open. This does not indicate an infection or a need for antibiotics. This is a normal deviation but requires intervention to prevent further complications.

Rho immune globulin will be ordered postpartum if which situation occurs? a. Mother Rh-, baby Rh+ b. Mother Rh-, baby Rh- c. Mother Rh+, baby Rh+ d. Mother Rh+, baby Rh-

ANS: A An Rh- mother delivering an Rh+ baby may develop antibodies to fetal cells that entered her bloodstream when the placenta separated. The Rho immune globulin works to destroy the fetal cells in the maternal circulation before sensitization occurs. The other blood type combinations would not necessitate the use of Rhogam.

11. The placenta allows exchange of oxygen, nutrients, and waste products between the mother and fetus by a. Contact between maternal blood and fetal capillaries within the chorionic villi b. Interaction of maternal and fetal pH levels within the endometrial vessels c. A mixture of maternal and fetal blood within the intervillous spaces d. Passive diffusion of maternal carbon dioxide and oxygen into the fetal capillaries

ANS: A Feedback A Fetal capillaries within the chorionic villi are bathed with oxygen- and nutrient-rich maternal blood within the intervillous spaces.

Maternal nutritional status is an especially significant factor of the many factors that influence the outcome of pregnancy because: a. It is very difficult to adjust because of people's ingrained eating habits b. It is an important preventive measure for a variety of problems c. Women love obsessing about their weight and diets d. A woman's preconception weight becomes irrelevant

ANS: B Nutritional status draws so much attention not only for its effect on a healthy pregnancy and birth but also because significant changes are within relatively easy reach. Pregnancy is a time when many women are motivated to learn about adequate nutrition and make changes to their diet that will be of benefit to their baby. Pregnancy is not the time to begin a weight loss diet. Clients and their caregivers should still be concerned with appropriate weight gain.

38. The process in which bilirubin is changed from a fat-soluble product to a water-soluble product is known as: a. Enterohepatic circuit. c. Unconjugation of bilirubin. b. Conjugation of bilirubin. d. Albumin binding.

ANS: B Conjugation of bilirubin is the process of changing the bilirubin from a fat-soluble to a water-soluble product. This is the route by which part of the bile produced by the liver enters the intestine, is reabsorbed by the liver, and then is recycled into the intestine. Unconjugated bilirubin is fat soluble. Albumin binding is to attach something to a protein molecule.

4. A newborn is placed under a radiant heat warmer, and the nurse evaluates the infant's body temperature every hour. Maintaining the newborn's body temperature is important for preventing: a. Respiratory depression. c. Tachycardia. b. Cold stress. d. Vasoconstriction.

ANS: B Loss of heat must be controlled to protect the infant from the metabolic and physiologic effects of cold stress, and that is the primary reason for placing a newborn under a radiant heat warmer. Cold stress results in an increased respiratory rate and vasoconstriction.

4. A new mother recalls that she should feed her newborn when she exhibits feeding readiness cues rather than waiting until her infant is crying frantically. Based on this information, this woman should feed her infant when she a. waves her arms in the air. b. makes sucking motions. c. has hiccups. d. stretches out her legs straight.

ANS: B Sucking motions, rooting, mouthing, and hand-to-mouth motions are examples of feeding-readiness cues. The other observations are not feeding cues.

When assessing the fetus using Leopold maneuvers, the nurse feels a round, firm, movable fetal part in the fundal portion of the uterus and a long, smooth surface in the mother's right side close to midline. What is the likely position of the fetus? a. ROA b. LSP c. RSA d. LOA

ANS: C Feedback A Fetal position is denoted with a three-letter abbreviation. The first letter indicates the presenting part in either the right or left side of the maternal pelvis. The second letter indicates the anatomic presenting part of the fetus. The third letter stands for the location of the presenting part in relation to the anterior, posterior, or transverse portion of the maternal pelvis. Palpation of a round, firm fetal part in the fundal portion of the uterus would be the fetal head, indicating that the fetus is in a breech position with the sacrum as the presenting part in the maternal pelvis. Palpation of the fetal spine along the mother's right side denotes the location of the presenting part in the mother's pelvis. The ability to palpate the fetal spine indicates that the fetus is anteriorly positioned in the maternal pelvis. RO/A/ denotes a fetus that is positioned anteriorly in the right side of the maternal pelvis with the occiput as the presenting part. B Fetal position is denoted with a three-letter abbreviation. The first letter indicates the presenting part in either the right or left side of the maternal pelvis. The second letter indicates the anatomic presenting part of the fetus. The third letter stands for the location of the presenting part in relation to the anterior, posterior, or transverse portion of the maternal pelvis. Palpation of a round, firm fetal part in the fundal portion of the uterus would be the fetal head, indicating that the fetus is in a breech position with the sacrum as the presenting part in the maternal pelvis. Palpation of the fetal spine along the mother's right side denotes the location of the presenting part in the mother's pelvis. The ability to palpate the fetal spine indicates that the fetus is anteriorly positioned in the maternal pelvis. LSP describes a fetus that is positioned posteriorly in the left side of the pelvis with the sacrum as the presenting part. C Fetal position is denoted with a three-letter abbreviation. The first letter indicates the presenting part in either the right or left side of the maternal pelvis. The second letter indicates the anatomic presenting part of the fetus. The third letter stands for the location of the presenting part in relation to the anterior, posterior, or transverse portion of the maternal pelvis. Palpation of a round, firm fetal part in the fundal portion of the uterus would be the fetal head, indicating that the fetus is in a breech position with the sacrum as the presenting part in the maternal pelvis. Palpation of the fetal spine along the mother's right side denotes the location of the presenting part in the mother's pelvis. The ability to palpate the fetal spine indicates that the fetus is anteriorly positioned in the maternal pelvis. This fetus is positioned anteriorly in the right side of the maternal pelvis with the sacrum as the presenting part. RS/A/ is the correct three-letter abbreviation to indicate this fetal position. D Fetal position is denoted with a three-letter abbreviation. The first letter indicates the presenting part in either the right or left side of the maternal pelvis. The second letter indicates the anatomic presenting part of the fetus. The third letter stands for the location of the presenting part in relation to the anterior, posterior, or transverse portion of the maternal pelvis. Palpation of a round, firm fetal part in the fundal portion of the uterus would be the fetal head, indicating that the fetus is in a breech position with the sacrum as the presenting part in the maternal pelvis. Palpation of the fetal spine along the mother's right side denotes the location of the presenting part in the mother's pelvis. The ability to palpate the fetal spine indicates that the fetus is anteriorly positioned in the maternal pelvis. /A/ fetus that is LOA, would be positioned anteriorly in the left side of the pelvis with the occiput as the presenting part.

The nurse thoroughly dries the infant immediately after birth primarily to a. Stimulate crying and lung expansion. b. Remove maternal blood from the skin surface. c. Reduce heat loss from evaporation. d. Increase blood supply to the hands and feet.

ANS: C Feedback A Rubbing the infant does stimulate crying, but it is not the main reason for drying the infant. B Drying the infant after birth does not remove all of the maternal blood. C Infants are wet with amniotic fluid and blood at birth, which accelerates evaporative heat loss. D The main purpose of drying the infant is to prevent heat loss.

Infant Jill is preterm and on a respirator, with intravenous lines and much equipment around her when her parents come to visit for the first time. It is important for the nurse to: a. Suggest that the parents visit for only a short time to reduce their anxieties. b. Reassure the parents that the baby is progressing well. c. Encourage the parents to touch Jill. d. Discuss the care they will give Jill when she goes home.

ANS: C Touching the infant will increase the development of attachment.

1. A pregnant woman who abuses cocaine admits to exchanging sex for her drug habit. This behavior puts her at a greater risk for a. Depression of the central nervous system b. Hypotension and vasodilation c. Sexually transmitted diseases d. Postmature birth

ANS: C Feedback A Cocaine is a central nervous system stimulant. B Cocaine causes hypertension and vasoconstriction. C Sex acts exchanged for drugs place the woman at increased risk for sexually transmitted diseases because of multiple partners and lack of protection. D Premature delivery of the infant is one of the most common problems associated with cocaine use during pregnancy.

A prenatal client in the third trimester of pregnancy is diagnosed with varicosities in the vulva and perineum. Which self-care strategy should the nurse teach? a. Change shoes several times throughout the day. b. Elevate legs and feet while sitting or lying down. c. Elevate legs level with hips while sitting or lying down. d. Use supportive hose in the afternoon and evening.

Elevate legs level with hips while sitting or lying down. Rationale: The hips, as well as the feet and legs, must be elevated to promote venous drainage into the trunk. Supportive hose need to be applied in the morning, rather than starting in the afternoon or evening. Changing shoes might help with back pain but not with varicosities.

For which of the following infectious diseases can a woman be immunized? a. Rubella b. Toxoplasmosis c. Cytomegalovirus d. Herpesvirus type 2

a. Rubella

15. When assessing a newborn boy at 12 hours of age, the nurse notes a rash on his abdomen and thighs composed of reddish macules, papules, and small vesicles. The nurse would: a. document the finding as erythema toxicum. b. isolate the newborn and his mother until infection is ruled out. c. apply an antiseptic ointment to each lesion. d. request nonallergenic linen from the laundry.

15. a; the rash described is erythema toxicum; it is an inflammatory response that has no clinical significance and requires no treatment because it will disappear spontaneously.

16. A breastfed full-term newborn girl is 12 hours old and is being prepared for early discharge. Which of the following assessment findings, if present, could delay discharge? a. Dark green-black stool, tarry in consistency b. Yellowish tinge in sclera and on face c. Swollen breasts with a scant amount of thin discharge d. Blood-tinged mucoid vaginal discharge

16. b; physiologic jaundice does not appear until 24 hours after birth; further investigation would be needed if it appears during the first 24 hours, because that would be consistent with pathologic jaundice; a, c, and d are all expected findings.

42. What are modes of heat loss in the newborn (Select all that apply)? a. Perspiration b. Convection c. Radiation d. Conduction e. Urination

ANS: B, C, D Convection, radiation, evaporation, and conduction are the four modes of heat loss in the newborn. Perspiration and urination are not modes of heat loss in newborns.

18. When assessing a newborn after birth, the nurse notes flat, irregular, pinkish marks on the bridge of the nose, nape of neck, and over the eyelids. The areas blanch when pressed with a finger. The nurse would document this finding as: a. milia. b. nevus vasculosus. c. telangiectatic nevi. d. nevus flammeus.

18. c; telangiectatic nevi (nevus simplex) are also known as stork bite marks and can also appear on the eyelids; milia are plugged sebaceous glands and appear like white pimples; nevus vasculosus or a strawberry mark is a raised, sharply demarcated, bright or dark red swelling; nevus flammeus is a port-wine, flat red to purple lesion that does not blanch with pressure.

Examples of situations when the birth attendant may do an episiotomy include the following. Select all that apply. A Fetal shoulder dystocia B Forceps- or vacuum extractor-assisted births C Breech presentation D Fetus in an occiput posterior position

A, B, D Fetal shoulder dystocia, forceps- or vacuum extractor-assisted births, or a fetus in an occiput posterior position are all indications for an episiotomy. Most breech presentations are delivered by cesarean.

A woman is 35 weeks' pregnant during her clinic visit. She complains of numerous vaginal infections during the pregnancy. She tells the nurse, "I'm afraid I have diabetes, because I have some infections." The best response by the nurse would be A. "Diabetes is a possibility. I will set you up for testing." B. "A vaginal infection is a symptom of diabetes, but it also is a problem with normal pregnancies due to the changes in your vaginal area." C. "Itching is a problem with pregnancies and it makes you think you have an infection. The physician can order you some cream to help with the itching and pain." D. "This seems to be a concern with all of our patients today."

B During pregnancy, the glycogen levels of the vaginal area increase. This favors the growth of yeast-causing infections. Diabetes is a possibility, but there are other considerations that need to be assessed first. Vulva itching is not a common problem with pregnancy. These symptoms should be investigated for the cause and treated.

A woman is concerned that she has developed numerous nosebleeds during this pregnancy. She feels this is a sign of leukemia and wants to be screened. The nurse's response to the woman should be based on the fact that A. leukemia is a major concern during pregnancy. B. nosebleeds are a common occurrence during pregnancy. C. nosebleeds are rare in pregnancy; therefore further assessment is necessary. D. platelet count decreases significantly during pregnancy.

B With the higher levels of estrogen causing increased vascularity in the upper respiratory tract, epistaxis is a common occurrence. Leukemia rates do not increase during pregnancy. There is a slight decrease in the platelet count but within normal range.

An infant was born 2 hours ago at 37 weeks of gestation and weighing 4.1 kg. The infant appears chubby with a flushed complexion and is very tremulous. The tremors are most likely the result of: a. Birth injury. b. Hypoglycemia. c. Hypocalcemia. d. Seizures.

B Hypoglycemia is common in the macrosomic infant. Signs of hypoglycemia include jitteriness, apnea, tachypnea, and cyanosis.

Which major neonatal complication is carefully monitored after the birth of the infant of a diabetic mother? a. Hypoglycemia b. Hypercalcemia c. Hypoinsulinemia d. Hypobilirubinemia

a. Hypoglycemia

When a pregnant client with diabetes experiences hypoglycemia while hospitalized, which should the nurse have the client do? a. Eat a candy bar. b. Eat six saltine crackers or drink 8 oz of milk. c. Drink 4 oz of orange juice followed by 8 oz of milk. d. Drink 8 oz of orange juice with 2 teaspoons of sugar added.

b. Eat six saltine crackers or drink 8 oz of milk.

Which instructions should the nurse include when teaching a pregnant client with Class II heart disease? a. Advise her to gain at least 30 pounds. b. Instruct her to avoid strenuous activity. c. Inform her of the need to limit fluid intake. d. Explain the importance of a diet high in calcium.

b. Instruct her to avoid strenuous activity.

Which form of heart disease in women of childbearing years usually has a benign effect on pregnancy? a. Cardiomyopathy b. Mitral valve prolapse c. Rheumatic heart disease d. Congenital heart disease

b. Mitral valve prolapse

Which disease process improves during pregnancy? a. Epilepsy b. Bell's palsy c. Rheumatoid arthritis d. Systemic lupus erythematosus (SLE)

c. Rheumatoid arthritis

The results of a pregnant patient's glucose tolerance test (GTT) were 158 mg/dL. What is the next test that the nurse will include in the patient's teaching plan? a. Urinalysis b. Amniocentesis c. Nonstress test d. Oral glucose tolerance test (OGTT)

d. Oral glucose tolerance test (OGTT)

Antiinfective prophylaxis is indicated for a pregnant client with a history of mitral valve stenosis related to rheumatic heart disease because the client is at risk of developing: a. hypertension. b. postpartum infection. c. bacterial endocarditis. d. upper respiratory infections.

.c. bacterial endocarditis.

An infant with severe meconium aspiration syndrome (MAS) is not responding to conventional treatment. Which highly technical method of treatment may be necessary for an infant who does not respond to conventional treatment? a. Extracorporeal membrane oxygenation b. Respiratory support with a ventilator c. Insertion of a laryngoscope and suctioning of the trachea d. Insertion of an endotracheal tube

A Extracorporeal membrane oxygenation is a highly technical method that oxygenates the blood while bypassing the lungs, thus allowing the infant's lungs to rest and recover. The infant is likely to have been first connected to a ventilator. Laryngoscope insertion and tracheal suctioning are performed after birth before the infant takes the first breath. An endotracheal tube will be in place to facilitate deep tracheal suctioning and ventilation.

With regard to injuries to the infant's plexus during labor and birth, nurses should be aware that: a. If the nerves are stretched with no avulsion, they should recover completely in 3 to 6 months. b. Erb palsy is damage to the lower plexus. c. Parents of children with brachial palsy are taught to pick up the child from under the axillae. d. Breastfeeding is not recommended for infants with facial nerve paralysis until the condition resolves.

A If the nerves are stretched with no avulsion, they should recover completely in 3 to 6 months. However, if the ganglia are disconnected completely from the spinal cord, the damage is permanent. Erb palsy is damage to the upper plexus and is less serious than brachial palsy. Parents of children with brachial palsy are taught to avoid picking up the child under the axillae or by pulling on the arms. Breastfeeding is not contraindicated, but both the mother and infant will need help from the nurse at the start.

5. The nurse knows that proper placement of the tocotransducer for electronic fetal monitoring is a. Over the uterine fundus b. On the fetal scalp c. Inside uterus d. Over the mother's lower abdomen

ANS: A Feedback A The tocotransducer monitors uterine activity and should be placed over the fundus, where the most intensive uterine contractions occur. B The tocotransducer monitors uterine contractions. C The tocotransducer is for external use. D The most intensive uterine contractions occur at the fundus; this is the best placement area.

8. The nurse providing newborn stabilization must be aware that the primary side effect of maternal narcotic analgesia in the newborn is a. respiratory depression. b. bradycardia. c. acrocyanosis. d. tachypnea.

ANS: A An infant delivered within 5 hours of maternal analgesic administration (timing depends on drug used) is at risk for respiratory depression from the sedative effects of the opioid. Bradycardia, acrocyanosis, and tachypnea are not anticipated side effects of maternal analgesics. PTS: 1 DIF: Cognitive Level: Knowledge/Remembering REF: p. 365 | Table 18.1 OBJ: Nursing Process: Assessment MSC: Client Needs: Physiologic Integrity

A newborn assessment finding that would support the nursing diagnosis of postmaturity would be: a. loose skin. b. ruddy skin color. c. presence of vernix. d. absence of lanugo.

ANS: A Decreased placental function because of a prolonged pregnancy results in loss of subcutaneous tissue in the neonate, which is evidenced by loose skin. Ruddy skin color, presence of vernix, and absence of lanugo do not indicate a postmature infant.

18. A woman received an epidural anesthetic and now her blood pressure is 88/64 mm Hg. What action by the nurse takes priority? a. Turn the woman to the left side. b. Place a wedge under the woman's right hip. c. Call the provider or nurse-anesthetist immediately. d. Decrease the intravenous infusion rate.

ANS: A If hypotension occurs after administration of an epidural, turn the patient to the left lateral side-lying position, and infuse intravenous crystalloids. These actions will improve placental blood flow. Oxygen administration is also recommended, but placing the patient on the left side takes priority. The providers should be notified after corrective actions have occurred. PTS: 1 DIF: Cognitive Level: Application/Applying REF: p. 373 | Table 18.2 OBJ: Nursing Process: Implementation MSC: Client Needs: Safe and Effective Care Environment

39. Which newborn reflex is elicited by stroking the lateral sole of the infant's foot from the heel to the ball of the foot? a. Babinski c. Stepping b. Tonic neck d. Plantar grasp

ANS: A The Babinski reflex causes the toes to flare outward and the big toe to dorsiflex. The tonic neck reflex (also called the fencing reflex) refers to the posture assumed by newborns when in a supine position. The stepping reflex occurs when infants are held upright with their heel touching a solid surface and the infant appears to be walking. Plantar grasp reflex is similar to the palmar grasp reflex: when the area below the toes is touched, the infant's toes curl over the nurse's finger.

Which nutrient's recommended dietary allowance (RDA) is higher during lactation than during pregnancy? a. Energy (kcal) b. Iron c. Vitamin A d. Folic acid

ANS: A Energy (kcal) Needs for energy, protein, calcium, iodine, zinc, the B vitamins, and vitamin C remain greater than nonpregnant needs. The need for iron is not higher during lactation than pregnancy. A lactating woman does not have a greater requirement for vitamin A than a nonpregnant woman. Folic acid requirements are the highest during the first trimester.

4. The student nurse learns that breastfed babies are less likely to develop certain health conditions as adults. Which conditions does this include? (Select all that apply.) a. Diabetes b. Asthma c. Obesity d. Kidney failure e. Some cancers

ANS: A, B, C, E Breastfed infants are less likely to develop diabetes, asthma, obesity, and some cancers than bottle-fed infants. No difference is seen in the development of kidney failure.

Infants born between 34 0/7 and 36 6/7 weeks of gestation are called late preterm infants because they have many needs similar to those of preterm infants. Because they are more stable than early preterm infants, they may receive care that is much like that of a full-term baby. The mother-baby or nursery nurse knows that these babies are at increased risk for which of the following? Select all that apply. a. Problems with thermoregulation b. Cardiac distress c. Hyperbilirubinemia d. Sepsis e. Hyperglycemia

ANS: A, C, D

The preterm infant who should receive gavage feedings instead of being given a bottle is one who: a. Sometimes gags when a feeding tube is inserted. b. Has a sustained respiratory rate of 70 breaths/minute. c. Sucks on a pacifier during gavage feedings. d. Has an axillary temperature of 98.4° F, an apical pulse of 149 beats/min, and respirations of 54 breaths/minute.

ANS: B An infant with an elevated respiratory rate may aspirate if fed by nipple.

6. The nurse working with a pregnant woman explains that a major advantage of nonpharmacologic pain management is that a. more complete pain relief is possible. b. no side effects or risks to the fetus are involved. c. the woman remains fully alert at all times. d. a more rapid labor is likely.

ANS: B Because nonpharmacologic pain management does not include analgesics, adjunct drugs, or anesthesia, it is harmless to the mother and the fetus. There is less pain relief with nonpharmacologic pain management during childbirth. The woman's alertness is not altered by medication, but the increase in pain will decrease alertness. Pain management may or may not alter the length of labor. At times when pain is decreased, the mother relaxes and labor progresses at a quicker pace. PTS: 1 DIF: Cognitive Level: Comprehension/Understanding REF: p. 356 OBJ: Integrated Process: Teaching-Learning MSC: Client Needs: Physiologic Integrity

40. Infants in whom cephalhematomas develop are at increased risk for: a. Infection. c. Caput succedaneum. b. Jaundice. d. Erythema toxicum.

ANS: B Cephalhematomas are characterized by bleeding between the bone and its covering, the periosteum. Because of the breakdown of the red blood cells within a hematoma, the infants are at greater risk for jaundice. Cephalhematomas do not increase the risk for infections. Caput is an edematous area on the head from pressure against the cervix. Erythema toxicum is a benign rash of unknown cause that consists of blotchy red areas.

3. The nurse caring for women in labor understands that childbirth pain is different from other types of pain in that it is a. more responsive to pharmacologic management. b. associated with a physiologic process. c. designed to make one withdraw from the stimulus. d. less intense.

ANS: B Childbirth pain is part of a normal process, whereas other types of pain usually signify an injury or illness. Childbirth pain is not more or less responsive to medication. The pain with childbirth is a normal process; it is not caused by the type of injury when withdrawal from the stimuli is seen. Childbirth pain is not less intense than other types of pain. PTS: 1 DIF: Cognitive Level: Comprehension/Understanding REF: p. 354 OBJ: Nursing Process: Assessment MSC: Client Needs: Physiologic Integrity

7. What action by the nurse is most important to prevent the kidnapping of newborns from the hospital? a. Instruct the mother not to give her infant to anyone except the one nurse assigned to her that day. b. Question anyone who is seen walking in the hallways carrying an infant. c. Allow no visitors in the maternity area except those who have identification bracelets. d. Restrict the amount of time infants are out of the nursery.

ANS: B Infants should be transported in the hallways only in their cribs. It is impossible for one nurse to be on call for one mother and baby for the entire shift, so the parents need to be able to identify the nurses who are working on the unit. Limiting visitors may cut the new family off from vital support. Infants should be with their parents the majority of the time.

Which of the following is most helpful in preventing premature birth? a. High socioeconomic status b. Adequate prenatal care c. Aid to Families With Dependent Children d. Women, Infants, and Children nutritional program

ANS: B Prenatal care is vital in identifying possible problems.

In caring for the preterm infant, what complication is thought to be a result of high arterial blood oxygen level? a. Necrotizing enterocolitis (NEC) b. Retinopathy of prematurity (ROP) c. Small for gestational age d. Intraventricular hemorrhage (IVH)

ANS: B ROP is thought to occur as a result of high levels of oxygen in the blood

18. By knowing about variations in infants' blood count, nurses can explain to their clients that: a. A somewhat lower than expected red blood cell count could be the result of delay in clamping the umbilical cord. b. The early high white blood cell (WBC) count is normal at birth and should decrease rapidly. c. Platelet counts are higher than in adults for a few months. d. Even a modest vitamin K deficiency means a problem with the ability of the blood to clot properly.

ANS: B The WBC count is high the first day of birth and then declines rapidly. Delayed clamping of the cord results in an increase in hemoglobin and the red blood cell count. The platelet count essentially is the same for newborns and adults. Clotting is sufficient to prevent hemorrhage unless the vitamin K deficiency is significant.

24. When instructing the woman in early labor, the nurse teaches her that an important aspect of proper breathing technique is a. breathing no more than three times the normal rate. b. beginning and ending with a cleansing breath. c. holding the breath no longer than 10 seconds. d. adhering exactly to the techniques as they were taught.

ANS: B The cleansing breath helps the woman clear her mind to focus on relaxing and signals the coach that the contraction is beginning or ending. It is important to prevent hyperventilation; however, the cleansing breaths are the most important aspect of the breathing techniques. The woman should hold her breath for no more than 6 to 8 seconds. The woman needs to be flexible and change her breathing techniques as needed to keep her comfortable. PTS: 1 DIF: Cognitive Level: Comprehension/Understanding REF: p. 359 OBJ: Integrated Process: Teaching-Learning MSC: Client Needs: Psychosocial Integrity

14. A new mother is concerned because her 1-day-old newborn is taking only 1 ounce at each feeding. The nurse should explain that the a. infant does not require as much formula in the first few days of life. b. infant's stomach capacity is small at birth but will expand within a few days. c. infant tires easily during the first few days but will gradually take more formula. d. infant is probably having difficulty adjusting to the formula.

ANS: B The infant's stomach capacity at birth is 10 to 20 mL and increases to 60 to 90 mL by the end of the first week. One ounce is 30 mL. The infant's requirements are the same, but the stomach capacity needs to increase before taking in adequate amounts. The infant's sleep patterns do change, but the infant should be awake enough to feed. There are other symptoms that occur if there is a formula intolerance.

2. A new father wants to know what medication was put into his infant's eyes and why it is needed. The nurse explains to the father that the purpose of the ophthalmic ointment is to a. destroy an infectious exudate caused by Staphylococcus that could make the infant blind. b. prevent gonorrheal and chlamydial infection of the infant's eyes potentially acquired from the birth canal. c. prevent potentially harmful exudate from invading the tear ducts of the infant's eyes, leading to dry eyes. d. prevent the infant's eyelids from sticking together and help the infant see.

ANS: B The ointment is used to prevent potential gonorrheal and chlamydial infection of the infant's eyes.

13. An unfortunate but essential role of the nurse is protecting the infant from abduction. Which statement regarding the profile of a potential abductor is the most accurate? a. Male gender b. A young woman who has had a previous pregnancy loss c. A middle-aged woman past childbearing age d. A female with a number of children of her own

ANS: B The woman is usually of childbearing age and may have had a previous pregnancy loss or has been unable to have a child of her own. She may want an infant to solidify the relationship with her husband or boyfriend and may have pretended to be pregnant. The women are usually familiar with the facility and its routines.

15. A nurse is preparing a woman for a nonstress test (NST). What nursing action is most appropriate? a. Position the woman on her left side. b. Seat the woman comfortably in a recliner. c. Have the woman to drink 1 liter of water prior to the test. d. Place conduction gel on the obese woman's abdomen.

ANS: B To correctly position the pregnant patient for an NST, the woman usually sits in a reclining chair. Alternatively she can be in a semi-Fowler position with a lateral tilt. This will optimize uterine perfusion and prevent supine hypotension. The woman does not need to drink water. Conduction gel is used in all NST tests. PTS: 1 DIF: Cognitive Level: Application/Applying REF: p. 281 OBJ: Nursing Process: Implementation MSC: Client Needs: Health Promotion and Maintenance

Nurses must be aware of the conditions that increase the risk of hemorrhage, one of the most common complications of the puerperium. What are these conditions? (Select all that apply.) a. Primipara b. Rapid or prolonged labor c. Overdistention of the uterus d. Uterine fibroids e. Preeclampsia

ANS: B, C, D, E Rapid or prolonged labor, overdistention of the uterus, uterine fibroids, and preeclampsia are all risk factors for postpartum hemorrhage. Being a primipara is not a risk factor.

13. The labor of a pregnant woman with preeclampsia is going to be induced. Before initiating the Pitocin infusion, the nurse reviews the woman's latest laboratory test findings, which reveal a low platelet count, an elevated aspartate transaminase (AST) level, and a falling hematocrit. The nurse notifies the physician, because the lab results are indicative of a. Eclampsia b. Disseminated intravascular coagulation c. HELLP syndrome d. Rh incompatibility

ANS: C Feedback A Eclampsia is determined by the presence of seizures. B DIC is a potential complication associated with HELLP syndrome. C HELLP syndrome is a laboratory diagnosis for a variant of severe preeclampsia that involves hepatic dysfunction characterized by hemolysis (H), elevated liver enzymes (EL), and low platelets (LP). D These are not clinical indications of Rh incompatibility. PTS: 1 DIF: Cognitive Level: Comprehension REF: pp. 600-601 OBJ: Nursing Process: Diagnosis MSC: Client Needs: Physiologic Integrity

14. The nurse is explaining how to assess edema to the nursing students working on the antepartum unit. Which score indicates edema of lower extremities, face, hands, and sacral area? a. +1 edema b. +2 edema c. +3 edema d. +4 edema

ANS: C Feedback A Edema classified as +1 indicates minimal edema of the lower extremities. B Marked edema of the lower extremities is termed +2 edema. C Edema of the extremities, face, and sacral area is classified as +3 edema. D Generalized massive edema (+4) includes accumulation of fluid in the peritoneal cavity. PTS: 1 DIF: Cognitive Level: Knowledge REF: p. 593 | Table 25-2 OBJ: Nursing Process: Assessment MSC: Client Needs: Physiologic Integrity

23. The nurse should suspect uterine rupture if a. Fetal tachycardia occurs. b. The woman becomes dyspneic. c. Contractions abruptly stop during labor. d. Labor progresses unusually quickly.

ANS: C Feedback A Fetal tachycardia is a sign of hypoxia. With a large rupture, the nurse should be alert for the earlier signs. B This is not an early sign of a rupture. C A large rupture of the uterus will disrupt its ability to contract. D Contractions will stop with a rupture. PTS: 1 DIF: Cognitive Level: Comprehension REF: p. 661 OBJ: Nursing Process: Assessment MSC: Client Needs: Physiologic Integrity

The nurse caring for a woman in labor understands that the primary risk associated with an amniotomy is a. Maternal infection b. Maternal hemorrhage c. Prolapse of the umbilical cord d. Separation of the placenta

ANS: C Feedback A Infection is a risk of amniotomy, but not the primary concern. B Maternal hemorrhage is not associated with amniotomy. C When the membranes are ruptured, the umbilical cord may come downward with the flow of amniotic fluid and become trapped in front of the presenting part. D This may occur if the uterus is overdistended before the amniotomy, but it is not the major concern.

In caring for the postterm infant, thermoregulation can be a concern, especially in an infant who also has a(n): a. Hematocrit level of 58%. b. WBC count of 15,000 cells/mm3. c. RBC count of 5 million. d. Blood glucose level of 25 mg/dl.

ANS: D Because glucose is necessary to produce heat, the infant who is also hypoglycemic will not be able to produce enough body heat.

A client in the prenatal clinic complains of nausea and vomiting. Which intervention should the nurse suggest? a. Eat dry crackers or toast before arising in the morning. b. Consume liquids with meals. c. Eat foods high in fiber. d. Brush teeth right after eating.

Eat dry crackers or toast before arising in the morning. Rationale: Eating dry crackers or toast before arising in the morning is a good intervention for a client complaining of prenatal nausea. Foods high in fiber help with constipation problems, not with nausea. Brushing teeth after meals can trigger vomiting. Consuming liquids with meals can cause over-distention of the stomach.

Nursing intervention for pregnant clients with diabetes is based on the knowledge that the need for insulin is: a. varied depending on the stage of gestation. b. increased throughout pregnancy and the postpartum period. c. decreased throughout pregnancy and the postpartum period. d. should not change because the fetus produces its own insulin.

a. varied depending on the stage of gestation.

A client has tested HIV-positive and has now discovered that she is pregnant. Which statement indicates that she understands the risks of this diagnosis? a. "I know I will need to have an abortion as soon as possible." b. "Even though my test is positive, my baby might not be affected." c. "My baby is certain to have AIDS and die within the first year of life." d. "This pregnancy will probably decrease the chance that I will develop AIDS."

b. "Even though my test is positive, my baby might not be affected."

A client has a history of drug use and is screened for hepatitis B during the first trimester. Which action is appropriate? a. Practice respiratory isolation. b. Plan for retesting during the third trimester. c. Discuss the recommendation to bottle feed her baby. d. Anticipate administering the vaccination for hepatitis B as soon as possible.

b. Plan for retesting during the third trimester.

The labor nurse is providing care to a patient at 37 weeks' gestation who is an insulin-dependent diabetic. The health care provider prescribes an infusion of insulin throughout her induction to be titrated to keep her blood glucose levels below 110 mg/dL. What type of insulin will the nurse select to prepare the infusion? a. NPH insulin b. Regular insulin c. Lispro (Humalog) d. Aspart (Novolog)

b. Regular insulin

When planning intrapartum care for a client with heart disease, the nurse should include: a. taking vital signs according to standard protocols. b. continuously monitoring cardiac rhythm with telemetry. c. massaging the uterus to hasten birth of the placenta. d. maintaining the infusion of intravenous fluids to avoid dehydration.

b. continuously monitoring cardiac rhythm with telemetry.

The nurse is reviewing the instructions given to a patient at 24 weeks' gestation for a glucose tolerance test (GCT). The nurse determines that the patient understands the teaching when she makes which statement? a. "I have to fast the night before the test." b. "I will drink a sugary solution containing 100 grams of glucose." c. "I will have blood drawn at 1 hour after I drink the glucose solution." d. "I should keep track of my baby's movements between now and the test."

c. "I will have blood drawn at 1 hour after I drink the glucose solution."

A client who has type 2 diabetes is pregnant with her second child. She was not diagnosed with diabetes until after her first pregnancy. Past obstetric history is unremarkable—spontaneous vaginal birth of a male weighing 7 pounds, 15 ounces. The client is now concerned over what will happen during this subsequent pregnancy as a result of her disease process. What impact could the disease process have on her upcoming birth? a. Client will not be able to receive an epidural for pain management. b. Client will not be able to have a vaginal birth. c. A planned birth will be instituted by her health care provider. d. It is likely that she will deliver a fetus who is small for gestational age (SGA).

c. A planned birth will be instituted by her health care provider.

The most important instruction to include in a teaching plan for a client in early pregnancy who has Class I heart disease is she: a. must report any nausea or vomiting. b. may experience mild fatigue in early pregnancy. c. must report any chest discomfort or productive cough. d. should plan to increase her daily exercise gradually throughout pregnancy.

c. must report any chest discomfort or productive cough.

A client, who delivered her third child yesterday, has just learned that her two school-age children have contracted chickenpox. What should the nurse tell her? a. Her two children should be treated with acyclovir before she goes home from the hospital. b. The baby will acquire immunity from her and will not be susceptible to chickenpox. c. The children can visit their mother and baby in the hospital as planned but must wear gowns and masks. d. She must make arrangements to stay somewhere other than her home until the children are no longer contagious.

d. She must make arrangements to stay somewhere other than her home until the children are no longer contagious.

A client at 38 weeks' gestation tells the nurse, "I had white liquid leaking out of my nipples while warm water was running over my breasts in the shower last night." The nurse explains that: a. "You have a galactoma, and will require a brain scan." b. "You will need to pump your breasts because your milk has come in early." c. "The liquid was colostrum, just removed with warm water." d. "I will need to culture the liquid because it might indicate mastitis."

"The liquid was colostrum, just removed with warm water." Rationale: Leakage of colostrum in the third trimester is expected. Leakage of colostrum is not asign of mastitis, a galactoma, or early milk production.

14.A newborn, at 5 hours old, wakes from a sound sleep and becomes very active and begins to cry. Which of the following signs if exhibited by this newborn would indicate expected adaptation to extrauterine life? (Circle all that apply.) a. Increased mucus production b. Passage of meconium c. Heart rate of 160 beats per minute d. Respiratory rate of 24 breaths per minute and irregular e. Retraction of sternum with inspiration f. Expiratory grunting with nasal flaring

14. a, b, and c; the newborn at 5 hours old is in the second period of reactivity, during which tachycardia, tachypnea, increased muscle tone, skin color changes, increased mucus production, and passage of meconium are normal findings; temperature should range between 36.5° and 37.2° C, and respiratory rate should range between 30 and 60 BPM; expiratory grunting and nasal flaring and retractions of the sternum are signs of respiratory distress.

17. As part of a thorough assessment, the newborn should be checked for hip dislocation and dysplasia. Which of the following techniques would be used? a. Check for syndactyly bilaterally b. Stepping or walking reflex c. Magnet reflex d. Ortolani's maneuver

17. d; b and c are common newborn reflexes used to assess integrity of neuromuscular system; syndactyly refers to webbing of the finger

Following an amniotomy, the priority nursing intervention is to A assess the fetal heart rate. B assess the color and amount of amniotic fluid. C assess the maternal vital signs including temperature. D place dry sheets and pads under the woman.

A An immediate and continuing risk is that the umbilical cord will slip down in the gush of fluid. The cord can be compressed between the fetal presenting part and the woman's pelvis. Nonreassuring FHR patterns may occur. Assessing the color and amount of amniotic fluid, monitoring maternal temperature, and keeping the woman dry are important interventions but not the priority intervention.

One side effect of oxytocin stimulation is hypertonic contractions. This can be detrimental to the fetus because A there is a reduction of placental blood flow. B it produces a prolapsed cord. C it increases maternal renal blood flow. D it decreases maternal blood pressure.

A Hypertonic contractions can reduce placental blood flow and therefore reduce fetal oxygenation. Hypertonic contractions do not increase the risk for prolapsed cord, increased maternal renal blood flow, or decreased blood pressure.

Early signs of infection are often subtle and could indicate other conditions. One of the frequent signs is A. temperature instability. B. seizure activity. C. decreased pulse rate. D. ruddy skin color.

A Signs of infection include temperature instability, respiratory problems, and changes in feeding habits or behavior not seizure activity, and decreased pulse rate. Ruddy skin color is usually a sign of a high hematocrit.

The nurse noted that the woman's Bishop score was 9. This indicates that the woman A has a high likelihood of successful induction. B does not have a high likelihood of successful induction. C has a high likelihood of developing gestational diabetes. D does not have a high likelihood of developing gestational diabetes.

A The Bishop scoring system uses five factors to estimate cervical readiness for labor. A score of 8 or greater has a high level of successful induction. The Bishop scoring system does not refer to gestational diabetes.

The nurse notes that the hemoglobin level of a woman at 35 weeks of gestation is 11.5 g/dL. The nurse's next action should be to A. note that this is within the normal range for pregnancy. B. note that this is within the normal range for an average adult. C. call the physician; this shows mild anemia. D. recall that the RBC count increases slightly during pregnancy.

A The normal range of hemoglobin for pregnancy is greater than 11 g/dL in the first and third trimesters and greater than 10.5 g/dL in the second trimester. The normal range of hemoglobin for an average female is 12 to 16 g/dL. With pregnancy the levels are lower due to the increased iron requirements of the fetus. The RBC count decreases slightly because of hemodilution.

During her first prenatal visit to the clinic, a woman gives the following obstetric history: a boy born 9 years ago at full term, twin girls born 5 years ago at 36 weeks, a miscarriage at 9 weeks 2 years ago. The nurse correctly records her obstetric history as A. gravida 4, para 2, aborta 1. B. gravida 3, para 3, aborta 1. C. gravida 4, para 3, aborta 1. D. gravida 3, para 2, aborta 1.

A The woman is currently pregnant and has been pregnant 3 more times; that makes her a gravida 4. She has delivered two pregnancies after 20 weeks of gestation; that makes her a para 2. The twin girls count as one pregnancy. She delivered one pregnancy prior to 20 weeks; that makes her an aborta 1.

What intervention would make phototherapy most effective in reducing the indirect bilirubin in an affected newborn? A. Expose as much skin as possible. B. Increase oral intake of water between and before feedings. C. Place eye patches on the newborn. D. Wrap the infant in triple blankets to prevent cold stress.

A With bili lights, the infant wears only a diaper to ensure maximal exposure of the skin. The eyes are closed and patches placed over them to prevent injury. However, this is not what affects the bilirubin levels. The infant can be placed in an incubator or under a radiant warmer to maintain heat.

The abuse of which of the following substances during pregnancy is the leading cause of cognitive impairment in the United States? a. Alcohol b. Marijuana c. Tobacco d. Heroin

A Alcohol abuse during pregnancy is recognized as one of the leading causes of cognitive impairment in the United States.

Which infant would be more likely to have Rh incompatibility? a. Infant of an Rh-negative mother and a father who is Rh positive and homozygous for the Rh factor b. Infant who is Rh negative and whose mother is Rh negative c. Infant of an Rh-negative mother and a father who is Rh positive and heterozygous for the Rh factor d. Infant who is Rh positive and whose mother is Rh positive

A If the mother is Rh negative and the father is Rh positive and homozygous for the Rh factor, all the children will be Rh positive. Only Rh-positive children of an Rh-negative mother are at risk for Rh incompatibility. If the mother is Rh negative and the father is Rh positive and heterozygous for the factor, there is a 50% chance that each infant born of the union will be Rh positive and a 50% chance that each will be born Rh negative.

An infant at 26 weeks of gestation arrives intubated from the delivery room. The nurse weighs the infant, places him under the radiant warmer, and attaches him to the ventilator at the prescribed settings. A pulse oximeter and cardiorespiratory monitor are placed. The pulse oximeter is recording oxygen saturations of 80%. The prescribed saturations are 92%. The nurse's most appropriate action would be to: a. Listen to breath sounds and ensure the patency of the endotracheal tube, increase oxygen, and notify a physician. b. Continue to observe and make no changes until the saturations are 75%. c. Continue with the admission process to ensure that a thorough assessment is completed. d. Notify the parents that their infant is not doing well.

A Listening to breath sounds and ensuring the patency of the endotracheal tube, increasing oxygen, and notifying a physician are appropriate nursing interventions to assist in optimal oxygen saturation of the infant. Oxygenation of the infant is crucial. O2 saturation should be maintained above 92%. Oxygenation status of the infant is crucial. The nurse should delay other tasks to stabilize the infant. Notifying the parents that the infant is not doing well is not an appropriate action. Further assessment and intervention are warranted before determination of fetal status.

A pregnant woman was admitted for induction of labor at 43 weeks of gestation with sure dates. A nonstress test (NST) in the obstetrician's office revealed a nonreactive tracing. On artificial rupture of membranes, thick, meconium-stained fluid was noted. The nurse caring for the infant after birth should anticipate: a. Meconium aspiration, hypoglycemia, and dry, cracked skin. b. Excessive vernix caseosa covering the skin, lethargy, and respiratory distress syndrome. c. Golden yellow- to green stained-skin and nails, absence of scalp hair, and an increased amount of subcutaneous fat. d. Hyperglycemia, hyperthermia, and an alert, wide-eyed appearance.

A Meconium aspiration, hypoglycemia, and dry, cracked skin are consistent with a postmature infant. Excessive vernix caseosa covering the skin, lethargy, and respiratory distress syndrome would be consistent with a very premature infant. The skin may be meconium stained, but the infant would most likely have longer hair and decreased amounts of subcutaneous fat. Postmaturity with a nonreactive NST would indicate hypoxia. Signs and symptoms associated with fetal hypoxia are hypoglycemia, temperature instability, and lethargy.

A careful review of the literature on the various recreational and illicit drugs reveals that: a. More longer-term studies are needed to assess the lasting effects on infants when mothers have taken or are taking illegal drugs. b. Heroin and methadone cross the placenta; marijuana, cocaine, and phencyclidine (PCP) do not. c. Mothers should discontinue heroin use (detox) any time they can during pregnancy. d. Methadone withdrawal for infants is less severe and shorter than heroin withdrawal.

A Studies on the effects of marijuana and cocaine use by mothers are somewhat contradictory. More long-range studies are needed. Just about all these drugs cross the placenta, including marijuana, cocaine, and PCP. Drug withdrawal is accompanied by fetal withdrawal, which can lead to fetal death. Therefore, detoxification from heroin is not recommended, particularly later in pregnancy. Methadone withdrawal is more severe and more prolonged than heroin withdrawal.

A premature infant with respiratory distress syndrome receives artificial surfactant. How would the nurse explain surfactant therapy to the parents? a. "Surfactant improves the ability of your baby's lungs to exchange oxygen and carbon dioxide." b. "The drug keeps your baby from requiring too much sedation." c. "Surfactant is used to reduce episodes of periodic apnea." d. "Your baby needs this medication to fight a possible respiratory tract infection."

A Surfactant can be administered as an adjunct to oxygen and ventilation therapy. With administration of artificial surfactant, respiratory compliance is improved until the infant can generate enough surfactant on his or her own. Surfactant has no bearing on the sedation needs of the infant. Surfactant is used to improve respiratory compliance, including the exchange of oxygen and carbon dioxide. The goal of surfactant therapy in an infant with respiratory distress syndrome (RDS) is to stimulate production of surfactant in the type 2 cells of the alveoli. The clinical presentation of RDS and neonatal pneumonia may be similar. The infant may be started on broad-spectrum antibiotics to treat infection.

A pregnant woman presents in labor at term, having had no prenatal care. After birth her infant is noted to be small for gestational age with small eyes and a thin upper lip. The infant also is microcephalic. On the basis of her infant's physical findings, this woman should be questioned about her use of which substance during pregnancy? a. Alcohol b. Heroin c. Cocaine d. Marijuana

A The description of the infant suggests fetal alcohol syndrome, which is consistent with maternal alcohol consumption during pregnancy. Fetal brain, kidney, and urogenital system malformations have been associated with maternal cocaine ingestions. Heroin use in pregnancy frequently results in intrauterine growth restriction. The infant may have a shrill cry and sleep cycle disturbances and present with poor feeding, tachypnea, vomiting, diarrhea, hypothermia or hyperthermia, and sweating. Studies have found a higher incidence of meconium staining in infants born of mothers who used marijuana during pregnancy.

In the assessment of a preterm infant, the nurse notices continued respiratory distress even though oxygen and ventilation have been provided. The nurse should suspect: a. Hypovolemia and/or shock. b. Central nervous system injury. c. A nonneutral thermal environment. d. Pending renal failure.

A The nurse should suspect hypovolemia and/or shock. Other symptoms could include hypotension, prolonged capillary refill, and tachycardia followed by bradycardia. Intervention is necessary.

During a prenatal examination, the woman reports having two cats at home. The nurse informs her that she should not be cleaning the litter box while she is pregnant. When the woman asks why, the nurse's best response would be: a. "Your cats could be carrying toxoplasmosis. This is a zoonotic parasite that can infect you and have severe effects on your unborn child." b. "You and your baby can be exposed to the human immunodeficiency virus (HIV) in your cats' feces." c. "It's just gross. You should make your husband clean the litter boxes." d. "Cat feces are known to carry Escherichia coli, which can cause a severe infection in both you and your baby."

A Toxoplasmosis is a multisystem disease caused by the protozoal Toxoplasma gondii parasite, commonly found in cats, dogs, pigs, sheep, and cattle. About 30% of women who contract toxoplasmosis during gestation transmit the disease to their children. Clinical features ascribed to toxoplasmosis include hydrocephalus or microcephaly, chorioretinitis, seizures, or cerebral calcifications. HIV is not transmitted by cats. Although suggesting that the woman's husband clean the litter boxes may be a valid statement, it is not appropriate, does not answer the client's question, and is not the nurse's best response. E. coli is found in normal human fecal flora. It is not transmitted by cats.

The most important nursing action in preventing neonatal infection is: a. Good handwashing. b. Separate gown technique. c. Isolation of infected infants. d. Standard Precautions.

A Virtually all controlled clinical trials have demonstrated that effective handwashing is responsible for the prevention of nosocomial infection in nursery units. Measures to be taken include Standard Precautions, careful and thorough cleaning, frequent replacement of used equipment, and disposal of excrement and linens in an appropriate manner. Overcrowding must be avoided in nurseries. However, the most important nursing action for preventing neonatal infection is effective handwashing.

A woman is expecting her second child. She expressed concern to the nurse about how her 4-year-old will adapt to the new baby. The following are some suggestions the nurse should include in her teaching. Select all that apply. A. Come in and listen to the baby's heartbeat. B. Spend more time with grandmother to prepare him for being away from mother during the birth. C. Take a sibling class offered by the hospital. D. Decide which of your toys you would like to give to the new baby.

A, B, C A 4-year-old is curious about the changes in mother's body and the baby. By being included in the process, the child will not feel left out. It will also give the child an opportunity to ask questions. Children need to prepare for being away from mother during the birth and hospitalization. Starting early in the pregnancy to spend more time with the individual who will care for them will assist in the transition. Sibling classes provide an opportunity for children to discuss what newborns are like and what changes the new baby will bring to the family. Children need to be reassured that they are still maintaining an important role in the family. When they are asked to give up their possessions for the new baby, they may feel resentment.

Risk factors associated with necrotizing enterocolitis (NEC) include (Select all that apply): a. Polycythemia. b. Anemia. c. Congenital heart disease. d. Bronchopulmonary dysphasia. e. Retinopathy.

A, B, C Risk factors for NEC include asphyxia, respiratory distress syndrome, umbilical artery catheterization, exchange transfusion, early enteral feedings, patent ductus arteriosus, congenital heart disease, polycythemia, anemia, shock, and gastrointestinal infection. Bronchopulmonary dysphasia and retinopathy are not associated with NEC.

Many common drugs of abuse cause significant physiologic and behavioral problems in infants who are breastfed by mothers currently using (Select all that apply): a. Amphetamine. b. Heroin. c. Nicotine. d. PCP. e. Morphine.

A, B, C, D Amphetamine, heroin, nicotine, and PCP are contraindicated during breastfeeding because of the reported effects on the infant. Morphine is a medication that often is used to treat neonatal abstinence syndrome.

Infants born between 34 0/7 and 36 6/7 weeks of gestation are called late-preterm infants because they have many needs similar to those of preterm infants. Because they are more stable than early-preterm infants, they may receive care that is much like that of a full-term baby. The mother-baby or nursery nurse knows that these babies are at increased risk for (Select all that apply): a. Problems with thermoregulation b. Cardiac distress c. Hyperbilirubinemia d. Sepsis e. Hyperglycemia

A, C, D Thermoregulation problems, hyperbilirubinemia, and sepsis are all conditions related to immaturity and warrant close observation. After discharge the infant is at risk for rehospitalization related to these problems. AWHONN launched the Near-Term Infant Initiative to study the problem and ways to ensure that these infants receive adequate care. The nurse should ensure that this infant is feeding adequately before discharge and that parents are taught the signs and symptoms of these complications. Late-preterm infants are also at increased risk for respiratory distress and hypoglycemia.

Obstetricians today are seeing more morbidly obese pregnant women (those that weigh 400 lb or greater). A new medical subspecialty referred to as ____________ obstetrics has subsequently arisen.

ANS: Bariatric To best manage the conditions of these women and to meet their logistical needs, the subspecialty of bariatric obstetrics has been developed. Extra-wide blood pressure cuffs, surgical tables, and scales that can safely hold the weight of these clients are necessary to deliver safe care. Special techniques for ultrasound and longer surgical instruments are also required.

1. The standard of care for women who are dependent on heroin or other narcotics is ___________ maintenance treatment (MMT).

ANS: methadone Methadone maintenance treatment should be offered as part of a comprehensive care program that includes behavior therapy and support services. MMT has been shown to decrease opioid and other drug abuse, decrease criminal activity, improve individual functioning, and decrease the HIV rate.

1. A newborn infant weighing 8 lb needs naloxone (Narcan). This infant should receive approximately _____ mg.

ANS: 0.36 The dose of naloxone is 0.1 mg/kg. This baby weighs 3.6 kg, so 0.1 3.6 = 0.36 mg. PTS: 1 DIF: Cognitive Level: Application/Applying REF: Table 18.1 OBJ: Nursing Process: Implementation MSC: Client Needs: Physiologic Integrity

1. Nurses need to know that when any woman is admitted to the hospital and is _____ to _____ weeks pregnant, she should receive antenatal glucocorticoids unless she has chorioamnionitis. Because these drugs require a 24-hour period to become effective, timely administration is essential.

ANS: 24; 34 All women between 24 and 34 weeks of gestation who are at risk for preterm birth within 7 days should receive treatment with a single course of antenatal glucocorticoids. PTS: 1 DIF: Cognitive Level: Comprehension REF: p. 654 OBJ: Nursing Process: Planning MSC: Client Needs: Physiologic Integrity

1. A pregnant woman has the following assessments determined from a biophysical profile: reactive nonstress test, 3 fetal breathing movements within 30 minutes, 1 trunk movement in 30 minutes, opened and closed hand twice in 30 minutes, largest amniotic pocket of 1 cm. Calculate this woman's score. This woman's score is _____.

ANS: 8 The scoring is as follows for each criteria: 2-2-1-2-1 = 8. PTS: 1 DIF: Cognitive Level: Analysis/Analyzing REF: Table 15.1 OBJ: Nursing Process: Assessment MSC: Client Needs: Physiologic Integrity

2. __________ is defined as long, difficult, or abnormal labor. It is caused by various conditions associated with the five factors affecting labor.

ANS: Dystocia A dysfunctional labor may result from problems with the powers of labor, the passenger, the passage, the psyche or a combination of these. PTS: 1 DIF: Cognitive Level: Comprehension REF: p. 636 OBJ: Nursing Process: Evaluation MSC: Client Needs: Health Promotion and Maintenance

The NICU nurse begins her shift by assessing one of the preterm infants assigned to her care. The infant's color is pale, his O2 saturation has decreased, and he is grimacing. This infant is displaying common signs of ____.

ANS: Pain

Inquiring about past pregnancies is an important part of the nursing assessment. Women who have had a previous cesarean birth may request a trial of labor and a ______ delivery.

ANS: VBAC Although vaginal birth after cesarean is less common, it may be chosen for a variety of reasons. The nurse should be aware of the need for increased support of the woman in labor, and for complications that may occur.

3. A nurse is caring for a patient in the active phase of labor. The woman's BOW spontaneously ruptures. Suddenly the woman complains of dyspnea and appears restless and cyanotic. Additionally, she becomes hypotensive and tachycardic. The nurse immediately suspects the presence of a(n) _____________.

ANS: amniotic fluid embolism Anaphylactoid syndrome of pregnancy (ASP) is more commonly known as amniotic fluid embolism. This is a rare but devastating complication of pregnancy. It is characterized by the sudden, acute onset of hypoxia, hypotension or cardiac arrest, and coagulopathy. ASP can occur during labor, birth, or within 30 minutes after birth. This clinical presentation is similar to that observed in patients with anaphylactic or septic shock. In both of these conditions, a foreign substance is introduced into the circulation. PTS: 1 DIF: Cognitive Level: Analysis REF: p. 662 OBJ: Nursing Process: Assessment MSC: Client Needs: Physiologic Integrity TRUE/FALSE

The nurse is caring for a woman in labor at 39 weeks and 5 days of gestation. Her labor progress has slowed due to poor contractions. After discussion with the provider, a decision has been made that she is a good candidate for ___________ of labor

ANS: augmentation Augmentation of labor with oxytocin is considered when labor has begun spontaneously but progress has slowed or stopped. The rate of oxytocin may be lower than that of an induction.

3. The antidote administered to reverse magnesium toxicity is ______________.

ANS: calcium gluconate Calcium gluconate is the antidote necessary to reverse magnesium toxicity. The nurse caring for this patient should keep calcium gluconate in the room along with secured, syringes and needles. PTS: 1 DIF: Cognitive Level: Comprehension REF: p. 595 OBJ: Nursing Process: Planning MSC: Client Needs: Physiologic Integrity

In order to prevent neural tube defects, updated recommendations include an intake of 0.4 mg to 0.8 mg of ___________________ each day from one month prior to conception until 8 to 10 weeks of pregnancy.

ANS: folic acid Pregnant women should take 0.6 mg of folic acid daily for the duration of their pregnancy. Women who have given birth to an infant with a neural tube defect previously should take 4 mg of folic acid in the 4 weeks prior to pregnancy and throughout the first trimester.

Recurrent spontaneous abortion refers to a condition in which a woman experiences three or more consecutive abortions or miscarriages. This is also known as ________ abortion.

ANS: habitual Primary causes are believed to be genetic or chromosomal abnormalities of the fetus. For the mother who repeatedly aborts, the cause is often an anomaly of the reproductive tract such as bicornate uterus or incompetent cervix. Systemic illnesses such as lupus erythematosus and diabetes mellitus have been implicated in this condition as well. Treatment depends entirely on the cause and therefore varies between medical and surgical approaches. PTS: 1 DIF: Cognitive Level: Knowledge REF: p. 578 OBJ: Nursing Process: Diagnosis MSC: Client Needs: Physiologic Integrity

A popular preinduction cervical ripening agent that is Food and Drug Administration (FDA) approved for the treatment of peptic ulcers is __________.

ANS: misoprostol Cytotec This synthetic prostaglandin tablet is used primarily for the prevention of peptic ulcers. Because of its low cost, stability, and ease of use, many facilities use this medication for cervical ripening and the induction of labor. The manufacturer does not intend to seek FDA approval for other indications; however, ACOG supports its use for these purposes.

During pregnancy many women become increasingly concerned about their ability to protect and provide for the fetus. This concern is often manifested as _____________.

ANS: narcissism Narcissism is an undue preoccupation with one's self and introversion (concentration on one's self and one's body). Selecting the right foods and clothing may be more important than ever before, out of concern for the growing fetus.

Which comment by a woman in her first trimester indicates ambivalent feelings? a. "I wanted to become pregnant, but I'm scared about being a mother." b. "I haven't felt well since this pregnancy began." c. "I'm concerned about the amount of weight I've gained." d. "My body is changing so quickly."

ANS: A Feedback A Ambivalence refers to conflicting feelings. B This does not reflect conflicting feelings. C By expressing concerns over a normal occurrence, the woman is trying to confirm the pregnancy. D The woman is trying to confirm the pregnancy when she expresses concerns over normal pregnancy changes. She is not expressing conflicting feelings.

A pregnant woman's mother is worried that her daughter is not "big enough" at 20 weeks. The nurse palpates and measures the fundal height at 20 cm, which is even with the woman's umbilicus. What should the nurse report to the woman and her mother? a. "The body of the uterus is at the belly button level, just where it should be at this time." b. "You're right. We'll inform the practitioner immediately." c. "When you come for next month's appointment, we'll check you again to make sure that the baby is growing." d. "Lightening has occurred, so the fundal height is lower than expected."

ANS: A Feedback A At 20 weeks, the fundus is usually located at the umbilical level. Because the uterus grows in a predictable pattern, obstetric nurses should know that the uterus of 20 weeks of gestation is located at the level of the umbilicus. B This is incorrect information. At 20 weeks the uterus should be at the umbilical level. C By avoiding the direction question, this might increase the anxiety of both the mother and grandmother. D The descent of the fetal head (lightening) occurs in late pregnancy.

A gravida patient at 32 weeks of gestation reports that she has severe lower back pain. The nurse's assessment should include a. Observation of posture and body mechanics b. Palpation of the lumbar spine c. Exercise pattern and duration d. Ability to sleep for at least 6 hours uninterrupted

ANS: A Feedback A Correct posture and body mechanics can reduce lower back pain caused by increasing lordosis. B Pregnancy should not cause alterations in the spine. Any assessment for malformation should be done early in the pregnancy. C Certain exercises can help relieve back pain. D Rest is important for well-being, but the main concern with back pain is to assess posture and body mechanics.

A patient notices that the doctor writes "positive Chadwick's sign" on her chart. She asks the nurse what this means. The nurse's best response is a. "It refers to the bluish color of the cervix in pregnancy." b. "It means the cervix is softening." c. "The doctor was able to flex the uterus against the cervix." d. "That refers to a positive sign of pregnancy."

ANS: A Feedback A Increased vascularity of the pelvic organs during pregnancy results in the bluish color of the cervix, vagina, and labia, called Chadwick's sign. B Softening of the cervix is Goodell's sign. C The softening of the lower segment of the uterus (Hegar's sign) can allow the uterus to be flexed against the cervix. D Chadwick's sign is a probable indication of pregnancy.

The multiple marker screen is used to assess the fetus for which condition? a. Down syndrome b. Diaphragmatic hernia c. Congenital cardiac abnormality d. Anencephaly

ANS: A Feedback A The maternal serum level of alpha-fetoprotein is used to screen for Trisomy 18 or 21, neural tube defects, and other chromosomal anomalies. B The quadruple marker test does not detect this fetal anomaly. Additional testing, such as ultrasonography would be required to diagnose diaphragmatic hernia. C Congenital cardiac abnormality would most likely be identified during an ultrasound examination. D The quadruple marker test would not detect anencephaly.

8. Which factor is most likely to result in fetal hypoxia during a dysfunctional labor? a. Incomplete uterine relaxation b. Maternal fatigue and exhaustion c. Maternal sedation with narcotics d. Administration of tocolytic drugs

ANS: A Feedback A A high uterine resting tone, with inadequate relaxation between contractions, reduces maternal blood flow to the placenta and decreases fetal oxygen supply. B Maternal fatigue usually does not decrease uterine blood flow. C Maternal sedation will sedate the fetus but should not decrease blood flow. D Tocolytic drugs decrease contractions. This will increase uterine blood flow. PTS: 1 DIF: Cognitive Level: Comprehension REF: pp. 637-638 OBJ: Nursing Process: Assessment MSC: Client Needs: Physiologic Integrity

Before the physician performs an external version, the nurse should expect an order for a a. Tocolytic drug b. Contraction stress test (CST) c. Local anesthetic d. Foley catheter

ANS: A Feedback A A tocolytic drug will relax the uterus before and during version, making manipulation easier. B CST is used to determine the fetal response to stress. C A local anesthetic is not used with external version. D The bladder should be emptied, but catheterization is not necessary.

A woman at 40 weeks of gestation should be instructed to go to a hospital or birth center for evaluation when she experiences a. A trickle of fluid from the vagina b. Thick pink or dark red vaginal mucus c. Irregular contractions for 1 hour d. Fetal movement

ANS: A Feedback A A trickle of fluid from the vagina may indicate rupture of the membranes requiring evaluation for infection or cord compression. B Bloody show may occur before the onset of true labor. It does not require professional assessment unless the bleeding is pronounced. C This is a sign of false labor and does not require further assessment. D The lack of fetal movement needs further assessment.

22. What is the priority nursing assessments for a woman receiving tocolytic therapy with terbutaline? a. Fetal heart rate, maternal pulse, and blood pressure b. Maternal temperature and odor of amniotic fluid c. Intake and output d. Maternal blood glucose

ANS: A Feedback A All assessments are important, but those most relevant to the medication include the fetal heart rate and maternal pulse, which tend to increase, and the maternal blood pressure, which tends to exhibit a wide pulse pressure. B These are important if the membranes have ruptured, but they are not relevant to the medication. C This is not an important assessment to monitor for side effects of terbutaline. D This is not an important assessment to monitor for side effects of terbutaline. PTS: 1 DIF: Cognitive Level: Application REF: p. 653 OBJ: Nursing Process: Planning MSC: Client Needs: Health Promotion and Maintenance

3. Spontaneous termination of a pregnancy is considered to be an abortion if a. The pregnancy is less than 20 weeks. b. The fetus weighs less than 1000 g. c. The products of conception are passed intact. d. No evidence exists of intrauterine infection.

ANS: A Feedback A An abortion is the termination of pregnancy before the age of viability (20 weeks). B The weight of the fetus is not considered because some fetuses of an older age may have a low birth weight. C A spontaneous abortion may be complete or incomplete. D A spontaneous abortion may be caused by many problems, one being intrauterine infection. PTS: 1 DIF: Cognitive Level: Knowledge REF: p. 576 OBJ: Nursing Process: Assessment MSC: Client Needs: Health Promotion and Maintenance

13. A woman in preterm labor at 30 weeks of gestation receives two 12 mg doses of betamethasone intramuscularly. The purpose of this pharmacologic treatment is to a. Stimulate fetal surfactant production. b. Reduce maternal and fetal tachycardia associated with ritodrine administration. c. Suppress uterine contractions. d. Maintain adequate maternal respiratory effort and ventilation during magnesium sulfate therapy.

ANS: A Feedback A Antenatal glucocorticoids given as intramuscular injections to the mother accelerate fetal lung maturity. B Inderal would be given to reduce the effects of ritodrine administration. C Betamethasone has no effect on uterine contractions. D Calcium gluconate would be given to reverse the respiratory depressive effects of magnesium sulfate therapy. PTS: 1 DIF: Cognitive Level: Comprehension REF: p. 653 OBJ: Nursing Process: Planning MSC: Client Needs: Physiologic Integrity

What occurrence is associated with cervical dilation and effacement? a. Bloody show b. False labor c. Lightening d. Bladder distention

ANS: A Feedback A As the cervix begins to soften, dilate, and efface, expulsion of the mucous plug that sealed the cervix during pregnancy occurs. This causes rupture of small cervical capillaries. B Cervical dilation and effacement do not occur with false labor. C Lightening is the descent of the fetus toward the pelvic inlet before labor. D Bladder distention occurs when the bladder is not empted frequently. It may slow down the decent of the fetus during labor.

7. The priority nursing intervention when admitting a pregnant woman who has experienced a bleeding episode in late pregnancy is to a. Assess fetal heart rate (FHR) and maternal vital signs. b. Perform a venipuncture for hemoglobin and hematocrit levels. c. Place clean disposable pads to collect any drainage. d. Monitor uterine contractions.

ANS: A Feedback A Assessment of the FHR and maternal vital signs will assist the nurse in determining the degree of the blood loss and its effect on the mother and fetus. B The most important assessment is to check mother/fetal well-being. The blood levels can be obtained later. C It is important to assess future bleeding, but the top priority is mother/fetal well-being. D Monitoring uterine contractions is important, but not the top priority. PTS: 1 DIF: Cognitive Level: Application REF: p. 587 OBJ: Nursing Process: Implementation MSC: Client Needs: Health Promotion and Maintenance

27. Approximately 12% to 26% of all clinically recognized pregnancies end in miscarriage. Which is the most common cause of spontaneous abortion? a. Chromosomal abnormalities b. Infections c. Endocrine imbalance d. Immunologic factors

ANS: A Feedback A At least 60% of pregnancy losses result from chromosomal abnormalities that are incompatible with life. B Maternal infection may be a cause of early miscarriage. C Endocrine imbalances such as hypothyroidism or diabetes are possible causes for early pregnancy loss. D Women who have repeated early pregnancy losses appear to have immunologic factors the play a role in spontaneous abortion incidents. PTS: 1 DIF: Cognitive Level: Knowledge REF: p. 576 OBJ: Nursing Process: Assessment MSC: Client Needs: Health Promotion and Maintenance

12. Which assessment finding should convince the nurse to "hold" the next dose of magnesium sulfate? a. Absence of deep tendon reflexes b. Urinary output of 100 mL total for the previous 2 hours c. Respiratory rate of 14 breaths/min d. Decrease in blood pressure from 160/100 to 140/85

ANS: A Feedback A Because absence of deep tendon reflexes is a sign of magnesium toxicity, the next scheduled dose should not be administered. Calcium gluconate is the antidote that should be administered. B An hourly output of less than 30 mL could indicate toxicity. C A respiratory rate of less than 12 breaths/min could indicate toxicity. D Decrease in blood pressure is an expected side effect of magnesium sulfate. PTS: 1 DIF: Cognitive Level: Comprehension REF: p. 600 OBJ: Nursing Process: Assessment MSC: Client Needs: Physiologic Integrity

Which patient status is an acceptable indication for serial oxytocin induction of labor? a. Past 42 weeks' gestation b. Multiple fetuses c. Polyhydramnios d. History of long labors

ANS: A Feedback A Continuing a pregnancy past the normal gestational period is likely to be detrimental to fetal health. B Multiple fetuses overdistend the uterus, making induction of labor high risk. C Polyhydramnios overdistends the uterus, making induction of labor high risk. D History of rapid labors is a reason for induction of labor because of the possibility that the baby would otherwise be born in uncontrolled circumstances.

19. What order should the nurse expect for a patient admitted with a threatened abortion? a. Bed rest b. Ritodrine IV c. NPO d. Narcotic analgesia every 3 hours, prn

ANS: A Feedback A Decreasing the woman's activity level may alleviate the bleeding and allow the pregnancy to continue. B Ritodrine is not the first drug of choice for tocolytic medications. C There is no reason for having the woman NPO. At times dehydration may produce contractions, so hydration is important. D Narcotic analgesia will not decrease the contractions. It may mask the severity of the contractions. PTS: 1 DIF: Cognitive Level: Comprehension REF: p. 577 OBJ: Nursing Process: Planning MSC: Client Needs: Health Promotion and Maintenance

4. Birth for the nulliparous woman with a fetus in a breech presentation is usually by a. Cesarean delivery b. Vaginal delivery c. Forceps-assisted delivery d. Vacuum extraction

ANS: A Feedback A Delivery for the nulliparous woman with a fetus in breech presentation is almost always cesarean section. The greatest fetal risk in the vaginal delivery of breech presentation is that the head (largest part of the fetus) is the last to be delivered. The delivery of the rest of the baby must be quick so that the infant can breathe. B The greatest fetal risk in the vaginal delivery of breech presentation is that the head (largest part of the fetus) is the last to be delivered. The delivery of the rest of the baby must be quick so the infant can breathe. C The physician may assist rotation of the head with forceps. A cesarean birth may be required. D Serious trauma to maternal or fetal tissues is likely if the vacuum extractor birth is difficult. Most breech births are difficult. PTS: 1 DIF: Cognitive Level: Knowledge REF: p. 640 OBJ: Nursing Process: Assessment MSC: Client Needs: Physiologic Integrity

Which mechanism of labor occurs when the largest diameter of the fetal presenting part passes the pelvic inlet? a. Engagement b. Extension c. Internal rotation d. External rotation

ANS: A Feedback A Engagement occurs when the presenting part fully enters the pelvic inlet. B Extension occurs when the fetal head meets resistance from the tissues of the pelvic floor and the fetal neck stops under the symphysis. This causes the fetal head to extend. C Internal rotation occurs when the fetus enters the pelvic inlet. The rotation allows the longest fetal head diameter to conform to the longest diameter of the maternal pelvis. D External rotation occurs after the birth of the head. The head then turns to the side so the shoulders can internally rotate and are positioned with their transverse diameter in the anteroposterior diameter of the pelvic outlet.

While assisting with a vacuum extraction birth, what should the nurse immediately report to the physician? a. Persistent fetal bradycardia below 100 bpm b. Maternal pulse rate of 100 bpm c. Maternal blood pressure of 120/70 mm Hg d. Decrease in intensity of uterine contractions

ANS: A Feedback A Fetal bradycardia may indicate fetal distress and may require immediate intervention. B Maternal pulse rate may increase due to the pushing process. C This blood pressure is within expected norms for this stage of labor. D The birth is imminent at this point.

3. Which technique is least effective for the woman with persistent occiput posterior position? a. Lie supine and relax. b. Sit or kneel, leaning forward with support. c. Rock the pelvis back and forth while on hands and knees. d. Squat.

ANS: A Feedback A Lying supine increases the discomfort of "back labor." B A sitting or kneeling position may help the fetal head to rotate to occiput anterior. C Rocking the pelvis encourages rotation from occiput posterior to occiput anterior. D Squatting aids both rotation and fetal descent. PTS: 1 DIF: Cognitive Level: Application REF: p. 639 OBJ: Nursing Process: Implementation MSC: Client Needs: Health Promotion and Maintenance

The nurse notes that a woman who has given birth 1 hour ago is touching her infant with the fingertips and talking to him softly in high-pitched tones. On the basis of this observation, the nurse should a. Document this evidence of normal early maternal-infant attachment behavior. b. Observe for other signs that the mother may not be accepting of the infant. c. Request a social service consult for psychosocial support. d. Determine whether the mother is too fatigued to interact normally with her infant.

ANS: A Feedback A Normal early maternal-infant behaviors are tentative and include fingertip touch, eye contact, and using a high-pitched voice when talking to the infant. B These are signs of normal attachment behavior; no other assessment is necessary at this point. C There is no indication at this point that social service consult is necessary. The signs are of normal attachment behavior. D The mother may be fatigued but is interacting with the infant in an expected manner.

The nurse auscultates the fetal heart rate (FHR) and determines a rate of 152. Which nursing intervention is appropriate? a. Inform the mother that the rate is normal. b. Reassess the FHR in 5 minutes because the rate is too high. c. Report the FHR to the physician or nurse-midwife immediately. d. Tell the mother that she is going to have a boy because the heart rate is fast.

ANS: A Feedback A The FHR is within the normal range, so no other action is indicated at this time. B The FHR is within the expected range; reassessment should occur, but not in 5 minutes. C The FHR is within the expected range; no further action is necessary at this point. D The sex of the baby cannot be determined by the FHR.

To adequately care for a laboring woman, the nurse should know that the _____ stage of labor varies the most in length. a. First b. Second c. Third d. Fourth

ANS: A Feedback A The first stage of labor is considered to last from the onset of regular uterine contractions to full dilation of the cervix. The first stage is much longer than the second and third stages combined. In a first pregnancy, the first stage of labor can take up to 20 hours. B The second stage of labor lasts from the time the cervix is fully dilated to the birth of the fetus. The average length is 20 minutes for a multiparous woman and 50 minutes for a nulliparous woman. C The third stage of labor lasts from the birth of the fetus until the placenta is delivered. This stage may be as short as 3 minutes or as long as 1 hour. D The fourth stage of labor, recovery, lasts about 2 hours after delivery of the placenta.

To be aware of potential risks to the laboring woman, the nurse understands that a breech presentation is associated with a. Umbilical cord compression b. More rapid labor c. A high risk of infection d. Maternal perineal trauma

ANS: A Feedback A The umbilical cord can be compressed between the fetal body and the maternal pelvis when the body has been born but the head remains within the pelvis. B Breech presentation is not associated with a more rapid labor. C There is no higher risk of infection with a breech birth. D There is no higher risk for perineal trauma with a breech birth.

A laboring woman is lying in the supine position. The most appropriate nursing action is to a. Ask her to turn to one side. b. Elevate her feet and legs. c. Take her blood pressure. d. Determine if fetal tachycardia is present.

ANS: A Feedback A The woman's supine position may cause the heavy uterus to compress her inferior vena cava, reducing blood return to her heart and reducing placental blood flow. This problem is relieved by having her turn onto her side. B Elevating her legs will not relieve the pressure from the inferior vena cava. C This position may produce hypotension in the woman, but the action should be to prevent this from happening, not to assess for the problem. D If the woman is allowed to stay in the supine position and blood flow to the placental is reduced significantly, fetal tachycardia may occur. The most appropriate nursing action is to prevent this from occurring by turning the woman to her side.

22. An important part of fetal surveillance is assessment and documentation of the fetal heart rate during the first stage of labor. In the low-risk patient assessments for variability and periodic changes if using the fetal monitor should be done a. Every 15-30 minutes b. Every 5-15 minutes c. Every 30-60 minutes d. Only before and after ambulation

ANS: A Feedback A During the active first stage of labor, FHR should be assessed every 15-30 minutes just after a contraction. B During the second stage of labor the FHR should be assessed every 5-15 minutes. C This is not an adequate assessment during any stage of labor. D The FHR should also be evaluated both before and during ambulation.

6. Perinatal nurses are legally responsible for a. Correctly interpreting FHR patterns, initiating appropriate nursing interventions, and documenting the outcomes b. Greeting the patient on arrival, assessing her, and starting an IV line c. Applying the external fetal monitor and notifying the care provider d. Making sure the woman is comfortable

ANS: A Feedback A Nurses who care for women during childbirth are legally responsible for correctly interpreting FHR patterns, initiating appropriate nursing interventions based on those patterns, and documenting the outcomes of those interventions. B These activities should be performed when any patient arrives to the maternity unit. The nurse is not the only one legally responsible for performing these functions. C This is a nursing function that is part of the standard of care for all obstetrical patients. This falls within the RN scope of practice. D Everyone caring for the pregnant woman should ensure that both she and her support partner are comfortable.

18. Which statement correctly describes the nurse's responsibility related to electronic monitoring? a. Teach the woman and her support person about the monitoring equipment and discuss any questions they have. b. Report abnormal findings to the physician before initiating corrective actions. c. Inform the support person that the nurse will be responsible for all comfort measures when the electronic equipment is in place. d. Document the frequency, duration, and intensity of contractions measured by the external device.

ANS: A Feedback A Teaching is an essential part of the nurse's role. B Corrective actions should be initiated first in order to correct abnormal findings as quickly as possible. C The support person should still be encouraged to assist with the comfort measures. D Electronic monitoring will record the contractions and FHR response.

13. The fetal heart rate baseline increases 15 beats per minute after vibroacoustic stimulation. The best interpretation of this is that the fetus is showing a. A reassuring response b. Progressive acidosis c. Parasympathetic stimulation d. A worsening hypoxia

ANS: A Feedback A The fetus with adequate reserve for the stress of labor will usually respond to vibroacoustic stimulation with a temporary increase in the fetal heart rate (FHR) over baseline of 15 bpm for 15 seconds or more. B An increase in the FHR after stimulation is reassuring. C An increase in the FHR after stimulation is a reassuring pattern and does not indicate problems with the parasympathetic nervous system. D An increase in the FHR with stimulation does not indicate hypoxia.

10. In which situation is a baseline fetal heart rate of 160 to 170 beats per minute be considered a normal finding? a. The fetus is at 28 weeks of gestation. b. The mother has been given an epidural block. c. The mother has a history of fast labors. d. The mother has mild preeclampsia but is not in labor.

ANS: A Feedback A The normal preterm fetus may have a baseline rate slightly higher than the term fetus because of an immature parasympathetic nervous system that does not yet exert a slowing effect on the fetal heart rate (FHR). B Any change in the FHR with an epidural is not considered an expected outcome. C Fast labors should not alter the FHR normally. D Preeclampsia should not cause a normal elevation of the FHR.

2. Which maternal condition is considered a contraindication for the application of internal monitoring devices? a. Unruptured membranes b. Cervix is dilated to 4 cm c. External monitors are currently being used d. Fetus has a known heart defect

ANS: A Feedback A To apply internal monitoring devices, the membranes must be ruptured. B Cervical dilation of 4 cm permits the insertion of fetal scalp electrodes and intrauterine catheter. C The external monitor can be discontinued after the internal ones are applied. D A compromised fetus should be monitored with the most accurate monitoring devices.

To help a woman reduce the severity of nausea caused by morning sickness, the nurse might suggest that she: a. Try a tart food or drink, such as lemonade, or salty foods, such as potato chips b. Drink plenty of fluids early in the day c. Brush her teeth immediately after eating d. Never snack before bedtime

ANS: A Interestingly, some women can tolerate tart or salty foods when they are nauseated. The woman should avoid drinking too much when nausea is most likely, but she should be sure to make up the fluid levels later in the day when she feels better. The woman should avoid brushing her teeth immediately after eating. A small snack of cereal and milk or yogurt before bedtime may help the stomach in the morning.

After you complete your nutritional counseling for a pregnant woman, you ask her to repeat your instructions so you can assess her understanding of the instructions given. Which statement indicates that she understands the role of protein in her pregnancy? a. "Protein will help my baby grow." b. "Eating protein will prevent me from becoming anemic." c. "Eating protein will make my baby have strong teeth after he is born." d. "Eating protein will prevent me from being diabetic."

ANS: A Protein is the nutritional element basic to growth. An adequate protein intake is essential to meeting the increasing demands of pregnancy. These demands arise from the rapid growth of the fetus; the enlargement of the uterus, mammary glands, and placenta; the increase in the maternal blood volume; and the formation of amniotic fluid. Iron intake prevents anemia. Calcium intake is needed for fetal bone and tooth development. Glycemic control is needed in diabetics; protein is one nutritional factor to consider, but this is not the primary role of protein intake.

Which nutritional recommendation about fluids is accurate? a. A woman's daily intake should be six to eight glasses of water, milk, and/or juice. b. Coffee should be limited to no more than 2 cups, but tea and cocoa can be consumed without worry. c. Of the artificial sweeteners, only aspartame has not been associated with any maternity health concerns. d. Water with fluoride is especially encouraged because it reduces the child's risk of tooth decay.

ANS: A Six to eight glasses is still the standard for fluids; however, they should be the right fluids. All beverages containing caffeine, including tea, cocoa, and some soft drinks, should be avoided or should be drunk only in limited amounts. Artificial sweeteners, including aspartame, have no ill effects on the normal mother or fetus. However, mothers with phenylketonuria (PKU) should avoid aspartame. No evidence indicates that prenatal fluoride consumption reduces childhood tooth decay. However, it still helps the mother.

5. In counseling a patient who has decided to relinquish her baby for adoption, the nurse should a. Affirm her decision while acknowledging her maturity in making it. b. Question her about her feelings regarding adoption. c. Tell her she can always change her mind about adoption. d. Ask her if anyone is coercing her into the decision to relinquish her baby.

ANS: A Feedback A A supportive, affirming approach by the nurse will strengthen the patient's resolve and help her to appreciate the significance of the event. The teen needs help in coping with her feelings about this decision. B It is important for the nurse to support and affirm the decision the patient has made. This will strengthen the patient's resolve to follow through. Later the patient should be given an opportunity to express her feelings. C This should not be an option after the baby is born and placed with the adoptive parents. D It is important that the teenager is treated as an adult, with the assumption that she is capable of making an important decision on her own.

18. A common effect of both smoking and cocaine use on the pregnant woman is a. Vasoconstriction b. Increased appetite c. Changes in insulin metabolism d. Increased metabolism

ANS: A Feedback A Both smoking and cocaine use cause vasoconstriction, which results in impaired placental blood flow to the fetus. B Both smoking and cocaine use decrease the appetite. C Smoking and cocaine use do not change insulin metabolism. D Smoking can increase metabolism.

17. In helping bereaved parents cope and move on, nurses should keep in mind that a. A perinatal or parental grief support group is more likely to be helpful if the needs of the parents are matched with the focus of the group. b. When pictures of the infant are taken for keepsakes, no close-ups should be taken of any congenital anomalies. c. No significant differences exist in grieving individuals from various cultures, ethnic groups, and religions. d. In emergency situations, nurses who are so disposed must resist the temptation to baptize the infant in the absence of a priest or minister.

ANS: A Feedback A For example, a religious-based group may not work for nonreligious parents. B Close-up pictures of the baby must be taken as the infant was, congenital anomalies and all. C Although death and grieving are events shared by all people, mourning rituals, traditions, and taboos vary by culture, ethnicity, and religion. Differences must be respected. D Baptism for some religious groups can be performed by a layperson, such as a nurse, in an emergency situation when a priest is not available.

15. Despite warnings, prenatal exposure to alcohol continues to far exceed exposure to illicit drugs. A diagnosis of Fetal Alcohol Syndrome is made when there are visible markers in each of three categories. Which is not a recognized category for diagnosis of FAS? a. Respiratory conditions b. Impaired growth c. CNS abnormality d. Craniofacial dysmorphologies

ANS: A Feedback A Respiratory difficulties are not a category of conditions that are related to FAS. Other abnormalities related to FAS include: organ deformities, genital malformations and kidney and urinary defects. B Impaired growth is a visible marker for FAS. C A CNS abnormality with neurologic and intellectual impairments is a category used to assist in the diagnosis of FAS. D An infant with FAS manifests at least two craniofacial abnormalities such as microcephaly, short palpebral fissures, poorly developed philtrum, thin upper lip or flattening of the maxillary.

16. When the nurse is alone with a battered patient, the patient seems extremely anxious and says, "It was all my fault. The house was so messy when he got home and I know he hates that." The best response by the nurse is a. "No one deserves to be hurt. It's not your fault. How can I help you?" b. "What else do you do that makes him angry enough to hurt you?" c. "He will never find out what we talk about. Don't worry. We're here to help you." d. "You have to remember that he is frustrated and angry so he takes it out on you."

ANS: A Feedback A The nurse should stress that the patient is not at fault. B This is placing the blame on the woman. C This is false reassurance. In order to assist the woman, many times the batterer will find out about the conversation. D This is placing the blame on the woman and finding excuses for the batterer.

7. What is most likely to be a concern for the older mother? a. The importance of having enough rest and sleep b. Information about effective contraceptive methods c. Nutrition and diet planning d. Information about exercise and fitness

ANS: A Feedback A The woman who delays childbearing may have unique concerns, one of which is having less energy than younger mothers. B The older mother usually has more financial means to search out effective contraceptive methods. C The older mother is better off financially and can afford better nutrition. D Information about exercise and fitness is readily available.

13. A first-time father is changing the diaper of his 1-day-old daughter. He asks the nurse, "What is this black, sticky stuff in her diaper?" The nurse's best response is: a. "That's meconium, which is your baby's first stool. It's normal." b. "That's transitional stool." c. "That means your baby is bleeding internally." d. "Oh, don't worry about that. It's okay."

ANS: A "That's meconium, which is your baby's first stool. It's normal" is an accurate statement and the most appropriate response. Transitional stool is greenish brown to yellowish brown and usually appears by the third day after initiation of feeding. "That means your baby is bleeding internally" is not accurate. "Oh, don't worry about that. It's okay" is not an appropriate statement. It is belittling to the father and does not educate him about the normal stool patterns of his daughter.

12. A client is warm and asks for a fan in her room for her comfort. The nurse enters the room to assess the mother and her infant and finds the infant unwrapped in his crib with the fan blowing over him on "high." The nurse instructs the mother that the fan should not be directed toward the newborn and the newborn should be wrapped in a blanket. The mother asks why. The nurse's best response is: a. "Your baby may lose heat by convection, which means that he will lose heat from his body to the cooler ambient air. You should keep him wrapped and prevent cool air from blowing on him." b. "Your baby may lose heat by conduction, which means that he will lose heat from his body to the cooler ambient air. You should keep him wrapped and prevent cool air from blowing on him." c. "Your baby may lose heat by evaporation, which means that he will lose heat from his body to the cooler ambient air. You should keep him wrapped and prevent cool air from blowing on him." d. "Your baby will get cold stressed easily and needs to be bundled up at all times."

ANS: A "Your baby may lose heat by convection, which means that he will lose heat from his body to the cooler ambient air. You should keep him wrapped and prevent cool air from blowing on him" is an accurate statement. Conduction is the loss of heat from the body surface to cooler surfaces, not air, in direct contact with the newborn. Evaporation is loss of heat that occurs when a liquid is converted into a vapor. In the newborn heat loss by evaporation occurs as a result of vaporization of moisture from the skin. Cold stress may occur from excessive heat loss, but this does not imply that the infant will become stressed if not bundled at all times. Furthermore, excessive bundling may result in a rise in the infant's temperature.

20. As related to the normal functioning of the renal system in newborns, nurses should be aware that: a. The pediatrician should be notified if the newborn has not voided in 24 hours. b. Breastfed infants likely will void more often during the first days after birth. c. "Brick dust" or blood on a diaper is always cause to notify the physician. d. Weight loss from fluid loss and other normal factors should be made up in 4 to 7 days.

ANS: A A newborn who has not voided in 24 hours may have any of a number of problems, some of which deserve the attention of the pediatrician. Formula-fed infants tend to void more frequently in the first 3 days; breastfed infants void less during this time because the mother's breast milk has not come in yet. Brick dust may be uric acid crystals; blood spotting could be caused by withdrawal of maternal hormones (pseudomenstruation) or a circumcision. The physician must be notified only if there is no apparent cause of bleeding. Weight loss from fluid loss may take 14 days to regain.

31. What statement by the woman after a childbirth education class demonstrates that she needs more information? a. "I'm having a pudendal block so control my labor pain." b. "I may get a headache after a subarachnoid block." c. "I don't want IV opioids as they may cause breathing problems." d. "Some anesthetic agents may cause itching but it can be treated."

ANS: A A pudendal block numbs the lower vagina and perineum for vaginal birth. There is no relief of labor pain because it is done just before birth. This woman needs further education. The other statements are all accurate. PTS: 1 DIF: Cognitive Level: Evaluation/Evaluating REF: Table 18.2 OBJ: Integrated Process: Teaching-Learning MSC: Client Needs: Physiologic Integrity

7. A new mother states that her infant must be cold because the baby's hands and feet are blue. The nurse explains that this is a common and temporary condition called: a. Acrocyanosis. c. Harlequin color. b. Erythema neonatorum. d. Vernix caseosa.

ANS: A Acrocyanosis, or the appearance of slightly cyanotic hands and feet, is caused by vasomotor instability, capillary stasis, and a high hemoglobin level. Acrocyanosis is normal and appears intermittently over the first 7 to 10 days. Erythema toxicum (also called erythema neonatorum) is a transient newborn rash that resembles flea bites. The harlequin sign is a benign, transient color change in newborns. Half of the body is pale, and the other half is ruddy or bluish red with a line of demarcation. Vernix caseosa is a cheeselike, whitish substance that serves as a protective covering.

In providing support to a new mother who must return to full-time employment 6 weeks after a vaginal delivery, which action by the nurse is best? a. Allow her to express her positive and negative feelings freely. b. Reassure her that she'll get used to leaving her baby. c. Discuss child care arrangements with her. d. Allow her to solve the problem on her own.

ANS: A Allowing the patient to express feelings will provide positive support in her process of maternal adjustment. Simply reassuring the mother blocks further communication and belittles her feelings. Discussing child care arrangements should wait until she has expressed herself. She should be instrumental in solving the problem; however, allowing her time to express her feelings and talk the problem over will assist her in making this decision.

34. The nurse should immediately alert the physician when: a. The infant is dusky and turns cyanotic when crying. b. Acrocyanosis is present at age 1 hour. c. The infant's blood glucose level is 45 mg/dL. d. The infant goes into a deep sleep at age 1 hour.

ANS: A An infant who is dusky and becomes cyanotic when crying is showing poor adaptation to extrauterine life. Acrocyanosis is an expected finding during the early neonatal life. This is within normal range for a newborn. Infants enter the period of deep sleep when they are about 1 hour old.

4. Excessive anxiety in labor heightens the woman's sensitivity to pain by increasing a. muscle tension. b. blood flow to the uterus. c. the pain threshold. d. rest time between contractions.

ANS: A Anxiety and fear increase muscle tension, diverting oxygenated blood to the woman's brain and skeletal muscles. Prolonged tension results in general fatigue, increased pain perception, and reduced ability to use coping skills. It can also decrease blood flow to the uterus, the pain threshold, and the amount of rest the mother gets between contractions. PTS: 1 DIF: Cognitive Level: Knowledge REF: p. 356 OBJ: Nursing Process: Assessment MSC: Client Needs: Psychosocial Integrity

31. Nurses can prevent evaporative heat loss in the newborn by: a. Drying the baby after birth and wrapping the baby in a dry blanket. b. Keeping the baby out of drafts and away from air conditioners. c. Placing the baby away from the outside wall and the windows. d. Warming the stethoscope and the nurse's hands before touching the baby.

ANS: A Because the infant is wet with amniotic fluid and blood, heat loss by evaporation occurs quickly. Heat loss by convection occurs when drafts come from open doors and air currents created by people moving around. If the heat loss is caused by placing the baby near cold surfaces or equipment, it is referred to as a radiation heat loss. Conduction heat loss occurs when the baby comes in contact with cold objects or surfaces.

29. A collection of blood between the skull bone and its periosteum is known as a cephalhematoma. To reassure the new parents whose infant develops such a soft bulge, it is important that the nurse be aware that this condition: a. May occur with spontaneous vaginal birth. b. Happens only as the result of a forceps or vacuum delivery. c. Is present immediately after birth. d. Will gradually absorb over the first few months of life.

ANS: A Bleeding may occur during a spontaneous vaginal delivery as a result of the pressure against the maternal bony pelvis. The soft, irreducible fullness does not pulsate or bulge when the infant cries. Low forceps and other difficult extractions may result in bleeding. However, cephalhematomas can also occur spontaneously. The swelling may appear unilaterally or bilaterally and is usually minimal or absent at birth. It increases over the first 2 to 3 days of life. Cephalhematomas disappear gradually over 2 to 3 weeks. A less common condition results in calcification of the hematoma, which may persist for months.

27. The best reason for recommending formula over breastfeeding is that a. the mother has a medical condition or is taking drugs that could be passed along to the infant via breast milk. b. the mother lacks confidence in her ability to breastfeed. c. other family members or care providers also need to feed the baby. d. the mother sees bottle-feeding as more convenient.

ANS: A Breastfeeding is contraindicated when mothers have certain viruses, are undergoing chemotherapy, or are using/abusing drugs. Some women lack confidence in their ability to produce breast milk of adequate quantity or quality. The key to encouraging these mothers to breastfeed is anticipatory guidance beginning as early as possible in pregnancy. A major barrier for many women is the influence of family and friends. She may view formula feeding as a way to ensure that the father and other family members can participate. Each encounter with the family is an opportunity for the nurse to educate, dispel myths, and clarify information regarding the benefits of breastfeeding. Many women see bottle-feeding as more convenient and less embarrassing than breastfeeding. They may also see breastfeeding as incompatible with an active social life. There may be modesty issues related to feeding the infant in public. Although concerning, these are not legitimate reasons to formula-feed an infant. Often this decision is made without complete information regarding the benefits of breastfeeding.

2. A pregnant woman wants to breastfeed her infant; however, her husband is not convinced that there are any scientific reasons to do so. Which statement by the nurse is true? Bottle-feeding using commercially prepared infant formulas a. increases the risk that the infant will develop allergies. b. helps the infant sleep through the night. c. ensures that the infant is getting iron in a form that is easily absorbed. d. requires that multivitamin supplements be given to the infant.

ANS: A Breastfeeding is less likely to cause allergies. Newborns should be fed through the night regardless of feeding method. Iron is better absorbed from breast milk than from formula. Commercial formulas are designed to meet the nutritional needs of the infant and to resemble breast milk. No supplements are necessary.

25. According to the recommendations of the American Academy of Pediatrics (AAP) on infant nutrition a. Infants should be given only human milk for the first 6 months of life. b. Infants fed on formula should be started on solid food sooner than breastfed infants. c. If infants are weaned from breast milk before 12 months, they should receive cow's milk, not formula. d. After 6 months, mothers should shift from breast milk to cow's milk.

ANS: A Breastfeeding/human milk should also be the sole source of milk for the second 6 months. Infants start on solids when they are ready, usually at 6 months, whether they start on formula or breast milk. If infants are weaned from breast milk before 12 months, they should receive iron-fortified formula, not cow's milk. Breastfeeding/human milk should also be the sole source of milk for the second 6 months.

23. The nurse teaching a childbirth preparation class teaches the participants that the first type of breathing technique used in labor is called a. slow-paced. b. modified-paced. c. patterned-paced. d. pant-blow.

ANS: A Breathing for the first stage of labor consists of a cleansing breath and various breathing techniques known as paced breathing. The first type used in labor is the slow-paced. Modified-paced breathing is used when the slow-paced breathing is no longer effective Patterned-paced breathing is used later in the labor and has the woman focusing on a pattern of breathing. Pant-blow breathing can be used to prevent pushing before the cervix is completely dilated. PTS: 1 DIF: Cognitive Level: Comprehension/Understanding REF: p. 359 OBJ: Integrated Process: Teaching-Learning MSC: Client Needs: Health Promotion and Maintenance

22. A new mother asks if she should feed her newborn colostrum, because it is not "real milk." The nurse's best answer is that a. colostrum is high in antibodies, protein, vitamins, and minerals. b. colostrum is lower in calories than milk and should be supplemented by formula. c. giving colostrum helps the mother learn how to breastfeed. d. colostrum is unnecessary for newborns.

ANS: A Colostrum is important because it has high levels of the nutrients needed by the neonate and helps protect against infection. Supplementation is not necessary. It will decrease stimulation to the breast and decrease the production of milk. It is important for the mother to feel comfortable in this role before discharge, but the importance of the colostrum to the infant is top priority. Colostrum provides immunities and enzymes necessary to clean the gastrointestinal system, among other things.

3. The nurse providing care for the pregnant woman understands that a factor indicating the need for fetal diagnostic procedures is a. maternal diabetes. b. maternal age older than 30 years. c. previous infant more than 3000 g at birth. d. weight gain of 25 pounds.

ANS: A Diabetes is a risk factor in pregnancy because of possible impairment of placental perfusion. Other indications for testing include a maternal age greater than 35 years, having had another infant weighing greater than 4000 g at birth, or excessive weight gain. A weight gain of 25 to 35 pounds is recommended for the woman who begins pregnancy at a normal weight. PTS: 1 DIF: Cognitive Level: Knowledge/Remembering REF: Box 15.1 OBJ: Nursing Process: Assessment MSC: Client Needs: Health Promotion and Maintenance

30. A woman has received an epidural block. What action by the nurse takes priority? a. Instruct her to call for help when getting out of bed. b. Assess the woman for a post-procedure headache. c. Determine type and time of last oral intake. d. Administer metoclopramide within the first hour.

ANS: A Due to variable leg strength and sensation with an epidural block, the woman who is able to get out of bed needs to call for assistance for safety. Post-procedure headaches are associated with subarachnoid blocks. Oral intake and pro-motility agents are important for the woman having general anesthesia. PTS: 1 DIF: Cognitive Level: Application/Applying REF: Table 18.2 OBJ: Nursing Process: Implementation MSC: Client Needs: Safe and Effective Care Environment

9. A new mother asks the nurse about the "white substance" covering her infant. The nurse explains that the purpose of vernix caseosa is to a. Protect the fetal skin from amniotic fluid. b. Promote normal peripheral nervous system development. c. Allow transport of oxygen and nutrients across the amnion. d. Regulate fetal temperature.

ANS: A Feedback A Prolonged exposure to amniotic fluid during the fetal period could result in breakdown of the skin without the protection of the vernix caseosa.

1. Which part of the mature sperm contains the male chromosomes? a. The head of the sperm b. The middle portion of the sperm c. X-bearing sperm d. The tail of the sperm

ANS: A Feedback A The head of the sperm contains the male chromosomes that will join the chromosomes of the ovum.

20. The various systems and organs develop at different stages. Which statement is accurate? a. The cardiovascular system is the first organ system to function in the developing human. b. Hematopoiesis originating in the yolk sac begins in the liver at 10 weeks. c. The body changes from straight to C-shaped at 8 weeks. d. The gastrointestinal system is mature at 32 weeks.

ANS: A Feedback A The heart is developmentally complete by the end of the embryonic stage.

2. One of the assessments performed in the delivery room is checking the umbilical cord for blood vessels. Which finding is considered within normal limits? a. Two arteries and one vein b. Two arteries and two veins c. Two veins and one artery d. One artery and one vein

ANS: A Feedback A The umbilical cord contains two arteries and one vein to transport blood between the fetus and the placenta.

21. Parents have been asked by the neonatologist to provide breast milk for their newborn son, who was born prematurely at 32 weeks of gestation. The nurse who instructs them about pumping, storing, and transporting the milk needs to assess their knowledge of lactation. What statement is valid? a. A premature infant more easily digests breast milk than formula. b. A glass of wine just before pumping will help reduce stress and anxiety. c. The mother should only pump as much as the infant can drink. d. The mother should pump every 2 to 3 hours, including during the night.

ANS: A Human milk is the ideal food for preterm infants, with benefits that are unique in addition to those received by term, healthy infants. Greater physiologic stability occurs with breastfeeding compared with formula feeding. Consumption of alcohol during lactation is approached with caution. Excessive amounts can have serious effects on the infant and can adversely affect the mother's milk ejection reflex. It is generally taught that lactating mothers avoid it. To establish an optimal milk supply, the mother should be instructed to pump 8 to 10 times a day for 10 to 15 minutes on each breast. The mother should be instructed to pump 8 to 10 times a day for 10 to 15 minutes on each breast.

2. Part of the health assessment of a newborn is observing the infant's breathing pattern. A full-term newborn's breathing pattern is predominantly: a. Abdominal with synchronous chest movements. b. Chest breathing with nasal flaring. c. Diaphragmatic with chest retraction. d. Deep with a regular rhythm.

ANS: A In normal infant respiration the chest and abdomen rise synchronously, and breaths are shallow and irregular. Breathing with nasal flaring is a sign of respiratory distress. Diaphragmatic breathing with chest retraction is a sign of respiratory distress. Infant breaths are not deep with a regular rhythm.

The mother-baby nurse is able to recognize reciprocal attachment behavior. What does this refer to? a. The positive feedback an infant exhibits toward parents during the attachment process b. Behavior during the sensitive period when the infant is in the quiet alert stage c. Unidirectional behavior exhibited by the infant, initiated and enhanced by eye contact d. Behavior by the infant during the sensitive period to elicit feelings of "falling in love" from the parents

ANS: A In this definition, reciprocal refers to the feedback from the infant during the attachment process.

2. Which analysis of maternal serum may predict chromosomal abnormalities in the fetus? a. Multiple-marker screening b. Lecithin/sphingomyelin (L/S) ratio c. Biophysical profile d. Type and crossmatch of maternal and fetal serum

ANS: A Maternal serum can be analyzed for abnormal levels of alpha-fetoprotein, human chorionic gonadotropin, and estriol. This multiple-marker screening may predict chromosomal defects in the fetus. The L/S ratio is used to determine fetal lung maturity. A biophysical profile is used for evaluating fetal status during the antepartum period. Five variables are used, but none is concerned with chromosomal problems. The blood type and crossmatch will not predict chromosomal defects in the fetus. PTS: 1 DIF: Cognitive Level: Knowledge/Remembering REF: p. 278 OBJ: Nursing Process: Assessment MSC: Client Needs: Physiologic Integrity

28. The nurse providing couplet care should understand that nipple confusion results when a. breastfeeding babies receive supplementary bottle feedings. b. the baby is weaned too abruptly. c. pacifiers are used before breastfeeding is established. d. twins are breastfed together.

ANS: A Nipple confusion can result when babies go back and forth between bottles and breasts, especially before breastfeeding is established in 3 to 4 weeks, because the two require different skills. Abrupt weaning can be distressing to mother and/or baby but should not lead to nipple confusion. Pacifiers used before breastfeeding is established can be disruptive, but this does not lead to nipple confusion. Breastfeeding twins require some logistic adaptations, but this should not lead to nipple confusion.

11. Nurses can help parents deal with the issue and fact of circumcision if they explain a. the pros and cons of the procedure during the prenatal period. b. that the American Academy of Pediatrics (AAP) recommends that all newborn males be routinely circumcised. c. that circumcision is rarely painful and that any discomfort can be managed without medication. d. that the infant will likely be alert and hungry shortly after the procedure.

ANS: A Parents need to make an informed choice regarding newborn circumcision based on the most current evidence and recommendations. Health care providers and nurses who care for childbearing families should provide factual, unbiased information regarding circumcision and give parents opportunities to discuss the risks and benefits of the procedure. The AAP and other professional organizations note the benefits but stop short of recommendation for routine circumcision. Circumcision is painful and must be managed with environmental, nonpharmacologic, and pharmacologic measures. Infants may or may not be alert and hungry after the procedure.

The most important reason to protect the preterm infant from cold stress is that: a. it could make respiratory distress syndrome worse. b. shivering to produce heat may use up too many calories. c. a low temperature may make the infant less able to digest nutrients. d. cold decreases circulation to the extremities.

ANS: A Rationale: A. Cold stress may interfere with the production of surfactant, making respiratory distress syndrome worse. B. Preterm infants do not shiver to produce heat. C. Cold stress does interfere with ability to eat, but not with the ability to digest the nutrients. D. Decrease circulation is not the top priority in caring for an infant with cold stress.

18. A nurse is teaching a woman how to do "kick counts." What information about this assessment is most appropriate? a. Notify your provider if the baby's movement patterns change. b. Count the number of fetal movements over 2 hours. c. Call the OB triage area if there are fewer than 10 movements/hour. d. Have your partner verify your count at the same time you perform it.

ANS: A Since there is no consensus on how the mother should be taught to perform this assessment, it is more important that she become familiar with her baby's movements and patterns and notify the provider about any change from normal. PTS: 1 DIF: Cognitive Level: Comprehension/Understanding REF: p. 285 OBJ: Integrated Process: Teaching-Learning MSC: Client Needs: Physiologic Integrity

16. With regard to the respiratory development of the newborn, nurses should be aware that: a. The first gasping breath is an exaggerated respiratory reaction within 1 minute of birth. b. Newborns must expel the fluid from the respiratory system within a few minutes of birth. c. Newborns are instinctive mouth breathers. d. Seesaw respirations are no cause for concern in the first hour after birth.

ANS: A The first breath produces a cry. Newborns continue to expel fluid for the first hour of life. Newborns are natural nose breathers; they may not have the mouth-breathing response to nasal blockage for 3 weeks. Seesaw respirations instead of normal abdominal respirations are not normal and should be reported.

15. Nursing follow-up care often includes home visits for the new mother and her infant. Which information related to home visits is correct? a. Ideally the visit is scheduled between 24 and 72 hours after discharge. b. Home visits are available in all areas. c. Visits are completed within a 30-minute time frame. d. Blood draws are not a part of the home visit.

ANS: A The home visit is ideally scheduled during the first 24 to 72 hours after discharge. This timing allows early assessment and intervention for problems with feedings, jaundice, newborn adaptation, and maternal-infant interaction. Because home visits are expensive, they are not available in all geographic areas. Visits are usually 60 to 90 minutes in length to allow enough time for assessment and teaching. When jaundice is found, the nurse can discuss the implications and check the transcutaneous bilirubin level or draw blood for testing.

7. The best time to teach nonpharmacologic pain control methods to an unprepared laboring woman is during which phase? a. Latent phase b. Active phase c. Transition phase d. Second stage

ANS: A The latent phase of labor is the best time for intrapartum teaching, because the woman is usually anxious enough to be attentive, yet comfortable enough to understand the teaching. During the active phase, the woman is focused internally and unable to concentrate on teaching. During transition, the woman is focused on keeping control; she is unable to focus on anyone else or learn at this time. During the second stage, the woman is focused on pushing. She normally handles the pain better at this point because she is active in doing something to hasten the delivery. PTS: 1 DIF: Cognitive Level: Knowledge/Remembering REF: p. 357 OBJ: Nursing Process: Planning MSC: Client Needs: Health Promotion and Maintenance

1. A pregnant woman's biophysical profile score is 8. She asks the nurse to explain the results. The nurse's best response is a. "The test results are within normal limits." b. "Immediate delivery by cesarean birth is needed." c. "Further tests are needed to determine the meaning of this score." d. "We will inform you of your options within the next week."

ANS: A The normal biophysical score ranges from 8 to 10 points if the amniotic fluid volume is adequate. A normal score allows conservative treatment of high-risk patients. Delivery can be delayed if fetal well-being is indicated. An immediate delivery is not needed. The results of the biophysical profile are usually available immediately after the procedure is performed. PTS: 1 DIF: Cognitive Level: Comprehension/Understanding REF: p. 283 | Table 15.1 OBJ: Integrated Process: Teaching-Learning MSC: Client Needs: Health Promotion and Maintenance

19. The priority nursing intervention for the patient who has received an epidural narcotic is a. monitoring respiratory rate hourly. b. administering analgesics as needed. c. monitoring blood pressure every 4 hours. d. assessing the level of anesthesia.

ANS: A The possibility of respiratory depression exists for up to 24 hours after administration of an epidural narcotic. The nurse should monitor the woman's respiratory rate hourly during this time frame. Epidural narcotic should be enough pain relief that further medication is not necessary. Administering any other narcotic may cause an overdose. The patient's blood pressure needs to be monitored. However, that is not the major concern with this medication. The epidural narcotic should provide pain relief but not anesthesia. PTS: 1 DIF: Cognitive Level: Comprehension/Understanding REF: p. 367 OBJ: Nursing Process: Implementation MSC: Client Needs: Physiologic Integrity

22. A woman had spinal anesthesia for delivery. Now she complains of a pounding headache rated 7/10. What action by the nurse is most appropriate? a. Prepare to assist with a blood patch procedure. b. Give the woman IV opioid pain medications. c. Increase the rate of her nonadditive IV fluids. d. Place a cool cloth on her forehead and dim the room lights.

ANS: A The subarachnoid block may cause a postspinal headache due to loss of cerebrospinal fluid from the puncture in the dura. When blood is injected into the epidural space in the area of the dural puncture ("blood patch"), it forms a seal over the hole to stop leaking of cerebrospinal fluid. The spinal anesthesia makes further narcotic administration inadvisable at this time. Increasing IV fluid rate is not needed for headache. A cool cloth and dim lights may be very comforting but will not eliminate this severe headache. PTS: 1 DIF: Cognitive Level: Application/Applying REF: p. 373 | Table 18.2 OBJ: Nursing Process: Implementation MSC: Client Needs: Physiologic Integrity

21. The student nurse is working with a laboring woman. What action by the student requires the registered nurse to intervene? a. Placing the woman in a supine position b. Assisting the woman to a sitting position c. Turning the woman to a side-lying position d. Providing safety while the woman labors while standing

ANS: A The supine position allows the heavy uterus to compress the inferior vena cava and can reduce placental blood flow, compromising fetal oxygen supply. The nurse should intervene to position the woman in any of the other positions, which are all appropriate for labor if no contraindications exist. PTS: 1 DIF: Cognitive Level: Application/Applying REF: p. 363 OBJ: Nursing Process: Implementation MSC: Client Needs: Health Promotion and Maintenance

27. The nurse caring for the newborn should be aware that the sensory system least mature at the time of birth is: a. Vision. c. Smell. b. Hearing. d. Taste.

ANS: A The visual system continues to develop for the first 6 months. As soon as the amniotic fluid drains from the ear (minutes), the infant's hearing is similar to that of an adult. Newborns have a highly developed sense of smell. The newborn can distinguish and react to various tastes.

29. The mother who is pumping for an occasional bottle would be most suited for which type of breast pump? a. Manual or hand pump b. Hospital-grade pump c. Electric self-cycling double pumps d. Smaller electric or battery-operated pump

ANS: A These are the least expensive and can be the most appropriate choice for mothers pumping for the occasional bottle. Full-service electric or hospital-grade pumps most closely duplicate the sucking action of the breastfeeding infant. These are used when mother and baby (preterm or sick) are separated for long periods. Self-cycling pumps are easy to use, efficient, and designed for working mothers. Smaller pumps operated with a battery are typically used when pumping occasionally.

2. In order to help patients manage discomfort and pain during labor, nurses should be aware that a. the predominant pain of the first stage of labor is the visceral pain located in the lower portion of the abdomen. b. somatic pain is the extreme discomfort between contractions. c. the somatic pain of the second stage of labor is more generalized and related to fatigue. d. pain during the third stage is a somewhat milder version of the second stage.

ANS: A This pain comes from cervical changes, distention of the lower uterine segment, and uterine ischemia. Somatic pain is a faster, sharp pain. Somatic pain is most prominent during late first-stage labor and during second-stage labor as the descending fetus puts direct pressure on maternal tissues. Second-stage labor pain is intense, sharp, burning, and localized. Third-stage labor pain is similar to that of the first stage. PTS: 1 DIF: Cognitive Level: Knowledge/Remembering REF: p. 355 OBJ: Nursing Process: Assessment MSC: Client Needs: Health Promotion and Maintenance

23. The cheeselike, whitish substance that fuses with the epidermis and serves as a protective coating is called: a. Vernix caseosa. c. Caput succedaneum. b. Surfactant. d. Acrocyanosis.

ANS: A This protection, vernix caseosa, is needed because the infant's skin is so thin. Surfactant is a protein that lines the alveoli of the infant's lungs. Caput succedaneum is the swelling of the tissue over the presenting part of the fetal head. Acrocyanosis is cyanosis of the hands and feet that results in a blue coloring.

A woman's chart indicates she has a second-degree laceration. When assessing this patient, the nurse plans to observe which of the following structures? (Select all that apply.) a. Vaginal mucosa b. Perineal skin c. Peritoneal muscle d. Anus e. Rectum

ANS: A, B, C A second-degree perineal laceration includes vaginal mucosa, perineal skin, and peritoneal muscle. A third-degree laceration involves the anus, while a fourth-degree laceration includes the rectum.

2. Which are examples of appropriate techniques to wake a sleepy infant for breastfeeding? (Select all that apply.) a. Unwrap the infant. b. Change the diaper. c. Talk to the infant. d. Slap the infant's hands and feet. e. Apply a cold towel to the infant's abdomen.

ANS: A, B, C Unwrapping the infant, changing the diaper, or talking to the infant are appropriate ways of waking the sleeping baby. Slapping the hands and feet and applying cold towels are not appropriate methods.

Most women with uncomplicated pregnancies can use the nurse as their primary source for nutritional information. There are times when the nurse or midwife should refer a client to a registered dietitian for in-depth nutritional counseling. These pregnant women include those with: a. Preexisting or gestational illness such as diabetes b. Ethnic or cultural food patterns c. Obesity d. Vegetarian diets

ANS: A, B, C, D The nurse should be especially aware that conditions such as diabetes can require in-depth dietary planning and evaluation. To prevent issues with hypoglycemia and hyperglycemia as well as an increased risk for perinatal morbidity and mortality, this client would benefit from a referral to a dietitian. Consultation with a dietitian may ensure that cultural food beliefs are congruent with modern knowledge of fetal development and that adjustments can be made to ensure that all nutritional needs are met. The obese pregnant client may be under the misapprehension that because of her excess weight that little or no weight gain is necessary. According to the Institute of Medicine, a client with a body mass index (BMI) in the obese range should gain at least 7 kg to ensure a healthy outcome. This client may well require in-depth counseling on optimum food choices. The vegetarian client needs to have her dietary intake carefully assessed to ensure that the optimum combination of amino acids and protein intake is achieved. Very strict vegetarians (vegans) who consume only plant products may also require vitamin B and mineral supplementation.

1. Congenital disorders refer to those conditions that are present at birth. These disorders may be inherited and caused by environmental factors or maternal malnutrition. Toxic exposures have the greatest effect on development between 15 and 60 days of gestation. For the nurse to be able to conduct a complete assessment of the newly pregnant client, she should be knowledgeable regarding known human teratogens, which include (select all that apply) a. Infections b. Radiation c. Maternal conditions d. Drugs e. Chemicals

ANS: A, B, C, D Feedback Correct Exposure to radiation and a number of infections may result in profound congenital deformities. These include varicella, rubella, syphilis, parvovirus, CMV, and toxoplasmosis. Certain maternal conditions such as diabetes and PKU may also affect organs and other parts of the embryo during this developmental period. Drugs such as antiseizure medication and some antibiotics, as well as chemicals including lead, mercury, tobacco, and alcohol, also may result in structural and functional abnormalities. Incorrect Coffee is not considered a teratogen.

1. Nurses use many different nonpharmacologic methods of pain management. Examples of nonpharmacologic pain management techniques include which of the following? (Select all that apply.) a. Swaddling b. Nonnutritive sucking (pacifier) c. Skin-to-skin contact with the mother d. Sucrose e. Acetaminophen

ANS: A, B, C, D These interventions are all appropriate nonpharmacologic techniques used to manage pain in neonates. Other interventions include soothing music, dim lighting and speaking to the infant in a quiet voice. Acetaminophen is a pharmacologic method of treating pain.

1. Many teens wait until the second or even third trimester to seek prenatal care. The nurse should understand that the reasons behind this delay include (select all that apply) a. Lack of realization that they are pregnant b. Uncertainty as to where to go for care c. Continuing to deny the pregnancy d. A desire to gain control over their situation e. Wanting to hide the pregnancy as long as possible

ANS: A, B, C, E Feedback Correct These are all valid reasons for the teen to delay seeking prenatal care. An adolescent often has little to no understanding of the increased physiologic needs that a pregnancy places on her body. Once care is sought, it is often sporadic and many appointments are missed. The nurse should formulate a diagnosis that assists the pregnant teen to receive adequate prenatal care. Incorrect The opposite is true. Planning for her pregnancy and impending birth actually provides some sense of control for the teen and increases feelings of competency. Receiving praise from the nurse when she attends her prenatal appointments will reinforce the young woman's positive self-image.

1. A patient is at 6 weeks' gestation and is having a transvaginal ultrasound. While preparing the patient for this procedure, she expresses concerns over the necessity for this test. The nurse explains that this diagnostic test may be necessary to determine which of the following? (Select all that apply.) a. Multifetal gestation b. Bicornuate uterus c. Presence and location of pregnancy (intrauterine or elsewhere) d. Amniotic fluid volume e. Presence of ovarian cysts

ANS: A, B, C, E A transvaginal ultrasound done in the first trimester can detect multifetal gestations, bicornuate uterus, presence and location of pregnancy, and presence of ovarian cysts. Amniotic fluid volume is assessed during the second and third trimesters. PTS: 1 DIF: Cognitive Level: Knowledge/Remembering REF: p. 275 OBJ: Integrated Process: Teaching-Learning MSC: Client Needs: Physiologic Integrity

1. While developing an intrapartum care plan for the patient in early labor, it is important that the nurse recognize that psychosocial factors may influence a woman's experience of pain. These include (Select all that apply.) a. culture. b. anxiety and fear. c. previous experiences with pain. d. intervention of caregivers. e. support systems.

ANS: A, B, C, E Culture: a woman's sociocultural roots influence how she perceives, interprets, and responds to pain during childbirth. Some cultures encourage loud and vigorous expressions of pain, whereas others value self-control. The nurse should avoid praising some behaviors (stoicism) while belittling others (noisy expression). Anxiety and fear: extreme anxiety and fear magnify sensitivity to pain and impair a woman's ability to tolerate it. Anxiety and fear increase muscle tension in the pelvic area, which counters the expulsive forces of uterine contractions and pushing efforts. Previous experiences with pain: fear and withdrawal are a natural response to pain during labor. Learning about these normal sensations ahead of time helps a woman suppress her natural reactions of fear regarding the impending birth. If a woman previously had a long and difficult labor, she is likely to be anxious. She may also have learned ways to cope and may use these skills to adapt to the present labor experience. Support systems: an anxious partner is less able to provide help and support to a woman during labor. A woman's family and friends can be an important source of support if they convey realistic and positive information about labor and delivery. Although this may be necessary for the well-being of the woman and her fetus, some interventions add discomfort to the natural pain of labor (i.e., fetal monitor straps). PTS: 1 DIF: Cognitive Level: Knowledge/Remembering REF: p. 356 OBJ: Nursing Process: Planning MSC: Client Needs: Psychosocial Integrity

3. A nurse is discussing the signs and symptoms of mastitis with a mother who is breastfeeding. What signs and symptoms should the nurse include in her discussion? (Select all that apply.) a. Breast tenderness b. Warmth in the breast c. An area of redness on the breast often resembling the shape of a pie wedge d. A small white blister on the tip of the nipple e. Fever and flulike symptoms

ANS: A, B, C, E Signs and symptoms of mastitis include breast tenderness and warmth, an area of redness on the breast, and fever or flulike symptoms. A small white blister on the tip of the nipple is generally associated with a plugged milk duct.

A pregnant woman reports that she works in a long-term care setting and is concerned about the impending flu season. She asks about receiving the flu vaccine. As the nurse, you are aware that some immunizations are safe to administer during pregnancy, whereas others are not. Which vaccines could this patient receive? Select all that apply. a. Tetanus b. Hepatitis A and B c. Measles, mumps, rubella (MMR) d. Influenza e. Varicella

ANS: A, B, D Feedback Correct Inactivated vaccines such as those for tetanus, hepatitis A, hepatitis B, and influenza are safe to administer for women who have a risk for contracting or developing the disease. Incorrect Immunizations with live virus vaccines such as MMR, varicella (chickenpox), or smallpox are contraindicated during pregnancy because of the possible teratogenic effects on the fetus.

Throughout the world the rate of ectopic pregnancy has increased dramatically over the past 20 years. This is believed to be due primarily to scarring of the fallopian tubes as a result of pelvic infection, inflammation, or surgery. The nurse who suspects that a patient has early signs of ectopic pregnancy should be observing her for symptoms such as (select all that apply) a. Pelvic pain b. Abdominal pain c. Unanticipated heavy bleeding d. Vaginal spotting or light bleeding e. Missed period

ANS: A, B, D, E Feedback Correct A missed period or spotting can easily be mistaken by the patient as early signs of pregnancy. More subtle signs depend on exactly where the implantation occurs. The nurse must be thorough in her assessment because pain is not a normal symptom of early pregnancy. Incorrect As the fallopian tube tears open and the embryo is expelled, the patient often exhibits severe pain accompanied by intraabdominal hemorrhage. This may progress to hypovolemic shock with minimal or even no external bleeding. In about half of women, shoulder and neck pain occurs due to irritation of the diaphragm from the hemorrhage. PTS: 1 DIF: Cognitive Level: Application REF: p. 580 OBJ: Nursing Process: Assessment MSC: Client Needs: Physiologic Integrity

MULTIPLE RESPONSE The nurse who elects to practice in the area of obstetrics often hears discussion regarding the "four Ps." These are the four major factors that interact during normal childbirth. What are the "four Ps"? a. Powers b. Passage c. Position d. Passenger e. Psyche

ANS: A, B, D, E Feedback Correct Powers: the two powers of labor are uterine contractions and pushing efforts. During the first stage of labor through full cervical dilation, uterine contractions are the primary force moving the fetus through the maternal pelvis. At some point after full dilation, the woman adds her voluntary pushing efforts to propel the fetus through the pelvis. Passage: the passage for birth of the fetus consists of the maternal pelvis and its soft tissues. The bony pelvis is more important to the successful outcome of labor, because bones and joints do not yield as readily to the forces of labor. Passenger: this is the fetus plus the membranes and placenta. Fetal lie, attitude, presentation, and position are all factors that affect the fetus as passenger. Psyche: the psyche is a crucial part of childbirth. Marked anxiety, fear, or fatigue decreases the woman's ability to cope. Incorrect Position is not one of the "four Ps."

MULTIPLE RESPONSE Approximately 60% to 80% of women with one low transverse uterine incision from a previous cesarean birth may have a successful vaginal delivery. Recommendations from ACOG related to VBAC risks include (select all that apply) a. Immediate availability of the obstetric provider b. Delivery at a tertiary care center c. Availability of anesthesia personnel d. Personnel who can assist with the cesarean birth e. Use of misoprostol for cervical ripening

ANS: A, C, D Feedback Correct A VBAC delivery should only be attempted with the obstetric provider in house, and anesthesia along with operative personnel readily available to perform a cesarean birth. Incorrect VBAC deliveries may be done in community hospitals if appropriate policies and guidelines for care are in place. Misoprostol administration is contraindicated in a patient with a previous uterine scar.

2. Along with gas exchange and nutrient transfer, the placenta produces many hormones necessary for normal pregnancy. These include (select all that apply) a. Human chorionic gonadotropin (hCG) b. Insulin c. Estrogen d. Progesterone e. Testosterone

ANS: A, C, D Feedback Correct hCG causes the corpus luteum to persist and produce the necessary estrogens and progesterone for the first 6 to 8 weeks. Estrogens cause enlargement of the woman's uterus and breasts; cause growth of the ductal system in the breasts; and, as term approaches, play a role in the initiation of labor. Progesterone causes the endometrium to change, providing early nourishment. Progesterone also protects against spontaneous abortion by suppressing maternal reactions to fetal antigens and reduces unnecessary uterine contractions. Other hormones produced by the placenta include hCT , hCA, and a number of growth factors.

In some Middle Eastern and African cultures, female genital mutilation is a prerequisite for marriage. Women who now live in North America need care from nurses who are knowledgeable about the procedure and comfortable with the abnormal appearance of her genitalia. When caring for this woman, the nurse can formulate a diagnosis with the understanding that the woman may be at risk for (select all that apply) a. Obstructed labor b. Increased signs of pain response c. Laceration d. Hemorrhage e. Infection

ANS: A, C, D, E Feedback Correct The woman is at risk for all of these complications. Female genital mutilation, cutting, or circumcision involves removal of some or all of the external female genitalia. The labia majora are often stitched together over the vaginal and urethral opening as part of this practice. Enlargement of the vaginal opening may be performed before or during the birth. Incorrect The woman is unlikely to give any verbal or nonverbal signs of pain. This lack of response does not indicate lack of pain. In fact, pelvic examinations are likely to be very painful because the introitus is so small and inelastic scar tissue makes the area especially sensitive. A pediatric speculum may be necessary, and the patient should be made as comfortable as possible.

MULTIPLE RESPONSE 1. Induction of labor is considered an acceptable obstetric procedure if it is a safe time to deliver the fetus. The charge nurse on the labor and delivery unit is often asked to schedule patients for this procedure and therefore must be cognizant of the specific conditions appropriate for labor induction, including which of the following? Select all that apply. a. Rupture of membranes at or near term b. Convenience of the woman or her physician c. Chorioamnionitis (inflammation of the amniotic sac) d. Postterm pregnancy e. Fetal death

ANS: A, C, D, E Feedback Correct A, C, D, E. These are all acceptable indications for induction. Other conditions include intrauterine growth retardation (IUGR), maternal-fetal blood incompatibility, hypertension, and placental abruption. Incorrect B. Elective inductions for convenience of the woman or her provider are not recommended; however, they have become common. Factors such as rapid labors and living a long distance from a health care facility may be a valid reason in such a circumstance.

3. Auditory screening of all newborns within the first month of life is recommended by the American Academy of Pediatrics. Reasons for having this testing performed include (Select all that apply.) a. To prevent or reduce developmental delay b. Reassurance for concerned new parents c. Early identification and treatment d. To help the child communicate better e. To achieve one of the Healthy People 2020 goals

ANS: A, C, D, E These are all appropriate reasons for auditory screening of the newborn. Infants who do not pass should be rescreened. If they still do not pass the test, they should have a full audiologic and medical evaluation by 3 months of age. If necessary, the infant should be enrolled in early intervention by 6 months of age. New parents are often anxious about this test and the impending results; however, it is not the reason for the screening to be performed. Auditory screening is usually done before hospital discharge. It is important for the nurse to ensure that the infant receive the appropriate testing and that the test is fully explained to the parents. For infants who are referred for further testing and follow-up, it is important for the nurse to provide further explanation and emotional support.

During pregnancy there are a number of changes that occur as a direct result of the presence of the fetus. Which of these adaptations meet this criteria? Select all that apply. a. Leukorrhea b. Development of the operculum c. Quickening d. Ballottement e. Lightening

ANS: A, C, E Feedback Correct Leukorrhea is a white or slightly gray vaginal discharge that develops in response to cervical stimulation by estrogen and progesterone. Quickening is the first recognition of fetal movements or "feeling life." Quickening is often described as a flutter and is felt earlier in multiparous women than in primiparas. Lightening occurs when the fetus begins to descent into the pelvis. This occurs two weeks before labor in the nullipara and at the start of labor in the multipara. Incorrect Mucous fills the cervical canal creating a plug otherwise known as the operculum. The operculum acts as a barrier against bacterial invasion during the pregnancy. Passive movement of the unengaged fetus is referred to as ballottement.

A patient who has undergone a D&C for early pregnancy loss is likely to be discharged the same day. The nurse must ensure that vital signs are stable, that bleeding has been controlled, and that the woman has adequately recovered from the administration of anesthesia. In order to promote an optimal recovery, discharge teaching should include (select all that apply) a. Iron supplementation b. Resumption of intercourse at 6 weeks post-procedure c. Referral to a support group if necessary d. Expectation of heavy bleeding for at least 2 weeks e. Emphasizing the need for rest

ANS: A, C, E Feedback Correct The woman should be advised to consume a diet high in iron and protein. For many women, iron supplementation also is necessary. Acknowledge that the patient has experienced a loss, albeit early. She can be taught to expect mood swings and possibly depression. Referral to a support group, clergy, or professional counseling may be necessary. Discharge teaching should emphasize the need for rest. Incorrect Nothing should be placed in the vagina for 2 weeks postprocedure. This includes tampons and vaginal intercourse. The purpose of this recommendation is to prevent infection. Should infection occur, antibiotics may be prescribed. The patient should expect a scant, dark discharge for 1 to 2 weeks. Should heavy, profuse, or bright bleeding occur she should be instructed to contact her provider. PTS: 1 DIF: Cognitive Level: Application REF: p. 579, 583 OBJ: Nursing Process: Implementation MSC: Client Needs: Physiologic Integrity

Many women given up smoking during pregnancy to protect the health of the fetus. The majority of women resumed smoking within the first 6 months postpartum. Factors that increase the likelihood of relapse include (Select all that apply.) a. living with a smoker. b. returning to work. c. weight concerns. d. successful breastfeeding. e. failure to breastfeed.

ANS: A, C, E Living with a smoker, weight concerns, and failure to breastfeed are all associated with a higher relapse rate after smoking cessation during pregnancy.

2. The nurse teaches a student that indications for percutaneous umbilical cord sampling (PUBS) include which of the following? (Select all that apply.) a. Rh disease b. Fetal well-being c. Infection d. Lung maturity e. Karyotyping

ANS: A, C, E Rh disease, infection, and, infrequently, for karyotyping are all indications for PUBS. NST or BPP are used to determination fetal well-being. An amniocentesis is done in order to determine lung maturity. PTS: 1 DIF: Cognitive Level: Comprehension/Understanding REF: p. 281 OBJ: Integrated Process: Teaching-Learning MSC: Client Needs: Physiologic Integrity

Because late preterm infants are more stable than early preterm infants, they may receive care that is much like that of a full-term baby. The mother-baby or nursery nurse knows that these infants are at increased risk for which of the following? (Select all that apply.) a. Sepsis b. Hyperglycemia c. Hyperbilirubinemia d. Cardiac distress e. Problems with thermoregulation

ANS: A, C, E Sepsis, hyperbilirubinemia, and problems with thermoregulation are all conditions that are related to immaturity and warrant close observation. After discharge, the infant is at risk for rehospitalization related to these problems. The Association of Women's Health, Obstetric and Neonatal Nurses (AWHONN) has launched the Near-Term Infant Initiative to study the problem and determine ways to ensure that these infants receive adequate care. The nurse should ensure that this infant is feeding adequately before discharge and that parents are taught the signs and symptoms of these complications. These infants are at risk for respiratory distress and hypoglycemia

2. The nurse should model and teach practices used to prevent sudden infant death syndrome. Which of the following do these include? (Select all that apply.) a. Fully supine position for all sleep b. Side-sleeping position as an acceptable alternative c. "Tummy time" for play d. Placing the infant's crib in the parents' room e. A soft mattress

ANS: A, D The back to sleep position is now recommended as the only position for every sleep period. Ideally the infant's crib should be placed in the parents' room. Side sleeping is not an acceptable alternative because of the possibility the infant will roll to the prone position. Tummy time helps develop muscles and reduces plagiocephaly. Mattresses in cribs should be firm.

4. The nursing faculty explains that the fetus can survive in a low-oxygen environment due to which of the following? (Select all that apply.) a. Fetal hemoglobin carries more oxygen than an adult's. b. The fetus has higher average hemoglobin and hematocrit. c. Hemoglobin carries more oxygen at low partial pressures of carbon dioxide. d. Fetal blood is more acidic than the maternal blood. e. The fetus does not need gas exchange while in utero.

ANS: A,B,C The fetus can survive in low oxygen environments due to its hemoglobin being able to carry more oxygen that the mom, having a higher level of hemoglobin and hematocrit, and the fact that hemoglobin can carry more oxygen at low partial pressures of carbon dioxide. Fetal blood is alkaline. The fetus does need gas exchange in utero.

Three servings of milk, yogurt, or cheese plus two servings of meat, poultry, or fish adequately supply the recommended amount of protein for a pregnant woman. Many patients are concerned about the increased levels of mercury in fish and may be afraid to include this source of nutrients in their diet. Sound advice by the nurse to assist the client in determining which fish is safe to consume would include: a. Canned white tuna is a preferred choice b. Avoid shark, swordfish, and mackerel c. Fish caught in local waterways is the safest d. Salmon and shrimp contain high levels of mercury

ANS: B

Mimicry refers to observing and copying the behaviors of other mothers. An example might be a. Babysitting for a neighbor's children b. Wearing maternity clothes before they are needed c. Daydreaming about the newborn d. Imagining oneself as a good mother

ANS: B Feedback A Babysitting other children is a form of role playing where the woman practices the expected role of motherhood. B Wearing maternity clothes before they are needed helps the expectant mother "feel" what it's like to be obviously pregnant. C Daydreaming is a type of fantasy where the woman "tries on" a variety of behaviors in preparation for motherhood. D Imagining herself as a good mother is the woman's effort to look for a good role fit. She observes behavior of other mothers and compares them with her own expectations.

A number of cardiovascular system changes occur during pregnancy. Which finding is considered normal for a woman during pregnancy? a. Cardiac output rises by 25% b. Increased pulse rate c. Increased blood pressure d. Decreased red blood cell (RBC) production

ANS: B Feedback A Cardiac output increases by 50% with half of this rise occurring in the first 8 weeks gestation. B The pulse increases about 15 to 20 beats/min, which persists to term. C In the first trimester, blood pressure usually remains the same as the prepregnancy level, but it gradually decreases up to about 20 weeks of gestation. During the second trimester, both the systolic and diastolic pressures decrease by about 5 to 10 mm Hg. D Production of RBCs accelerates during pregnancy.

A patient in her first trimester complains of nausea and vomiting. She asks, "Why does this happen?" The nurse's best response is a. "It is due to an increase in gastric motility." b. "It may be due to changes in hormones." c. "It is related to an increase in glucose levels." d. "It is caused by a decrease in gastric secretions."

ANS: B Feedback A Gastric motility decreases during pregnancy. B Nausea and vomiting are believed to be caused by increased levels of hormones, decreased gastric motility, and hypoglycemia. C Glucose levels decrease in the first trimester. D Gastric secretions do decrease, but this is not the main cause of nausea and vomiting.

Physiologic anemia often occurs during pregnancy as a result of a. Inadequate intake of iron b. Dilution of hemoglobin concentration c. The fetus establishing iron stores d. Decreased production of erythrocytes

ANS: B Feedback A Inadequate intake of iron may lead to true anemia. B When blood volume expansion is more pronounced and occurs earlier than the increase in red blood cells, the woman will have physiologic anemia, which is the result of dilution of hemoglobin concentration rather than inadequate hemoglobin. C If the woman does not take an adequate amount of iron, true anemia may occur when the fetus pulls stored iron from the maternal system. D There is an increased production of erythrocytes during pregnancy.

The nurse who practices in a prenatal clinic understands that a major concern of lower socioeconomic groups is to a. Maintain group health insurance on their families. b. Meet health needs as they occur. c. Practice preventive health care. d. Maintain an optimistic view of life.

ANS: B Feedback A Lower socioeconomic groups usually do not have group health insurances. B Because of economic uncertainty, lower socioeconomic groups place more emphasis on meeting the needs of the present rather than on future goals. C They may value health care, but cannot afford preventive health care. D They may struggle for basic needs and often do not see a way to improve their situation. It is difficult to maintain optimism.

Early pregnancy classes offered in the first and second trimesters cover a. Phases and stages of labor b. Coping with common discomforts of pregnancy c. Methods of pain relief d. Predelivery and postdelivery care of the patient having a cesarean delivery

ANS: B Feedback A Phases and stages of labor are taught in childbirth preparation classes. B Early pregnancy classes focus on the first two trimesters and cover information on adapting to pregnancy, dealing with early discomforts, and understanding what to expect in the months ahead. C Pain control is part of childbirth preparation classes. D This is taught in cesarean birth preparation classes.

The maternity nurse understands that vascular volume increases 40% to 60% during pregnancy to a. Compensate for decreased renal plasma flow. b. Provide adequate perfusion of the placenta. c. Eliminate metabolic wastes of the mother. d. Prevent maternal and fetal dehydration.

ANS: B Feedback A Renal plasma flow increases during pregnancy. B The primary function of increased vascular volume is to transport oxygen and nutrients to the fetus via the placenta. C Assisting with pulling metabolic wastes from the fetus for maternal excretion is one purpose of the increased vascular volume. D This is not the primary reason for the increase in volume.

Which complaint by a patient at 35 weeks of gestation requires additional assessment? a. Shortness of breath when climbing stairs b. Abdominal pain c. Ankle edema in the afternoon d. Backache with prolonged standing

ANS: B Feedback A Shortness of breath is an expected finding by 35 weeks. B Abdominal pain may indicate preterm labor or placental abruption. C Ankle edema in the afternoon is a normal finding at this stage of pregnancy. D Backaches while standing is a normal finding during the later stages of pregnancy.

Which situation best describes a man "trying on" fathering behaviors? a. Spending more time with his siblings b. Coaching a Little League baseball team c. Reading books on newborn care d. Exhibiting physical symptoms related to pregnancy

ANS: B Feedback A The man normally will seek closer ties with his father. B Interacting with children and assuming the behavior and role of a father best describes a man "trying on" being a father. C Men do not normally read information that is provided in advance. The nurse should be prepared to present the information after the baby is born, when it is more relevant. D This is called couvade.

Which finding in the urine analysis of a pregnant woman is considered a variation of normal? a. Proteinuria b. Glycosuria c. Bacteria in the urine d. Ketones in the urine

ANS: B Feedback A The presence of protein could indicate kidney disease or preeclampsia. B Small amounts of glucose may indicate "physiologic spilling." C Urinary tract infections are associated with bacteria in the urine. D An increase in ketones indicates that the patient is exercising too strenuously or has an inadequate fluid and food intake.

During the active phase of labor, the FHR of a low-risk patient should be assessed every a. 15 minutes b. 30 minutes c. 45 minutes d. 1 hour

ANS: B Feedback A 15-minute assessments are appropriate for a fetus at high risk. B For the fetus at low risk for complications, guidelines for frequency of assessments are at least every 30 minutes during the active phase of labor. C 45-minute assessments during the active phase of labor is not frequent enough to monitor for complications. D 1-hour assessments during the active phase of labor are not frequent enough to monitor for complications.

9. A woman with severe preeclampsia is being treated with bed rest and intravenous magnesium sulfate. The drug classification of this medication is a. Tocolytic b. Anticonvulsant c. Antihypertensive d. Diuretic

ANS: B Feedback A A tocolytic drug does slow the frequency and intensity of uterine contractions but is not used for that purpose in this scenario. B Anticonvulsant drugs act by blocking neuromuscular transmission and depress the central nervous system to control seizure activity. C Decreased peripheral blood pressure is a therapeutic response (side effect) of the anticonvulsant magnesium sulfate. D Diuresis is a therapeutic response to magnesium sulfate. PTS: 1 DIF: Cognitive Level: Comprehension REF: p. 594 OBJ: Nursing Process: Assessment MSC: Client Needs: Physiologic Integrity

4. An abortion in which the fetus dies but is retained in the uterus is called _____ abortion. a. Inevitable b. Missed c. Incomplete d. Threatened

ANS: B Feedback A An inevitable abortion means that the cervix is dilating with the contractions. B Missed abortion refers to a dead fetus being retained in the uterus. C An incomplete abortion means that not all of the products of conception were expelled. D With a threatened abortion the woman has cramping and bleeding but not cervical dilation. PTS: 1 DIF: Cognitive Level: Knowledge REF: p. 578 OBJ: Nursing Process: Assessment MSC: Client Needs: Physiologic Integrity

20. An important independent nursing action to promote normal progress in labor is a. Assessing the fetus b. Encouraging urination about every 1 to 2 hours c. Limiting contact with the woman's partner d. Regulating intravenous fluids

ANS: B Feedback A Assessment of the fetus is an important task, but will not promote normal progression of labor. B The bladder can reduce room in the woman's pelvis that is needed for fetal descent and can increase her discomfort. C The woman needs her support system during labor, and contact should not be limited. D Maintaining hydration is an important task, but it will not promote normal progression of labor. PTS: 1 DIF: Cognitive Level: Comprehension REF: p. 641 OBJ: Nursing Process: Implementation MSC: Client Needs: Health Promotion and Maintenance

24. Which laboratory marker is indicative of disseminated intravascular coagulation (DIC)? a. Bleeding time of 10 minutes b. Presence of fibrin split products c. Thrombocytopenia d. Hyperfibrinogenemia

ANS: B Feedback A Bleeding time in DIC is normal. B Degradation of fibrin leads to the accumulation of multiple fibrin clots throughout the body's vasculature. C Low platelets may occur with but are not indicative of DIC because they may result from other coagulopathies. D Hypofibrinogenemia occurs with DIC. PTS: 1 DIF: Cognitive Level: Knowledge REF: p. 578 OBJ: Nursing Process: Assessment MSC: Client Needs: Physiologic Integrity

To adequately teach patients about the process of labor, the nurse knows that which event is the best indicator of true labor? a. Bloody show b. Cervical dilation and effacement c. Fetal descent into the pelvic inlet d. Uterine contractions every 7 minutes

ANS: B Feedback A Bloody show can occur before true labor. B The conclusive distinction between true and false labor is that contractions of true labor cause progressive change in the cervix. C Fetal descent can occur before true labor. D False labor may have contractions that occur this frequently, but it is usually inconsistent.

The primary difference between the labor of a nullipara and that of a multipara is the a. Amount of cervical dilation b. Total duration of labor c. Level of pain experienced d. Sequence of labor mechanisms

ANS: B Feedback A Cervical dilation is the same for all labors. B Multiparas usually labor more quickly than nulliparas, making the total duration of their labor shorter. C Level of pain is individual to the woman, not to the number of labors she has experienced. D The sequence of labor mechanisms is the same with all labors.

11. Why is adequate hydration important when uterine activity occurs before pregnancy is at term? a. Fluid and electrolyte imbalance can interfere with the activity of the uterine pacemakers. b. Dehydration may contribute to uterine irritability for some women. c. Dehydration decreases circulating blood volume, which leads to uterine ischemia. d. Fluid needs are increased because of increased metabolic activity occurring during contractions.

ANS: B Feedback A Fluid and electrolyte imbalances are not associated with preterm labor. B Intravenous fluids are ordered according to their expected benefit. Adequate hydration promotes urination and decreased risk for infection. C The woman has an increase blood volume during pregnancy. D Fluid needs do not increase due to contractions. PTS: 1 DIF: Cognitive Level: Comprehension REF: p. 651 OBJ: Nursing Process: Assessment MSC: Client Needs: Physiologic Integrity

A 25-year-old primigravida is in the first stage of labor. She and her husband have been holding hands and breathing together through each contraction. Suddenly the woman pushes her husband's hand away and shouts, "Don't touch me!" This behavior is most likely a. Normal and related to hyperventilation b. Common during the transition phase of labor c. A sign that she needs analgesia d. Indicative of abnormal labor

ANS: B Feedback A Hyperventilation will produce signs of respiratory alkalosis. B The transition phase of labor is often associated with an abrupt change in behavior, including increased anxiety and irritability. C If she is in the transitional phase of labor, analgesia may not be appropriate if the birth is near. D This change of behavior is an expected occurrence during the transition phase.

Which maternal condition always necessitates delivery by cesarean section? a. Partial abruptio placentae b. Total placenta previa c. Ectopic pregnancy d. Eclampsia

ANS: B Feedback A If the mother has stable vital signs and the fetus is alive, a vaginal delivery can be attempted. If the fetus has died, a vaginal delivery is preferred. B In total placenta previa, the placenta completely covers the cervical os. The fetus would die if a vaginal delivery occurred. C The most common ectopic pregnancy is a tubal pregnancy, which is usually detected and treated in the first trimester. D Labor can be safely induced if the eclampsia is under control. PTS: 1 DIF: Cognitive Level: Comprehension REF: p. 583, 585 OBJ: Nursing Process: Assessment MSC: Client Needs: Physiologic Integrity

Which statement is the best rationale for assessing maternal vital signs between contractions? a. During a contraction, assessing fetal heart rates is the priority. b. Maternal circulating blood volume increases temporarily during contractions. c. Maternal blood flow to the heart is reduced during contractions. d. Vital signs taken during contractions are not accurate.

ANS: B Feedback A It is important to monitor fetal response to contractions, but the question is concerned with the maternal vital signs. B During uterine contractions, blood flow to the placenta temporarily stops, causing a relative increase in the mother's blood volume, which in turn temporarily increases blood pressure and slows pulse. C Maternal blood flow is increased during a contraction. D Vital signs are altered by contractions but are considered accurate for that period of time.

24. Rupture of the amniotic sac before the onset of true labor, regardless of length of gestation is called premature rupture of membranes (PROM). The first priority for the nurse is to determine whether membranes are truly ruptured. Other explanations for this increase in fluid discharge include all except a. Urinary incontinence b. Leaking of amniotic fluid c. Loss of mucous plug d. An increase in vaginal discharge

ANS: B Feedback A It is not uncommon for patients to mistake urinary incontinence for leakage of amniotic fluid. B Leaking of amniotic fluid is an indication of PROM. C Loss of the mucous plug can lead a woman to believe that her membranes have ruptured when they have not. D Late in pregnancy there may be an increase in vaginal discharge. This may be mistaken for rupture of membranes. PTS: 1 DIF: Cognitive Level: Application REF: p. 644 OBJ: Nursing Process: Assessment MSC: Client Needs: Physiologic Integrity

An indication for an episiotomy would be a woman who a. Has a routine vaginal birth b. Has fetal shoulder dystocia c. Is delivering a preterm infant d. Has a history of rapid deliveries

ANS: B Feedback A Once routine for all vaginal deliveries, the perceived benefits of reducing pain and perineal tearing have not proven true. B An episiotomy is indicated in the situation where the shoulder of the fetus become lodged under the mother symphysis during birth. C A preterm infant is smaller and does not need as much space for delivery. D Rapid deliveries are not an indication for a mediolateral episiotomy.

20. What data on a patient's health history places her at risk for an ectopic pregnancy? a. Use of oral contraceptives for 5 years b. Recurrent pelvic infections c. Ovarian cyst 2 years ago d. Heavy menstrual flow of 4 days' duration

ANS: B Feedback A Oral contraceptives do not increase the risk for ectopic pregnancies. B Infection and subsequent scarring of the fallopian tubes prevents normal movement of the fertilized ovum into the uterus for implantation. C Ovarian cysts do not cause scarring of the fallopian tubes. D This will not cause scarring of the fallopian tubes, which is the main risk factor for ectopic pregnancies. PTS: 1 DIF: Cognitive Level: Comprehension REF: p. 580 | Box 25-1 OBJ: Nursing Process: Assessment MSC: Client Needs: Physiologic Integrity

For which patient should the oxytocin (Pitocin) infusion be discontinued immediately? a. A woman in active labor with contractions every 31 minutes lasting 60 seconds each b. A woman in transition with contractions every 2 minutes lasting 90 seconds each c. A woman in active labor with contractions every 2 to 3 minutes lasting 70 to 80 seconds each d. A woman in early labor with contractions every 5 minutes lasting 40 seconds each

ANS: B Feedback A Oxytocin may assist this woman's contractions to become closer and more efficient. B This woman's contraction pattern represents hyperstimulation, and inadequate resting time occurs between contractions to allow placental perfusion. C There is an appropriate resting period between this woman's contractions. D There is an appropriate resting period between this woman's contractions for her stage of labor.

Which maternal factor may inhibit fetal descent and require further nursing interventions? a. Decreased peristalsis b. A full bladder c. Reduction in internal uterine size d. Rupture of membranes

ANS: B Feedback A Peristalsis does not influence fetal descent. B A full bladder may inhibit fetal descent because it occupies space in the pelvis needed by the fetal presenting part. C Contractions will reduce the internal uterine size in order to assist fetal descent. D Rupture of membranes will assist in the fetal descent.

2. A woman in labor at 34 weeks of gestation is hospitalized and treated with intravenous magnesium sulfate for 18 to 20 hours. When the magnesium sulfate is discontinued, which oral drug will probably be prescribed for at-home continuation of the tocolytic effect? a. Ritodrine b. Terbutaline c. Calcium gluconate d. Magnesium sulfate

ANS: B Feedback A Ritodrine is the only drug approved by the FDA for tocolysis; however, it is rarely used because of significant side effects. B The woman receiving decreasing doses of magnesium sulfate is often switched to oral terbutaline to maintain tocolysis. C Calcium gluconate reverses magnesium sulfate toxicity. The drug should be available for complications of magnesium sulfate therapy. D Magnesium sulfate is usually given intravenously or intramuscularly. The patient must be hospitalized for magnesium therapy because of the serious side effects of this drug. PTS: 1 DIF: Cognitive Level: Knowledge REF: p. 653 OBJ: Nursing Process: Planning MSC: Client Needs: Physiologic Integrity

If a woman's fundus is soft 30 minutes after birth, the nurse's first response should be to a. Take the blood pressure. b. Massage the fundus. c. Notify the physician or nurse-midwife. d. Place the woman in Trendelenburg position.

ANS: B Feedback A The blood pressure is an important assessment to determine the extent of blood loss, but it is not the top priority. B The nurse's first response should be to massage the fundus to stimulate contraction of the uterus to compress open blood vessels at the placental site, limiting blood loss. C Notification should occur after all nursing measures have been attempted with no favorable results. D Trendelenburg position is contraindicated for this woman at this point. This position does not allow for appropriate vaginal drainage of lochia. The lochia remaining in the uterus would clot and produce further bleeding.

When positioning the Foley catheter before cesarean birth, the nurse knows that the catheter drainage tube and catheter bag should be a. Positioned on top of the patient's leg b. Placed near the head of the table c. Clamped during the cesarean section d. Positioned at the foot of the surgeon under the sterile drapes

ANS: B Feedback A The drain tube of the catheter should be positioned under her leg to promote drainage and to keep the catheter away from the operative area. B The anesthesia clinician must monitor urine output during the surgery. C Urinary output must be continuously monitored. An early sign of hypovolemia is a decreasing urinary output. D The surgeon might step on the drainage bag if the catheter was below the drapes, and no one could monitor the urine output.

It is important for the nurse providing care during labor to be aware that pregnant women can usually tolerate the normal blood loss associated with childbirth because they have a. A higher hematocrit b. Increased blood volume c. A lower fibrinogen level d. Increased leukocytes

ANS: B Feedback A The hematocrit decreases with pregnancy due to the high fluid volume. B Women have a significant increase in blood volume during pregnancy. After delivery, the additional circulating volume is no longer necessary. C Fibrinogen levels increase with pregnancy. D Leukocyte levels increase during labor, but that is not the reason for the toleration of blood loss.

To teach and support the woman in labor, the nurse explains that the strongest part of a labor contraction is the a. Increment b. Acme c. Decrement d. Interval

ANS: B Feedback A The increment is the beginning of the contractions until it reaches the peak. B The acme is the peak or period of greatest strength during the middle of a contraction cycle. C The decrement occurs after the peak until the contraction ends. D The interval is the period between the end of the contraction and the beginning of the next.

A patient whose cervix is dilated to 5 cm is considered to be in which phase of labor? a. Latent phase b. Active phase c. Second stage d. Third stage

ANS: B Feedback A The latent phase is from the beginning of true labor until 3 cm of cervical dilation. B The active phase of labor is characterized by cervical dilation of 4 to 7 cm. C The second stage of labor begins when the cervix is completely dilated until the birth of the baby. D The third stage of labor is from the birth of the baby until the expulsion of the placenta.

After a forceps-assisted birth, the mother is observed to have continuous bright red lochia but a firm fundus. What other data indicates the presence of a potential vaginal wall hematoma? a. Mild, intermittent perineal pain b. Edema and discoloration of the labia and perineum c. Lack of an episiotomy d. Lack of pain in the perineal area

ANS: B Feedback A The pain with vaginal hematoma is severe and constant. B The nurse should monitor for edema and discoloration. Using a cold application to the labia and perineum reduces pain by numbing the area and limiting bruising and edema for the first 12 hours. C An episiotomy is performed as the fetal head distends the perineum. D The pain associated with vaginal hematoma is severe.

Which event indicates a complication of an external version? a. Maternal pulse rate of 100 bpm b. Fetal bradycardia persisting 10 minutes after the version c. Fetus returning to the original position d. Increased maternal anxiety after the version

ANS: B Feedback A There are few risks to the woman during an external version. B Fetal bradycardia after a version may indicate that the umbilical cord has become compressed, and the fetus is having hypoxia. C The fetus may return to the original position, but this is not a complication of the version. D Anxiety may occur before the version but should decrease after the procedure is completed.

What is an appropriate response to a woman's comment that she is worried about having a cesarean birth? a. "Don't worry. Everything will be okay." b. "What are your feelings about having a cesarean birth?" c. "I know you're worried, but this is a routine procedure." d. "Patients commonly worry about surgery."

ANS: B Feedback A This answer is stating that the patient's feelings are not important. B Allowing the patient to express her feelings is the most appropriate nursing response. The nurse should never provide the patient with false reassurance or disregard her feelings. C This is belittling the patient's concerns and does not allow her to express her concerns. D This answer is close ended and belittling to the patient's feelings.

A woman who is gravida 3 para 2 enters the intrapartum unit. The most important nursing assessments are a. Contraction pattern, amount of discomfort, and pregnancy history b. Fetal heart rate, maternal vital signs, and the woman's nearness to birth c. Identification of ruptured membranes, the woman's gravida and para, and her support person d. Last food intake, when labor began, and cultural practices the couple desires

ANS: B Feedback A This is an important nursing assessment, but does not take priority if the birth is imminent. B All options describe relevant intrapartum nursing assessments, but the focus assessment has priority. If the maternal and fetal conditions are normal and birth is not imminent, other assessments can be performed in an unhurried manner. C This is an assessment that can occur later in the admission process if time permits. D This part of the assessment can occur later in the admission process if time permits.

To assess the duration of labor contractions, the nurse determines the time a. From the beginning of one contraction to the beginning of the next b. From the beginning to the end of each contraction c. Of the strongest intensity of each contraction d. Of uterine relaxation between two contractions

ANS: B Feedback A This is the frequency of the contractions. B Duration of labor contractions is the average length of contractions from beginning to end. C This is the strength or intensity of the contractions. D This is the interval of the contraction phase.

The nurse knows that a urinary catheter is added to the instrument table if a forceps-assisted birth is anticipated. What is the correct rationale for this intervention? a. Spontaneous release of urine might contaminate the sterile field. b. An empty bladder provides more room in the pelvis. c. A sterile urine specimen is needed preoperatively. d. A Foley catheter prevents the membranes from spontaneously rupturing.

ANS: B Feedback A Urine is sterile. B Catheterization provides room for the application of the forceps blades and limits bladder trauma. C A clean-catch urinalysis is usually sufficient for preoperative treatment. D The membranes must be ruptured and the cervix completely dilated for a forceps-assisted birth.

8. A pregnant woman is being discharged from the hospital after placement of a cerclage because of a history of recurrent pregnancy loss secondary to an incompetent cervix. Discharge teaching should emphasize that a. Any vaginal discharge should be reported immediately to her care provider. b. The presence of any contractions, rupture of membranes, or severe perineal pressure should be reported. c. She will need to make arrangements for care at home, because her activity level will be restricted. d. She will be scheduled for a cesarean birth.

ANS: B Feedback A Vaginal bleeding needs to be reported to her primary care provider. B Nursing care should stress the importance of monitoring signs and symptoms of preterm labor. C Bed rest is an element of care. However, the woman may stand for periods of up to 90 minutes, which allows her the freedom to see her physician. Home uterine activity monitoring may be used to limit the woman's need for visits and to safely monitor her status at home. D The cerclage can be removed at 37 weeks of gestation (to prepare for a vaginal birth), or a cesarean birth can be planned. PTS: 1 DIF: Cognitive Level: Application REF: p. 579 OBJ: Nursing Process: Planning and Implementation MSC: Client Needs: Health Promotion and Maintenance

A maternal indication for the use of vacuum extraction is a. A wide pelvic outlet b. Maternal exhaustion c. A history of rapid deliveries d. Failure to progress past 0 station

ANS: B Feedback A With a wide pelvic outlet, vacuum extraction is not necessary. B With a mother who is exhausted will be unable to assist with the expulsion of the fetus. C With a rapid delivery, vacuum extraction is not necessary. D A station of 0 is too high for a vacuum extraction.

8. Why is continuous electronic fetal monitoring usually used when oxytocin is administered? a. The mother may become hypotensive. b. Uteroplacental exchange may be compromised. c. Maternal fluid volume deficit may occur. d. Fetal chemoreceptors are stimulated.

ANS: B Feedback A Hypotension is not a common side effect of oxytocin. B The uterus may contract more firmly, and the resting tone may be increased with oxytocin use. This response reduces entrance of freshly oxygenated maternal blood into the intervillous spaces, depleting fetal oxygen reserves. C All laboring women are at risk for fluid volume deficit; oxytocin administration does not increase the risk. D Oxytocin affects the uterine muscles.

7. Which is the most appropriate method of intrapartum fetal monitoring when a woman has a history of hypertension during pregnancy? a. Continuous auscultation with a fetoscope b. Continuous electronic fetal monitoring c. Intermittent assessment with a Doppler transducer d. Intermittent electronic fetal monitoring for 15 minutes each hour

ANS: B Feedback A It is not practical to provide continuous auscultation with a fetoscope. B Maternal hypertension may reduce placental blood flow through vasospasm of the spiral arteries. Reduced placental perfusion is best assessed with continuous electronic fetal monitoring to identify patterns associated with this condition. C This fetus needs continuous monitoring because it is at high risk for complications. D This fetus needs continuous monitoring because it is at high risk for complications.

1. The nurse sees a pattern on the fetal monitor that looks similar to early decelerations, but the deceleration begins near the acme of the contraction and continues well beyond the end of the contraction. Which nursing action indicates the proper evaluation of this situation? a. Continue to monitor these early decelerations, which occur as the fetal head is compressed during a contraction. b. This deceleration pattern is associated with uteroplacental insufficiency, so the nurse acts quickly to improve placental blood flow and fetal oxygen supply. c. This pattern reflects variable decelerations. No interventions are necessary at this time. d. Document this reassuring fetal heart rate pattern, but decrease the rate of the intravenous fluid.

ANS: B Feedback A These are late decelerations, not early; therefore interventions are necessary. B This is a description of a late deceleration. Oxygen should be given via snug facemask. Position the woman on her left side to increase placental blood flow. C Variable decelerations are caused by cord compression. A vaginal examination should be performed to identify this potential emergency. D This is not a reassuring pattern, so the intravenous rate should be increased to increase the mother's blood volume.

17. Assessment of a woman's nutritional status includes a diet history, medication regimen, physical examination, and relevant laboratory tests. A maternity nurse performing such an assessment should be aware that: a. Oral contraceptive use may interfere with the absorption of iron b. Illnesses that have created nutritional deficits, such as phenylketonuria (PKU), may require nutritional care before conception c. The woman's socioeconomic status and educational level are not relevant to her examination; they are the province of the social worker d. The only nutrition-related laboratory test most pregnant women need is testing for diabetes

ANS: B A registered dietitian can help with therapeutic diets. Oral contraceptive use may interfere with the absorption of folic acid. Iron deficiency can appear if placement of an intrauterine device (IUD) results in blood loss. A woman's finances affect her access to good nutrition; her education (or lack thereof) can influence the nurse's teaching decisions. The nutrition-related laboratory test that pregnant women usually need is a screen for anemia.

To prevent gastrointestinal upset, clients should be instructed to take iron supplements: a. On a full stomach b. At bedtime c. After eating a meal d. With milk

ANS: B Taking iron supplements at bedtime may reduce GI upset. Iron supplements are best absorbed if they are not taken on a full stomach. Iron supplements are best absorbed if they are taken when the stomach is empty. Iron can be taken at bedtime if abdominal discomfort occurs when it is taken between meals. Bran, tea, coffee, milk, and eggs may reduce absorption. Iron can be taken at bedtime if abdominal discomfort occurs when it is taken between meals.

2. During which phase of the cycle of violence does the batterer become contrite and remorseful? a. Battering phase b. Honeymoon phase c. Tension-building phase d. Increased drug-taking phase

ANS: B Feedback A During the battering phase violence actually occurs, and the victim feels powerless. B During the honeymoon phase, the battered person wants to believe that the battering will never happen again, and the batterer will promise anything to get back into the home. C During the tension-building phase, the batterer becomes increasingly hostile, swears, threatens, throws things, and pushes the battered. D Often the batterer increases the use of drugs during the tension-building phase.

14. A woman who is 6 months pregnant has sought medical attention, saying she fell down the stairs. What scenario would cause an emergency department nurse to suspect that the woman has been battered? a. The woman and her partner are having an argument that is loud and hostile. b. The woman has injuries on various parts of her body that are in different stages of healing. c. Examination reveals a fractured arm and fresh bruises. Her husband asks her about her pain. d. She avoids making eye contact and is hesitant to answer questions.

ANS: B Feedback A This is not always an indication of battering. Many times the batterer will be attentive and refuse to leave the woman's bedside. B The battered woman often has multiple injuries in various stages of healing. C With battering there are injuries in various stages of healing. D It is more normal for the woman to have a flat affect.

9. The parents of a newborn ask the nurse how much the newborn can see. The parents specifically want to know what type of visual stimuli they should provide for their newborn. The nurse responds to the parents by telling them: a. "Infants can see very little until about 3 months of age." b. "Infants can track their parent's eyes and distinguish patterns; they prefer complex patterns." c. "The infant's eyes must be protected. Infants enjoy looking at brightly colored stripes." d. "It's important to shield the newborn's eyes. Overhead lights help them see better."

ANS: B "Infants can track their parent's eyes and distinguish patterns; they prefer complex patterns" is an accurate statement. Development of the visual system continues for the first 6 months of life. Visual acuity is difficult to determine, but the clearest visual distance for the newborn appears to be 19 cm. Infants prefer to look at complex patterns, regardless of the color. Infants prefer low illumination and withdraw from bright light.

31. A woman wants to breastfeed, but her nipples are inverted and she is concerned it won't be possible. What does the nurse teach the woman about this condition? a. A woman with inverted nipples rarely is successful at breastfeeding. b. You can use a breast pump just prior to feeding to evert the nipples. c. Massage the breasts prior to feeding to allow milk let-down. d. Try changing the infant's position during feedings.

ANS: B A breast pump can be used just prior to feeding. As soon as the suction everts the nipple, the woman needs to place the baby to the breast. Women with inverted nipples can breastfeed. Massage and changing the infant's position will not affect the inverted nipples.

5. The nurse's initial action when caring for an infant with a slightly decreased temperature is to a. notify the physician immediately. b. place a cap on the infant's head. c. Keep the infant in the nursery for the next 4 hours. d. Assess for other signs of inaccurate gestational age.

ANS: B A cap will prevent further heat loss from the head, and having the mother place the infant skin-to-skin should increase the infant's temperature. Nursing actions are needed first to correct the problem. If the problem persists after interventions, notification may then be necessary. A slightly decreased temperature can be treated in the mother's room. This would be an excellent time for parent teaching on prevention of cold stress. There is no need for another gestational age assessment.

During which stage of role attainment do the parents become acquainted with their baby and combine parenting activities with cues from the infant? a. Anticipatory b. Formal c. Informal d. Personal

ANS: B A major task of the formal stage of role attainment is getting acquainted with the infant. The anticipatory stage begins during the pregnancy when the parents choose a physician and attend childbirth classes. The informal stage begins once the parents have learned appropriate responses to their infant's cues. The personal stage is attained when parents feel a sense of harmony in their role.

10. The nerve block used in labor that provides anesthesia to the lower vagina and perineum is called a(n) a. epidural. b. pudendal. c. local. d. spinal block.

ANS: B A pudendal block anesthetizes the lower vagina and perineum to provide anesthesia for an episiotomy and use of low forceps if needed. An epidural provides anesthesia for the uterus, perineum, and legs. A local provides anesthesia for the perineum at the site of the episiotomy. A spinal block provides anesthesia for the uterus, perineum, and down the legs. PTS: 1 DIF: Cognitive Level: Knowledge/Remembering REF: p. 372 | Table 18.2 OBJ: Nursing Process: Assessment MSC: Client Needs: Physiologic Integrity

When caring for a newly delivered woman, the nurse is aware that the best measure to prevent abdominal distention after a cesarean birth is a. rectal suppositories. b. early and frequent ambulation. c. tightening and relaxing abdominal muscles. d. providing carbonated beverages.

ANS: B Activity can aid the movement of accumulated gas in the gastrointestinal tract so early, and frequent ambulation is the best option. Rectal suppositories can be helpful after distention occurs but do not prevent it. Tightening and relaxing the abdominal muscles is not related. Carbonated beverages may increase distention.

5. The primary reason for evaluating alpha-fetoprotein (AFP) levels in maternal serum is to determine if the fetus has a. hemophilia. b. a neural tube defect. c. sickle cell anemia. d. a normal lecithin/sphingomyelin (L/S) ratio.

ANS: B An open neural tube allows a high level of AFP to seep into the amniotic fluid and enter the maternal serum. Hemophilia is a genetic defect and is best detected with chromosomal studies such as chorionic villus sampling or amniocentesis. Sickle cell is a genetic defect and is best detected with chromosomal studies such as chorionic villus sampling or amniocentesis. L/S ratios are determined with an amniocentesis, which is usually done in the third trimester. PTS: 1 DIF: Cognitive Level: Knowledge/Remembering REF: p. 277 OBJ: Nursing Process: Assessment MSC: Client Needs: Physiologic Integrity

25. Which patient is most likely to experience pain during labor? a. Gravida 2 who has not attended childbirth preparation classes b. Gravida 2 who is anxious because her last labor was difficult c. Gravida 1 whose fetus is in a breech presentation d. Gravida 3 who is using Lamaze breathing techniques

ANS: B Anxiety affects a woman's perception of pain. Tension during labor causes tightening of abdominal muscles, impeding contractions and increasing pain by stimulation of nerve endings. The gravida 2 has previous experience, and this will decrease anxiety. This woman will have more pain than if the infant is in vertex. Also, there is an increased likelihood that she will have a cesarean section and not go through labor. The gravida 3 has previous experience and has prepared herself for the labor. PTS: 1 DIF: Cognitive Level: Comprehension/Understanding REF: p. 356 OBJ: Nursing Process: Assessment MSC: Client Needs: Psychosocial Integrity

41. Plantar creases should be evaluated within a few hours of birth because: a. The newborn has to be footprinted. b. As the skin dries, the creases will become more prominent. c. Heel sticks may be required. d. Creases will be less prominent after 24 hours.

ANS: B As the infant's skin begins to dry, the creases will appear more prominent, and the infant's gestation could be misinterpreted. Footprinting will not interfere with the creases. Heel sticks will not interfere with the creases. The creases will appear more prominent after 24 hours.

To assess fundal contraction 6 hours after cesarean delivery, the nurse should a. palpate forcefully through the abdominal dressing. b. gently palpate, applying the same technique used for vaginal deliveries. c. place hands on both sides of the abdomen and press downward. d. rely on assessment of lochial flow rather than palpating the fundus.

ANS: B Assessment of the fundus is the same for both vaginal and cesarean deliveries; however, palpation should be gentle due to increased discomfort caused by the uterine incision. Forceful palpation should never be used. The top of the fundus, not the sides, should be palpated and massaged. The fundus should be palpated and massaged to prevent bleeding.

To promote bonding and attachment immediately after delivery, what action by the nurse is most important? a. Allow the mother quiet time with her infant. b. Assist the mother in assuming an en face position with her newborn. c. Teach the mother about the concepts of bonding and attachment. d. Assist the mother in feeding her baby.

ANS: B Assisting the mother in assuming an en face position with her newborn will support the bonding process. Quiet time with the infant is helpful but not as important as en face positioning. The mother has just delivered and is more focused on the infant; she will not be receptive to teaching at this time. This is a good time to initiate breastfeeding, but this is not as specific to bonding and attachment as the en face position.

A man calls the nurse's station stating that his wife, who delivered 2 days ago, is happy one minute and crying the next. The man says, "She was never like this before the baby was born." What response by the nurse is best? a. Tell him to ignore the mood swings, as they will go away. b. Reassure him that this behavior is normal. c. Advise him to get immediate psychological help for her. d. Instruct him in the signs, symptoms, and duration of postpartum blues.

ANS: B Before providing further instructions, inform family members of the fact that postpartum blues are a normal process to allay anxieties and increase receptiveness to learning. Telling him the mood swings will go away is belittling his concerns. Postpartum blues are a normal process that is short lived; no medical intervention is needed. Client teaching is important; however, his anxieties need to be allayed before he will be receptive to teaching.

14. The transition period between intrauterine and extrauterine existence for the newborn: a. Consists of four phases, two reactive and two of decreased responses. b. Lasts from birth to day 28 of life. c. Applies to full-term births only. d. Varies by socioeconomic status and the mother's age.

ANS: B Changes begin right after birth; the cutoff time when the transition is considered over (although the baby keeps changing) is 28 days. The transition period has three phases: first reactivity, decreased response, and second reactivity. All newborns experience this transition regardless of age or type of birth. Although stress can cause variation in the phases, the mother's age and wealth do not disturb the pattern.

30. The nurse notes a new mother is waiting until her newborn begins crying prior to breastfeeding her. What response by the nurse is best? a. Praise the mother for her efforts to nurse. b. Teach the mother signs of hunger in the newborn. c. Inform the mother she is inhibiting bonding. d. Demonstrate calming methods prior to feeding.

ANS: B Crying is a late sign of hunger in the newborn. The nurse should teach the mother other signs of hunger so the baby will be more ready to eat when the mother attempts to feed the baby. Of course the nurse should praise all attempts at breastfeeding, but this is not the best response. Telling the mother she is inhibiting bonding will discourage her. The nurse should also demonstrate calming methods, but the goal is to feed the infant when he or she displays early signs of hunger.

32. The nurse notices an infant has dimpling of the cheeks when breastfeeding. What action by the nurse is best? a. Tell the mother this is a sign of adequate feeding. b. Have the mother remove the baby from the breast and try again. c. Make a referral for a lactation consultation. d. Instruct the mother to feed for at least 15 minutes.

ANS: B Dimpling the cheeks is a sign of an infant-derived problem during breastfeeding. The nurse should have the mother gently remove the baby from the breast, reposition the infant if needed, and try to get the baby to latch on correctly so she can try again. The nurse may need to call for a lactation consultant, but all mother-baby or labor and delivery nurses should be able to provide basic assistance first. This is not a sign of adequate feeding so the nurse should not encourage her to keep going for another 15 minutes.

9. Which nursing intervention is necessary before a second trimester transabdominal ultrasound? a. Place the woman NPO for 12 hours. b. Instruct the woman to drink 1 to 2 quarts of water. c. Administer a soapsuds enema. d. Perform an abdominal prep.

ANS: B During the second trimester, a full bladder may be needed to displace the intestines and elevate the uterus for better visibility. If indicated, the woman should be instructed to drink several glasses of clear fluid an hour before the time of the examination and to delay urination until the examination is completed. Since she needs to fill her bladder, being NPO is not appropriate. Enemas and abdominal preps are not necessary for this procedure. PTS: 1 DIF: Cognitive Level: Knowledge/Remembering REF: p. 276 OBJ: Nursing Process: Implementation MSC: Client Needs: Physiologic Integrity

11. To prevent breast engorgement, the new breastfeeding mother should be instructed to a. apply cold packs to the breast before feeding. b. breastfeed frequently and for adequate lengths of time. c. limit her intake of fluids for the first few days. d. feed her infant no more than every 4 hours.

ANS: B Engorgement occurs when the breasts are not adequately emptied at each feeding or if feedings are not frequent enough. Warm packs should be applied to the breast before feedings. Fluid intake should not be limited with a breastfeeding mother; that will decrease the amount of breast milk produced. Breast milk moves through the stomach within 1.5 to 2 hours, so waiting 4 hours to feed is too long. Frequent feedings are important to empty the breast and to establish lactation.

10. A woman who is 16 weeks pregnant asks the nurse, "Is it possible to tell by ultrasound if the baby is a boy or girl yet?" The best answer is a. "A baby's sex is determined as soon as conception occurs." b. "The baby has developed enough that we can determine the sex by examining the genitals through ultrasound." c. "Boys and girls look alike until approximately 20 weeks after conception, and then they begin to look different." d. "It might be possible to determine your baby's sex, but the external organs look very similar right now."

ANS: B Feedback B Although gender is determined at conception, the external genitalia of males and females look similar through the 9th week. By the 12th week, the external genitalia are distinguishable as male or female.

12. A patient is sent from the physician's office for assessment because of too little amniotic fluid. The nurse is aware that oligohydramnios can result in a. Excessive fetal urine secretion b. Newborn respiratory distress c. Central nervous system abnormality d. Gastrointestinal blockage

ANS: B Feedback B Because an abnormally small amount of amniotic fluid restricts normal lung development, the infant may have inadequate respiratory function after birth, when the placenta no longer performs respiratory function.

7. A woman is 16 weeks pregnant with her first baby. She asks how long it will be before she feels the baby move. The best answer is a. "You should have felt the baby move by now." b. "Within the next month, you should start to feel fluttering sensations." c. "The baby is moving, but you can't feel it yet." d. "Some babies are quiet, and you don't feel them move."

ANS: B Feedback B Maternal perception of fetal movement usually begins 17 to 20 weeks after conception.

19. With regard to the structure and function of the placenta, the maternity nurse should be aware that a. As the placenta widens, it gradually thins to allow easier passage of air and nutrients. b. As one of its early functions, the placenta acts as an endocrine gland. c. The placenta is able to keep out most potentially toxic substances, such as cigarette smoke, to which the mother is exposed. d. Optimal blood circulation is achieved through the placenta when the woman is lying on her back or standing.

ANS: B Feedback B The placenta produces four hormones necessary to maintain the pregnancy.

5. The upper uterus is the best place for the fertilized ovum to implant because it is here that the a. Placenta attaches most firmly b. Developing baby is best nourished c. Uterine endometrium is softer d. Maternal blood flow is lower

ANS: B Feedback B The uterine fundus is richly supplied with blood and has the thickest endometrium, both of which promote optimal nourishment of the fetus.

27. A woman is experiencing most of her labor pain in her back. What action by the nurse is best? a. Positioning the woman lying supine with head slightly elevated b. Showing the support person how to apply firm pressure to the sacrum c. Assisting the woman to sit upright with the legs straight d. Massaging her upper back during a contraction

ANS: B Firm pressure against the sacrum may be helpful in relieving the discomfort associated with back labor. The nurse can provide this action, but including the support person (if desired) is beneficial. The woman should not lie on her back. Sitting up with legs straight would put more pressure onto the lower back area. The massage should be in the lower back where the pain is located. PTS: 1 DIF: Cognitive Level: Application/Applying REF: p. 358 OBJ: Nursing Process: Implementation MSC: Client Needs: Physiologic Integrity

An infant presents with lethargy in the newborn nursery on the second day of life. On further examination, vital signs are stable but muscle tone is slightly decreased, with sluggish reflexes noted. Other physical characteristics are noted as being normal. Lab tests reveal a decreased hematocrit and increased blood sugar. The nurse suspects that the infant may be exhibiting signs and symptoms of: a. RDS. b. PIVH. c. BPD. d. ROP

ANS: B IVH or PIVH (intraventricular hemorrhage or periventricular hemorrhage) can be seen during the first week of life. Signs and symptoms are based on the extent of hemorrhage. Typically, one would see lethargy, decreased muscle tone and reflexes, decreased hematocrit, hyperglycemia, acidosis, and seizures. If the newborn had RDS or BPD, there would be more respiratory symptoms exhibited. If the infant had ROP, there would be signs and symptoms related to the eyes. Other physical characteristics are reported as being normal.

17. In order to prevent nipple trauma, the nurse should teach the new mother to a. limit the feeding time to less than 5 minutes. b. position the infant so the nipple is far back in the mouth. c. assess the nipples before each feeding. d. wash the nipples daily with mild soap and water.

ANS: B If the infant's mouth does not cover as much of the areola as possible, the pressure during sucking will be applied to the nipple, causing trauma to the area. Stimulating the breast for less than 5 minutes will not produce the extra milk the infant may need. Assessing the nipples for trauma is important, but it will not prevent sore nipples. Soap can be drying to the nipples and should be avoided during breastfeeding.

16. The registered nurse explains to the student that when giving a narcotic to a laboring woman, the nurse should inject the medication at the beginning of a contraction so that a. full benefit of the medication is received during that contraction. b. less medication will be transferred to the fetus. c. the medication will be rapidly circulated. d. the maternal vital signs will not be adversely affected.

ANS: B Injecting at the beginning of a contraction, when blood flow to the placenta is normally reduced, limits transfer to the fetus. The full benefit will be received by the woman; however, it will decrease the amount reaching the fetus. It will not increase the circulation of the medication. It will not alter the vital signs any more than giving it at another time. PTS: 1 DIF: Cognitive Level: Comprehension/Understanding REF: p. 367 OBJ: Nursing Process: Implementation MSC: Client Needs: Physiologic Integrity

23. What information about iron supplementation should the nurse teach a new mother? a. Start iron supplementation shortly after birth if the infant is breastfeeding exclusively. b. Iron-fortified formula will meet the infant's iron requirements. c. Iron supplements must be given when the infant begins teething. d. Infants need a multivitamin with iron every day.

ANS: B Iron-fortified formula will meet the infant's initial iron requirements. Solid foods added to the diet maintain iron needs as formula intake decreases. Term infants who are exclusively breastfed have adequate iron stored until they are age 6 months. Iron supplements are not necessary for adequate teething. Vitamins and minerals are added to processed formulas and cereals. It should not be necessary for the child to receive a multivitamin with iron unless the infant is at risk for undernutrition.

Following a traumatic birth of a 10-pound infant, the nurse should assess: a. gestational age status. b. flexion of both upper extremities. c. infant's percentile on growth chart. d. blood sugar to detect hyperglycemia.

ANS: B Large infants are at risk for shoulder dystocia, which may result in clavicle fracture or damage to the brachial plexus. Gestational age or the infant's growth chart percentile will not provide data about potential injuries from a traumatic birth. A large infant is at risk for hypoglycemia.

6. The student nurse learns that the hormone necessary for milk production is a. estrogen. b. prolactin. c. progesterone. d. lactogen.

ANS: B Prolactin, secreted by the anterior pituitary, is a hormone that causes the breasts to produce milk. Estrogen decreases the effectiveness of prolactin and prevents mature breast milk from being produced. Progesterone decreases the effectiveness of prolactin and prevents mature breast milk from being produced. Human placental lactogen decreases the effectiveness of prolactin and prevents mature breast milk from being produced.

20. The nurse should explain to new parents that the most serious consequence of propping an infant's bottle is a. dental caries. b. aspiration. c. ear infections. d. colic.

ANS: B Propping the bottle increases the likelihood of choking and aspiration if regurgitation occurs. Dental caries become a problem when milk stays on the gums for a long period of time. This may cause a buildup of bacteria that will alter the growing teeth buds. However, this is not the most serious consequence. Ear infections can occur when the warm formula runs into the ear and bacterial growth occurs. However, this is not the most serious consequence. Colic can occur, but it is not the most serious consequence.

Following the vaginal delivery of a macrosomic infant, the nurse should assess the infant for: a. Hyperglycemia. b. Clavicle fractures. c. Hyperthermia. d. An increase in red blood cells.

ANS: B Rationale: A. LGA infants are prone to hypoglycemia. B. Macrosomic infants may have a complicated delivery and are susceptible to birth injuries such as fractured clavicles, cephalohematomas, and brachial palsy. C. Hyperthermia is usually caused by inappropriate monitoring of infants under radiant heaters. D. An increase in RBCs is not an expected occurrence with LGA infants.

The postpartum woman who continually repeats the story of her labor, delivery, and recovery experiences is a. providing others with her knowledge of events. b. making the birth experience "real." c. taking hold of the events leading to her labor and delivery. d. accepting her response to labor and delivery.

ANS: B Reliving the birth experience makes the event real and helps the mother realize that the pregnancy is over and that the infant is born and is now a separate individual. She is in the taking-in phase, trying to make the birth experience seem real. This process meets her needs, not those of others.

13. The purpose of initiating contractions in a CST is to a. determine the degree of fetal activity. b. apply a stressful stimulus to the fetus. c. identifying fetal acceleration patterns. d. increase placental blood flow.

ANS: B The CST involves recording the response of the FHR to stress induced by uterine contractions. The NST and biophysical profiles look at fetal movements. The NST looks at fetal heart accelerations with fetal movements. The CST records the fetal response to stress. It does not increase placental blood flow. PTS: 1 DIF: Cognitive Level: Knowledge/Remembering REF: p. 282 OBJ: Nursing Process: Assessment MSC: Client Needs: Physiologic Integrity

25. An examiner who discovers unequal movement or uneven gluteal skin folds during the Ortolani maneuver would then: a. Tell the parents that one leg may be longer than the other, but they will equal out by the time the infant is walking. b. Alert the physician that the infant has a dislocated hip. c. Inform the parents and physician that molding has not taken place. d. Suggest that, if the condition does not change, surgery to correct vision problems may be needed.

ANS: B The Ortolani maneuver is a technique for checking hip integrity. Unequal movement suggests that the hip is dislocated. The physician should be notified.

1. A woman gave birth to a healthy 7-pound, 13-ounce infant girl. The nurse suggests that the woman place the infant to her breast within 15 minutes after birth. The nurse knows that breastfeeding is effective during the first 30 minutes after birth because this is the: a. Transition period. c. Organizational stage. b. First period of reactivity. d. Second period of reactivity.

ANS: B The first period of reactivity is the first phase of transition and lasts up to 30 minutes after birth. The infant is highly alert during this phase. The transition period is the phase between intrauterine and extrauterine existence. There is no such phase as the organizational stage. The second period of reactivity occurs roughly between 4 and 8 hours after birth, after a period of prolonged sleep.

16. A nurse is observing a student nurse apply erythromycin ophthalmic ointment. What action by the student requires the nurse to intervene? a. Applies ointment in thin ribbon b. Applies ointment from outer canthus to inner canthus. c. Holds the tube horizontally while applying ointment d. Wipes excess ointment away after 1 minute.

ANS: B The ointment should be applied from inner to outer canthus. When the student does this incorrectly, the nurse should intervene. The other actions are appropriate.

If the fundus is palpated on the right side of the abdomen above the expected level, the nurse should suspect that the patient has a. been lying on her right side too long. b. a distended bladder. c. stretched ligaments that are unable to support the uterus. d. a normal involution.

ANS: B The presence of a full bladder will displace the uterus. This finding does not signify a problem with positioning or ligaments, nor is it an expected finding.

17. A student nurse is preparing an injection of vitamin K (aquaMEPHYTON). What action by the student shows good understanding of this procedure? a. Draws up 1.5 mg of solution b. Protects solution from light c. Finds landmark for subQ injection d. Administers directly after circumcision

ANS: B The solution of vitamin K is light-sensitive, so it should be protected from light. The dose is 0.5 to 1 mg. It is given IM and should be administered prior to a circumcision.

A nurse is examining a woman 2 months after delivery. The woman has lost 25 pounds. What action by the nurse is best? a. Counsel her on other weight loss measures. b. Ask her for a dietary recall for 3 days. c. Instruct her on exercises for faster loss. d. Explain that her weight loss is affecting her breast milk.

ANS: B This woman has lost too much weight for being 8 weeks postpartum. Gradual weight loss is recommended, so the nurse should first assess the woman's eating habits by conducting a nutrition history. From that information the nurse can help the woman plan a safer weight loss plan. She does not need to lose weight faster, so counseling her on weight loss measures or more exercise is not beneficial. Telling her she is harming her baby is not therapeutic and may make her less likely to listen to the nurse.

An infant delivered preterm at 28 weeks' gestation weighs 1200 g. Based on this information, the infant is designated as: a. SGA. b. VLBW. c. ELBW. d. Low birth weight at term.

ANS: B VLBW (very-low-birth-weight) infants weigh 1500 g or less at birth. SGA infants fall below the tenth percentile in growth charts. ELBW (extremely-low-birth-weight) infants weigh 100 g or less at birth. Low birth weight pertains to an infant weighing 2500 g or less at birth. However, this option is incorrect because it specifies at term and the infant in question is designated as preterm at 28 weeks' gestation

33. Cardiovascular changes that cause the foramen ovale to close at birth are a direct result of: a. Increased pressure in the right atrium. b. Increased pressure in the left atrium. c. Decreased blood flow to the left ventricle. d. Changes in the hepatic blood flow.

ANS: B With the increase in the blood flow to the left atrium from the lungs, the pressure is increased, and the foramen ovale is functionally closed. The pressure in the right atrium decreases at birth. It is higher during fetal life. Blood flow increases to the left ventricle after birth. The hepatic blood flow changes, but that is not the reason for the closure of the foramen ovale.

29. A nurse admits a woman to the labor and delivery unit who has a history of IV drug abuse. In planning care for this patient, the nurse explains to the student that which pain control plan is contraindicated for this woman? a. Epidural anesthesia b. Bolus administration of butorphanol (Stadol) c. Promethazine (Phenergan) for opioid-induced nausea d. Naloxone (Narcan) if needed for respiratory depression

ANS: B Women who are opiate-dependent should not receive analgesics having mixed agonist and antagonist actions (butorphanol and nalbuphine). Epidural anesthesia not using these drugs is appropriate as are promethazine and naloxone if needed. PTS: 1 DIF: Cognitive Level: Knowledge/Remembering REF: Table 18.2 OBJ: Nursing Process: Planning MSC: Client Needs: Physiologic Integrity

Which statement made by a lactating woman would lead the nurse to believe that the woman might have lactose intolerance? a. "I always have heartburn after I drink milk." b. "If I drink more than a cup of milk, I usually have abdominal cramps and bloating." c. "Drinking milk usually makes me break out in hives." d. "Sometimes I notice that I have bad breath after I drink a cup of milk."

ANS: B "If I drink more than a cup of milk, I usually have abdominal cramps and bloating." One problem that can interfere with milk consumption is lactose intolerance, which is the inability to digest milk sugar because of a lack of the enzyme lactose in the small intestine. Milk consumption may cause abdominal cramping, bloating, and diarrhea in such people, although many lactose-intolerant individuals can tolerate small amounts of milk without symptoms. A woman with lactose intolerance is more likely to experience bloating and cramping, not heartburn. A client that breaks out in hives after consuming milk is more likely to have a milk allergy. This woman should be advised to simply brush her teeth after consuming dairy products.

1. The causes of preterm labor are not fully understood although many factors have been associated with early labor. These include (select all that apply) a. Singleton pregnancy b. History of cone biopsy c. Smoking d. Short cervical length e. Higher level of education

ANS: B, C, D Feedback Correct A history of cone biopsy, smoking, and short cervical length are maternal risk factors for preterm labor. Others include chronic illness, DES exposure as a fetus, uterine abnormalities, obesity, previous preterm labor or birth, number of embryos implanted, preeclampsia, anemia, or infection. Incorrect Uterine distention caused by multifetal pregnancy or hydramnios are risk factors for preterm labor. Low educational level, low socioeconomic status, little or no prenatal care, poor nutrition, or non-white ethnicity are all demographic risk factors for preterm labor and birth. PTS: 1 DIF: Cognitive Level: Comprehension REF: p. 647 | Table 27-2 OBJ: Nursing Process: Assessment MSC: Client Needs: Health Promotion and Maintenance COMPLETION

1. Some nipple conditions make it necessary to provide intervention before birth in the mother who plans to breastfeed. These include (Select all that apply.) a. Everted nipples b. Flat nipples c. Inverted nipples d. Nipples that contract when compressed e. Cracked nipples

ANS: B, C, D Flat nipples appear soft, like the areola, and do not stand erect unless stimulated by rolling them between the fingers. Inverted nipples are retracted into the breast tissue. These nipples appear normal; however, they will draw inward when the areola is compressed by the infant's mouth. Dome-shaped devices known as breast shells can be worn during the last weeks of pregnancy and between feedings after birth. The shells are placed inside the bra with the opening over the nipple. The shells exert slight pressure against the areola to help the nipples protrude. The helpfulness of breast shells is debated. A breast pump can be used to draw the nipples out before feedings after delivery. Everted nipples protrude and are normal. No intervention will be required. Cracked, blistered, and bleeding nipples occur after breastfeeding has been initiated and are the result of improper latch.

1. Labor is very stressful for the fetus. It is important for the intrapartum nurse to be knowledgeable regarding the mechanisms that regulate the fetal heart rate and keep the brain well oxygenated. When evaluating the patient's progress, the nurse knows that four of the five fetal factors that interact to regulate the heart rate are the a. Uterine activity b. Autonomic nervous system c. Baroreceptors d. Chemoreceptors e. Adrenal glands

ANS: B, C, D, E Feedback Correct The sympathetic and parasympathetic branches of the autonomic nervous system are balanced forces that regulate FHR. Sympathetic stimulation increases the heart rate, while parasympathetic responses, through stimulation of the vagus nerve, reduce the FHR and maintain variability. The baroreceptors stimulate the vagus nerve to slow the FHR and decrease the blood pressure. These are located in the carotid arch and major arteries. The chemoreceptors are cells that respond to changes in oxygen, carbon dioxide, and pH. They are found in the medulla oblongata and the aortic and carotid bodies. The adrenal medulla secretes epinephrine and norepinephrine in response to stress, causing an acceleration in FHR. Incorrect Hypertonic uterine activity can reduce the time available for exchange of oxygen and waste products; however, this is a maternal factor. The fifth fetal factor is the central nervous system. The fetal cerebral cortex causes the heart rate to increase during fetal movement and decrease when the fetus sleeps.

2. Approximately 82% of teen pregnancies are unintended. Seventy percent of teens have had sex by their 19th birthday. Factors that contribute to an increased risk for teen pregnancy include (select all that apply) a. High self-esteem b. Peer pressure c. Limited access to contraception d. Planning sexual activity e. Lack of role models

ANS: B, C, E Feedback Correct Peer pressure to begin sexual activity is a contributing factor towards teenage pregnancy. Limited access to contraceptive devices and lack of accurate information about how to use these devices are also factors. Lack of appropriate role models, desire to alleviate or escape the present situation at home, along with feelings of invincibility, also contribute to teenage pregnancy. Incorrect Low self-esteem and the consequent inability to set limits on sexual activity places the adolescent at risk for teen pregnancy. Ambivalence towards sexuality, and not planning intercourse, are more likely to result in teen pregnancy.

3. According to the NICHD Three-Tier System of Fetal Heart Rate Classification, Category III tracings include all FHR tracings not categorized as Category I or II. Which characteristics of the fetal heart belong in Category III? Select all that apply. a. Baseline rate of 110 to 160 bpm b. Tachycardia c. Absent baseline variability NOT accompanied by recurrent decelerations d. Variable decelerations with other characteristics such as shoulders or overshoots e. Absent baseline variability with recurrent variable decelerations f. Bradycardia

ANS: B, D, E, F Feedback Correct These characteristics are all considered non-reassuring or abnormal and belong in Category III. Incorrect A fetal heart rate of 110 to 160 bpm is considered normal and belongs in Category I. Absent baseline variability not accompanied by recurrent decelerations is a Category II characteristic.

A woman is currently pregnant; she has a 5-year-old son and a 3-year-old daughter. She had one other pregnancy that terminated at 8 weeks. Her gravida and para are a. Gravida 3 para 2 b. Gravida 4 para 3 c. Gravida 4 para 2 d. Gravida 3 para 3

ANS: C Feedback A Because she is currently pregnant, she is classified as a gravida 4; the pregnancy that was terminated at 8 weeks is classified as an abortion. B Gravida 4 is correct, but she is a para 2. The pregnancy that was terminated at 8 weeks is classified as an abortion. C She has had four pregnancies, including the current one (gravida 4). She had two pregnancies that terminated after 20 weeks (para 2). The pregnancy that terminated at 8 weeks is classified as an abortion. D Since she is currently pregnant, she is classified as a gravida 4, not a 3. The para is correct.

As a nurse in labor and delivery, you are caring for a Muslim woman during the active phase of labor. You note that when you touch her, she quickly draws away. You should a. Continue to touch her as much as you need to while providing care. b. Assume that she doesn't like you and decrease your time with. c. Limit touching to a minimum, as this may not be acceptable in her culture. d. Ask the charge nurse to reassign you to another patient.

ANS: C Feedback A By continuing to touch her, the nurse is showing disrespect for her cultural beliefs. B A Muslim's response to touch does not reflect like or dislike. C Touching is an important component of communication in various cultures, but if the patient appears to find it offensive, the nurse should respect her cultural beliefs and limit touching her. D This reaction may be offensive to the patient.

Alterations in hormonal balance and mechanical stretching are responsible for several changes in the integumentary system during pregnancy. Stretch marks often occur on the abdomen and breasts. These are referred to as a. Chloasma b. Linea nigra c. Striae gravidarum d. Angiomas

ANS: C Feedback A Chloasma is a facial melasma also known as the "mask of pregnancy." This condition is manifested by a blotchy, hyperpigmentation of the skin over the cheeks, nose and forehead especially in dark complexioned women. B Linea nigra is a pigmented line extending from the symphysis pubis to the top of the fundus in the midline. C Striae gravidarum or stretch marks appear in 50% to 90% of pregnant women during the second half of pregnancy. They most often occur on the breasts and abdomen. This integumentary alteration is the result of separation within the underlying connective (collagen) tissue. D Angiomas and other changes also may appear.

A woman in her first trimester of pregnancy can expect to visit her physician every 4 weeks so that a. She develops trust in the health care team. b. Her questions about labor can be answered. c. The condition of the expectant mother and fetus can be monitored. d. Problems can be eliminated.

ANS: C Feedback A Developing a trusting relationship should be established during these visits, but that is not the primary reason. B Most women do not have questions concerning labor until the last trimester of the pregnancy. C This routine allows monitoring of maternal health and fetal growth and ensures that problems will be identified early. D All problems cannot be eliminated because of prenatal visits, but they can be identified.

One of the most effective methods for preventing venous stasis is to a. Wear elastic stockings in the afternoons. b. Sleep with the foot of the bed elevated. c. Rest often with the feet elevated. d. Sit with the legs crossed.

ANS: C Feedback A Elastic stockings should be applied before lowering the legs in the morning. B Elevating the legs at night may cause pressure on the diaphragm and increase breathing problems. C Elevating the feet and legs improves venous return and prevents venous stasis. D Sitting with the legs crossed will decrease circulation in the legs and increase venous stasis.

Centering pregnancy is an example of an alternative model of prenatal care. Which statement accurately applies to the centering model of care? a. Group sessions begin with the first prenatal visit. b. At each visit blood pressure, weight, and urine dipsticks are obtained by the nurse. c. Eight to 12 women are placed in gestational-age cohort groups. d. Outcomes are similar to traditional prenatal care.

ANS: C Feedback A Group sessions begin at 12 to 16 weeks of gestation and end with an early postpartum visit. Prior to group sessions, the patient has an individual assessment, physical examination, and history. B At the beginning of each group meeting, patients measure their own BP, weight, and urine dips and enter these in their record. Fetal heart rate assessment and fundal height are obtained by the nurse. C Gestational age cohorts comprise the groups, with approximately 8 to 12 women in each group. This group remains intact throughout the pregnancy. Individual follow-up visits are scheduled as needed. D Results evaluating this approach have been very promising. In a recent study of adolescent patients, there was a decrease in LBW infants and an increase in breastfeeding rates.

Which suggestion is appropriate for the pregnant woman who is experiencing nausea and vomiting? a. Eat only three meals a day so the stomach is empty between meals. b. Drink plenty of fluids with each meal. c. Eat dry crackers or toast before arising in the morning. d. Drink coffee or orange juice immediately on arising in the morning.

ANS: C Feedback A Instruct the woman to eat five to six small meals rather than three full meals per day. Nausea is more intense when the stomach is empty. B Fluids should be taken separately from meals. Fluids overstretch the stomach and may precipitate vomiting. C This will assist with the symptoms of morning sickness. It is also important for the woman to arise slowly. D Coffee and orange juice stimulate acid formation in the stomach. It is best to suggest eating dry carbohydrates when rising in the morning.

What comment by a new mother exhibits understanding of her toddler's response to a new sibling? a. "I can't believe he is sucking his thumb again." b. "He is being difficult, and I don't have time to deal with him." c. "My husband is going to stay with the baby so I can take our son to the park tomorrow." d. "When we brought the baby home, we made our son stop sleeping in the crib."

ANS: C Feedback A It is normal for a child to regress when a new sibling is introduced into the home. B The toddler may have feelings of jealousy and resentment toward the new baby taking the attention from him. Frequent reassurance of parental love and affection are important. C It is important for a mother to seek time alone with her toddler to reassure him that he is loved. D Changes in sleeping arrangements should be made several weeks before the birth so that the child does not feel displaced by the new baby.

To relieve a leg cramp, the patient should be instructed to a. Massage the affected muscle. b. Stretch and point the toe. c. Dorsiflex the foot. d. Apply a warm pack.

ANS: C Feedback A Since she is prone to blood clots in the legs, massaging the affected leg muscle is contraindicated. B Pointing the toes will contract the muscle and not relieve the pain. C Dorsiflexion of the foot stretches the leg muscle and relieves the painful muscle contraction. D Warm packs can be used to relax the muscle, but more immediate relief is necessary, such as dorsiflexion of the foot.

Which patient at term should go to the hospital or birth center the soonest after labor begins? a. Gravida 2 para 1 who lives 10 minutes away b. Gravida 1 para 0 who lives 40 minutes away c. Gravida 3 para 2 whose longest previous labor was 4 hours d. Gravida 2 para 1 whose first labor lasted 16 hours

ANS: C Feedback A A gravida 2 is expected to have a longer labor than the gravida 3. The fact that she lives close to the hospital allows her to stay home for a longer period of time. B A gravida 1 is expected to have the longest labor. C Multiparous women usually have shorter labors than do nulliparous women. The woman described in option c is multiparous with a history of rapid labors, increasing the likelihood that her infant might be born in uncontrolled circumstances. D The gravida 2 is expected to have a longer labor than the gravida 3, especially since her first labor was 16 hours.

5. Which patient situation presents the greatest risk for the occurrence of hypotonic dysfunction during labor? a. A primigravida who is 17 years old b. A 22-year-old multiparous woman with ruptured membranes c. A multiparous woman at 39 weeks of gestation who is expecting twins d. A primigravida woman who has requested no analgesia during her labor

ANS: C Feedback A A young primigravida usually will have good muscle tone in the uterus. This prevents hypotonic dysfunction. B There is no indication that this woman's uterus is overdistended, which is the main cause of hypotonic dysfunction. C Overdistention of the uterus in a multiple pregnancy is associated with hypotonic dysfunction because the stretched uterine muscle contracts poorly. D A primigravida usually will have good uterine muscle tone, and there is no indication of an overdistended uterus. PTS: 1 DIF: Cognitive Level: Analysis REF: p. 636 OBJ: Nursing Process: Assessment MSC: Client Needs: Physiologic Integrity

6. What condition indicates concealed hemorrhage in an abruptio placentae? a. Decrease in abdominal pain b. Bradycardia c. Hard, boardlike abdomen d. Decrease in fundal height

ANS: C Feedback A Abdominal pain may increase. B The patient will have shock symptoms that include tachycardia. C Concealed hemorrhage occurs when the edges of the placenta do not separate. The formation of a hematoma behind the placenta and subsequent infiltration of the blood into the uterine muscle results in a very firm, boardlike abdomen. D The fundal height will increase as bleeding occurs. PTS: 1 DIF: Cognitive Level: Analysis REF: p. 586 OBJ: Nursing Process: Assessment MSC: Client Needs: Physiologic Integrity

Surgical, medical, or mechanical methods may be used for labor induction. Which technique is considered a mechanical method of induction? a. Amniotomy b. Intravenous Pitocin c. Transcervical catheter d. Vaginal insertion of prostaglandins

ANS: C Feedback A Amniotomy is a surgical method of augmentation and induction. B Intravenous Pitocin is a medical method of induction. C Placement of a balloon-tipped Foley catheter into the cervix is a mechanical method of induction. Other methods to expand and gradually dilate the cervix include Laminaria tents, Dilapan and Lamicel. D Insertion of prostaglandins is a medical method of induction.

21. A woman who is 32 weeks pregnant telephones the nurse at her obstetrician's office and complains of constant backache. She asks what pain reliever is safe for her to take. The best nursing response is a. "Back pain is common at this time during pregnancy because you tend to stand with a sway back." b. "Acetaminophen is acceptable during pregnancy; however, you should not take aspirin." c. "You should come into the office and let the doctor check you." d. "Avoid medication because you are pregnant. Try soaking in a warm bath or using a heating pad on low before taking any medication."

ANS: C Feedback A Back pain can also be a symptom of preterm labor and needs to be assessed. B The woman needs to be assessed for preterm labor before providing pain relief. *C A prolonged backache is one of the subtle symptoms of preterm labor. Early intervention may prevent preterm birth.* D The woman needs to be assessed for preterm labor before providing pain relief. PTS: 1 DIF: Cognitive Level: Application REF: p. 647 OBJ: Nursing Process: Implementation MSC: Client Needs: Physiologic Integrity

Which nursing assessment indicates that a woman who is in second-stage labor is almost ready to give birth? a. The fetal head is felt at 0 station during vaginal examination. b. Bloody mucus discharge increases. c. The vulva bulges and encircles the fetal head. d. The membranes rupture during a contraction.

ANS: C Feedback A Birth of the head occurs when the station is +4. A 0 station indicates engagement. B Bloody show occurs throughout the labor process and is not an indication of an imminent birth. C A bulging vulva that encircles the fetal head describes crowning, which occurs shortly before birth. D Rupture of membranes can occur at any time during the labor process and does not indicate an imminent birth.

23. The primary symptom present in abruptio placentae that distinguishes it from placenta previa is a. Vaginal bleeding b. Rupture of membranes c. Presence of abdominal pain d. Changes in maternal vital signs

ANS: C Feedback A Both may have vaginal bleeding. B Rupture of membranes may occur with both conditions. C Pain in abruptio placentae occurs in response to increased pressure behind the placenta and within the uterus. Placenta previa manifests with painless vaginal bleeding. D Maternal vital signs may change with both if bleeding is pronounced. PTS: 1 DIF: Cognitive Level: Knowledge REF: p. 585 OBJ: Nursing Process: Assessment MSC: Client Needs: Physiologic Integrity

Which factor ensures that the smallest anterior-posterior diameter of the fetal head enters the pelvis? a. Descent b. Engagement c. Flexion d. Station

ANS: C Feedback A Descent is the moving of the fetus through the birth canal. B Engagement occurs when the largest diameter of the fetal presenting part has passed the pelvic inlet. C Flexion of the fetal head allows the smallest head diameters pass through the pelvis. D The station is the relationship of the fetal presenting part to the level of the ischial spines.

18. What factor found in maternal history should alert the nurse to the potential for a prolapsed umbilical cord? a. Oligohydramnios b. Pregnancy at 38 weeks of gestation c. Presenting part at station -3 d. Meconium-stained amniotic fluid

ANS: C Feedback A Hydramnios puts the woman at high risk for a prolapsed umbilical cord. B A very small fetus, normally preterm, puts the woman at risk for a prolapsed umbilical cord. C Because the fetal presenting part is positioned high in the pelvis and is not well applied to the cervix, a prolapsed cord could occur if the membranes rupture. D Meconium-stained amniotic fluid shows that the fetus already has been compromised, but it does not increase the chance of a prolapsed cord. PTS: 1 DIF: Cognitive Level: Comprehension REF: p. 659 OBJ: Nursing Process: Assessment MSC: Client Needs: Physiologic Integrity

7. A woman is having her first child. She has been in labor for 15 hours. Two hours ago, her vaginal examination revealed the cervix to be dilated to 5 cm and 100% effaced, and the presenting part was at station 0. Five minutes ago, her vaginal examination indicated that there had been no change. What abnormal labor pattern is associated with this description? a. Prolonged latent phase b. Protracted active phase c. Secondary arrest d. Protracted descent

ANS: C Feedback A In the nulliparous woman, a prolonged latent phase typically lasts more than 20 hours. B A protracted active phase, the first or second stage of labor, would be prolonged (slow dilation). C With a secondary arrest of the active phase, the progress of labor has stopped. This patient has not had any anticipated cervical change, indicating an arrest of labor. D With protracted descent, the fetus would fail to descend at an anticipated rate during the deceleration phase and second stage of labor. PTS: 1 DIF: Cognitive Level: Analysis REF: p. 637 OBJ: Nursing Process: Assessment and Diagnosis MSC: Client Needs: Health Promotion and Maintenance

What results from the adaptation of the fetus to the size and shape of the pelvis? a. Lightening b. Lie c. Molding d. Presentation

ANS: C Feedback A Lightening is the descent of the fetus toward the pelvic inlet before labor. B Lie is the relationship of the long axis of the fetus to the long axis of the mother. C The sutures and fontanels allow the bones of the fetal head to move slightly, changing the shape of the fetal head so it can adapt to the size and shape of the pelvis. D Presentation is the fetal part that first enters the pelvic inlet.

10. What is the only known cure for preeclampsia? a. Magnesium sulfate b. Antihypertensive medications c. Delivery of the fetus d. Administration of acetylsalicylic acid (ASA) every day of the pregnancy

ANS: C Feedback A Magnesium sulfate is one of the medications used to treat but not to cure preeclampsia. B Antihypertensive medications are used to lower the dangerously elevated blood pressures in preeclampsia and eclampsia. C If the fetus is viable and near term, delivery is the only known "cure" for preeclampsia. D Low doses of ASA (60 to 80 mg) have been administered to women at high risk for developing preeclampsia. PTS: 1 DIF: Cognitive Level: Knowledge REF: p. 593 OBJ: Nursing Process: Assessment MSC: Client Needs: Physiologic Integrity

To monitor for potential hemorrhage in the woman who has just had a cesarean birth, the recovery room nurse should a. Maintain an intravenous infusion at 100 mL/hr. b. Assess the abdominal dressings for drainage. c. Assess the uterus for firmness every 15 minutes. d. Monitor her urinary output.

ANS: C Feedback A Maintaining proper fluid balance will not control hemorrhage. B This is an important assessment, but hemorrhage will first be noted vaginally. C Maintaining contraction of the uterus is important in controlling bleeding from the placental site. D This is an important assessment to prevent future hemorrhaging from occurring, but it is not the first priority assessment in the recovery room.

The priority nursing care associated with an oxytocin (Pitocin) infusion is a. Measuring urinary output b. Increasing infusion rate every 30 minutes c. Monitoring uterine response d. Evaluating cervical dilation

ANS: C Feedback A Monitoring urinary output is important with Pitocin, but not the top priority. B The infusion rate may be increased, but only after proper assessment that it is appropriate. C Because of the risk of hyperstimulation, which could result in decreased placental perfusion and uterine rupture, the nurse's priority intervention is monitoring uterine response. D Monitoring labor progression is important, but not the top priority.

21. What finding on a prenatal visit at 10 weeks might suggest a hydatidiform mole? a. Complaint of frequent mild nausea b. Blood pressure of 120/80 mm Hg c. Fundal height measurement of 18 cm d. History of bright red spotting for 1 day, weeks ago

ANS: C Feedback A Nausea increases in a molar pregnancy because of the increased production of hCG. B A woman with a molar pregnancy may have early-onset pregnancy-induced hypertension. C The uterus in a hydatidiform molar pregnancy is often larger than would be expected on the basis of the duration of the pregnancy. D The history of bleeding is normally described as being brownish. PTS: 1 DIF: Cognitive Level: Analysis REF: p. 582 OBJ: Nursing Process: Assessment MSC: Client Needs: Health Promotion and Maintenance

12. In planning for home care of a woman with preterm labor, the nurse needs to address which concern? a. Nursing assessments will be different from those done in the hospital setting. b. Restricted activity and medications will be necessary to prevent recurrence of preterm labor. c. Prolonged bed rest may cause negative physiologic effects. d. Home health care providers will be necessary.

ANS: C Feedback A Nursing assessments will differ somewhat from those performed in the acute care setting, but this is not the concern that needs to be addressed. B Restricted activity and medication may prevent preterm labor; however, not in all women. Additionally, the plan of care is individualized to meet the needs of each patient. C Prolonged bed rest may cause adverse effects such as weight loss, loss of appetite, muscle wasting, weakness, bone demineralization, decreased cardiac output, risk for thrombophlebitis, alteration in bowel functions, sleep disturbance, and prolonged postpartum recovery. D Many women will receive home health nurse visits, but care is individualized for each woman. PTS: 1 DIF: Cognitive Level: Analysis REF: p. 650 OBJ: Nursing Process: Planning MSC: Client Needs: Health Promotion and Maintenance

During labor, a vaginal examination should be performed only when necessary because of the risk of a. Fetal injury b. Discomfort c. Infection d. Perineal trauma

ANS: C Feedback A Properly performed vaginal examinations should not cause fetal injury. B Vaginal examinations may be uncomfortable for some women in labor, but that is not the main reason for limiting them. C Vaginal examinations increase the risk of infection by carrying vaginal microorganisms upward toward the uterus. D A properly performed vaginal examination should not cause perineal trauma.

25. A woman taking magnesium sulfate has respiratory rate of 10 breaths/min. In addition to discontinuing the medication, the nurse should a. Vigorously stimulate the woman. b. Instruct her to take deep breaths. c. Administer calcium gluconate. d. Increase her IV fluids.

ANS: C Feedback A Stimulation will not increase the respirations. B This will not be successful in reversing the effects of the magnesium sulfate. C Calcium gluconate reverses the effects of magnesium sulfate. D Increasing her IV fluids will not reverse the effects of the medication. PTS: 1 DIF: Cognitive Level: Application REF: p. 595 OBJ: Nursing Process: Implementation MSC: Client Needs: Physiologic Integrity

9. After a birth complicated by a shoulder dystocia, the infant's Apgar scores were 7 at 1 minute and 9 at 5 minutes. The infant is now crying vigorously. The nurse in the birthing room should a. Give supplemental oxygen with a small facemask. b. Encourage the parents to hold the infant. c. Palpate the infant's clavicles. d. Perform a complete newborn assessment.

ANS: C Feedback A The Apgar indicates that no respiratory interventions are needed. B The infant needs to be assessed for clavicle fractures before excessive movement. C Because of the shoulder dystocia, the infant's clavicles may have been fractured. Palpation is a simple assessment to identify crepitus or deformity that requires follow-up. D A complete newborn assessment is necessary for all newborns, but assessment of the clavicle is top priority for this infant. PTS: 1 DIF: Cognitive Level: Comprehension REF: p. 638 OBJ: Nursing Process: Assessment MSC: Client Needs: Physiologic Integrity

31. At 1 minute after birth, the nurse assesses the newborn to assign an Apgar score. The apical heart rate is 110 bpm, and the infant is crying vigorously with the limbs flexed. The infant's trunk is pink, but the hands and feet are blue. What is the Apgar score for this infant? a. 7 b. 8 c. 9 d. 10

ANS: C Feedback A The baby received 2 points for each of the categories except color. Since the infant's hands and feet were blue this category is given a grade of 1. B The baby received 2 points for each of the categories except color. Since the infant's hands and feet were blue this category is given a grade of 1. C The Apgar score is 9 because 1 point is deducted from the total score of 10 for the infant's blue hands and feet. D The infant had 1 point deducted because of the blue color of the hands and feet.

To provide safe care for the woman, the nurse understands that which condition is a contraindication for an amniotomy? a. Dilation less than 3 cm b. Cephalic presentation c. -2 station d. Right occiput posterior position

ANS: C Feedback A The dilation must be enough to determine labor. B The presenting part should be cephalic. Amniotomy is deferred if the presenting part is higher in the pelvis. C A prolapsed cord can occur if the membranes artificially rupture when the presenting part is not engaged. D This indicates a cephalic presentation, which is appropriate for an amniotomy.

The maternity nurse understands that as the uterus contracts during labor, maternal-fetal exchange of oxygen and waste products a. Continues except when placental functions are reduced b. Increases as blood pressure decreases c. Diminishes as the spiral arteries are compressed d. Is not significantly affected

ANS: C Feedback A The maternal blood supply to the placenta gradually stops with contractions. B The exchange of oxygen and waste products decreases. C During labor contractions, the maternal blood supply to the placenta gradually stops as the spiral arteries supplying the intervillous space are compressed by the contracting uterine muscle. D The exchange of oxygen and waste products is affected by contractions.

The greatest risk to the newborn after an elective cesarean birth is a. Trauma due to manipulation during delivery b. Tachypnea due to maternal anesthesia c. Prematurity due to miscalculation of gestation d. Tachycardia due to maternal narcotics

ANS: C Feedback A There is reduced trauma with a cesarean birth. B Maternal anesthesia may cause respiratory distress. C Regardless of the many criteria used to determine gestational age, inadvertent preterm birth still occurs. D Maternal narcotics may cause respiratory distress.

6. A primigravida at 40 weeks of gestation is having uterine contractions every 1.5 to 2 minutes and says that they are very painful. Her cervix is dilated 2 cm and has not changed in 3 hours. The woman is crying and wants an epidural. What is the likely status of this woman's labor? a. She is exhibiting hypotonic uterine dysfunction. b. She is experiencing a normal latent stage. c. She is exhibiting hypertonic uterine dysfunction. d. She is experiencing pelvic dystocia.

ANS: C Feedback A With hypotonic uterine dysfunction, the woman initially makes normal progress into the active stage of labor and then the contractions become weak and inefficient or stop altogether. B The contraction pattern seen in this woman signifies hypertonic uterine activity. C Women who experience hypertonic uterine dysfunction, or primary dysfunctional labor, often are anxious first-time mothers who are having painful and frequent contractions that are ineffective at causing cervical dilation or effacement to progress. D Pelvic dystocia can occur whenever contractures of the pelvic diameters reduce the capacity of the bony pelvis, including the inlet, midpelvis, outlet, or any combination of these planes. PTS: 1 DIF: Cognitive Level: Application REF: pp. 637-638 OBJ: Nursing Process: Diagnosis MSC: Client Needs: Health Promotion and Maintenance

4. How does the available staff influence the selection of either continuous electronic or intermittent auscultation as the fetal-monitoring method? a. There must be a 1:1 nurse-to-patient ratio regardless of the method used. b. Staffing patterns do not influence fetal monitoring choices. c. Use of intermittent auscultation requires a lower nurse-to-patient ratio. d. More nurses are needed when electronic fetal monitoring is used because of increased medical interventions.

ANS: C Feedback A A one-to-one ratio is needed during the second stage of labor or if a high-risk condition exists, regardless of the monitoring method used. B Staffing patterns do plan a role in maintaining safe monitoring practice of the labor patient. C Intermittent auscultation is more staff-intensive. D Less nursing time is needed with electronic monitoring, giving the nurse more time for teaching and supporting the laboring woman.

3. Which method of assessing the fetal heart rate requires the use of a gel? a. Fetoscope b. Tocodynamometer c. Doppler d. Scalp electrode

ANS: C Feedback A The fetoscope does not require the use of gel because ultrasonic transmission is not used. B Tocodynamometer does not require the use of gel. This device monitors uterine contractions. C Doppler is the only listed method involving ultrasonic transmission of fetal heart rates; it requires use of a gel. D The scalp electrode is attached to the fetal scalp; gel is not necessary.

17. Which nursing action is correct when initiating electronic fetal monitoring? a. Lubricate the tocotransducer with an ultrasound gel. b. Inform the patient that she should remain in the semi-Fowler's position. c. Securely apply the tocotransducer with a strap or belt. d. Determine the position of the fetus before attaching the electrode to the maternal abdomen.

ANS: C Feedback A The tocotransducer does not need gel to operate appropriately. B The patient should be encouraged to move around during labor. C The tocotransducer should fit snugly on the abdomen to monitor uterine activity accurately. D The tocotransducer should be placed at the fundal area of the uterus.

9. The nurse-midwife is concerned that a woman's uterine activity is too intense and that her obesity is preventing accurate assessment of the actual intrauterine pressure. On the basis of this information, the nurse should obtain a(n) a. Tocotransducer b. Scalp electrode c. Intrauterine pressure catheter d. Doppler transducer

ANS: C Feedback A The tocotransducer measures the uterine pressure externally; this not be accurate with obesity. B A scalp electrode measurers the fetal heart rate (FHR). C An intrauterine pressure catheter can measure actual intrauterine pressure. D A Doppler auscultates the FHR.

15. The nurse notes a pattern of late decelerations on the fetal monitor. The most appropriate action is to a. Continue observation of this reassuring pattern. b. Notify the physician or nurse-midwife. c. Give the woman oxygen by face mask. d. Place the woman in a Trendelenburg position.

ANS: C Feedback A This is not a reassuring pattern; interventions are needed. B Nursing interventions should be initiated before notifying the health care provider. C Late decelerations are associated with reduced placental perfusion. Giving the laboring woman oxygen increases the oxygen saturation in her blood, making more oxygen available to the fetus. D The Trendelenburg position will not increase the placental perfusion.

15. A primigravida is being monitored in her prenatal clinic for preeclampsia. What finding should concern her nurse? a. Blood pressure increase to 138/86 mm Hg b. Weight gain of 0.5 kg during the past 2 weeks c. A dipstick value of 3+ for protein in her urine d. Pitting pedal edema at the end of the day

ANS: C Feedback A Generally, hypertension is defined as a BP of 140/90 or an increase in systolic pressure of 30 mm Hg or 15 mm Hg diastolic pressure. B Preeclampsia may be manifested as a rapid weight gain of more than 2 kg in 1 week. C Proteinuria is defined as a concentration of 1+ or greater via dipstick measurement. A dipstick value of 3+ should alert the nurse that additional testing or assessment should be made. D Edema occurs in many normal pregnancies as well as in women with preeclampsia. Therefore, the presence of edema is no longer considered diagnostic of preeclampsia. PTS: 1 DIF: Cognitive Level: Analysis REF: p. 592 OBJ: Nursing Process: Diagnosis MSC: Client Needs: Physiologic Integrity

Pregnant adolescents are at high risk for _____ because of lower body mass indices (BMIs) and "fad" dieting. a. Obesity b. Gestational diabetes c. Low-birth-weight babies d. High-birth-weight babies

ANS: C Adolescents tend to have lower BMIs. Additionally, the fetus and still growing mother appear to compete for nutrients. These factors, along with inadequate weight gain, lend themselves to a higher incidence of low-birth-weight babies. Obesity is associated with a higher than normal BMI. Unless the teen has type 1 diabetes, an adolescent with a low BMI is less likely to develop gestational diabetes. This is often associated with obesity. High-birth-weight or large for gestational age (LGA) babies are most often associated with gestational diabetes.

12. With regard to weight gain during pregnancy, maternity nurses should know that: a. In this case, the woman's height is not a factor in determining her target weight b. Obese women may have their health concerns, but their risk of giving birth to a child with major congenital defects is the same as with normal-weight women c. Women with inadequate weight gain have an increased risk of delivering an infant with intrauterine growth restriction (IUGR) d. Greater than expected weight gain during pregnancy is almost always due to old-fashioned overeating

ANS: C IUGR is associated with women with inadequate weight gain. The primary factor in making a weight gain recommendation is the appropriateness of the prepregnancy weight for the woman's height. Obese women are twice as likely as normal-weight women to give birth to a child with major congenital defects. Overeating is only one of several likely causes.

While taking a diet history, the nurse might be told that the expectant mother has cravings for ice chips, cornstarch, and baking soda. This represents a nutritional problem known as: a. Preeclampsia b. Pyrosis c. Pica d. Purging

ANS: C The consumption of foods low in nutritional value or of nonfood substances (e.g., dirt, laundry starch) is called pica. Preeclampsia is a vasospastic disease process encountered after 20 weeks of gestation. Characteristics include increasing hypertension, proteinuria, and hemoconcentration. A burning sensation in the epigastric region is pyrosis, otherwise known as heartburn. Purging refers to self-induced vomiting after consuming large quantities of food.

A 27-year-old pregnant woman had a preconceptual body mass index (BMI) of 18.0. The nurse knows that this woman's total recommended weight gain during pregnancy should be at least: a. 20 kg (44 lb) b. 16 kg (35 lb) c. 12.5 kg (27.5 lb) d. 10 kg (22 lb)

ANS: C This woman has a normal BMI and should gain 11.5 to 16 kg during pregnancy. A weight gain of 20 kg (44 lb) is unhealthy for most women. This woman has a normal BMI and should gain 11.5 to 16 kg during pregnancy. A weight gain of 16 kg (35 lb) is the high end of the range of weight this woman should gain in her pregnancy. A weight gain of 10 kg (22 lb) is appropriate for an obese woman. This woman has a normal BMI, which indicates that her weight is average.

3. What is a major barrier to health care for teen mothers? a. The hospital/clinic is within walking distance of the girl's home. b. The institution is open days, evenings, and Saturday by special arrangement. c. The teen must be prepared to see a different nurse or doctor or both at every visit. d. The health care workers have a positive attitude.

ANS: C Feedback A If the hospital/clinic were within walking distance of the girl's home, it would prevent the teen from missing appointments because of transportation problems. B If the institution were open days, evenings, and Saturday by special arrangement, this availability would be helpful for teens who work, go to school, or have other time-of-day restrictions. Scheduling conflicts are a major barrier to health care. C Whenever possible, the teen should be scheduled to see the same nurses and practitioners for continuity of care. D A negative attitude is unfortunate, because it discourages families that would benefit most from consistent prenatal care.

11. When helping the mother, father, and other family members actualize the loss of the infant, nurses should a. Use the words lost or gone rather than dead or died. b. Make sure the family understands that it is important to name the baby. c. If the parents choose to visit with the baby, apply lotion to the baby and wrap the infant in a pretty blanket. d. Set a firm time for ending the visit with the baby so that the parents know when to let go.

ANS: C Feedback A Nurses must use dead and died to assist the bereaved in accepting reality. B Although naming the baby can be helpful, it is important not to create the sense that parents have to name the baby. In fact, some cultural taboos and religious rules prohibit the naming of an infant who has died. C Presenting the baby in a nice way stimulates the parents' senses and provides pleasant memories of their baby. D Parents need different time periods with their baby to say goodbye. Nurses need to be careful not to rush the process.

6. A woman who is older than 35 years may have difficulty achieving pregnancy, because a. Personal risk behaviors influence fertility. b. She has used contraceptives for an extended time. c. Her ovaries may be affected by the aging process. d. Prepregnancy medical attention is lacking.

ANS: C Feedback A The older adult participates in fewer risk behaviors than the younger adult. B The problem is the age of the ovaries, not the past use of contraceptives. C Once the mature woman decides to conceive, a delay in becoming pregnant may occur because of the normal aging of the ovaries. D Prepregnancy medical care is available and encouraged.

9. A patient at 24 weeks of gestation says she has a glass of wine with dinner every evening. The nurse will counsel her to eliminate all alcohol intake, because a. A daily consumption of alcohol indicates a risk for alcoholism. b. She will be at risk for abusing other substances as well. c. The fetus is placed at risk for altered brain growth. d. The fetus is at risk for multiple organ anomalies.

ANS: C Feedback A This is not the major risk for the infant. B This has not been proven. C The brain grows most rapidly in the third trimester and is most vulnerable to alcohol exposure during this time. D The major concerns are mental retardation, learning disabilities, high activity level, and short attention span.

4. Of adolescents who become pregnant, what percentage have had a previous birth? a. 10% b. 15% c. 19% d. 35%

ANS: C Feedback A This percentage rate is too low; it is actually 19%. B The percentage of teens who have had a previous birth is 19%. C 19% of pregnant adolescents have had one or more previous births. D This percentage rate is too high when measuring previous adolescent births.

Which statement is most true about large for gestational age (LGA) infants? a. They weigh more than 3500 g. b. They are above the 80th percentile on gestational growth charts. c. They are prone to hypoglycemia, polycythemia, and birth injuries. d. Postmaturity syndrome and fractured clavicles are the most common complications.

ANS: C All three of these complications are common in LGA infants.

21. With regard to the gastrointestinal (GI) system of the newborn, nurses should be aware that: a. The newborn's cheeks are full because of normal fluid retention. b. The nipple of the bottle or breast must be placed well inside the baby's mouth because teeth have been developing in utero, and one or more may even be through. c. Regurgitation during the first day or two can be reduced by burping the infant and slightly elevating the baby's head. d. Bacteria are already present in the infant's GI tract at birth because they traveled through the placenta.

ANS: C Avoiding overfeeding can also reduce regurgitation. The newborn's cheeks are full because of well-developed sucking pads. Teeth do develop in utero, but the nipple is placed deep because the baby cannot move food from the lips to the pharynx. Bacteria are not present at birth, but they soon enter through various orifices.

15. As the nurse assists a new mother with breastfeeding, she asks, "If formula is prepared to meet the nutritional needs of the newborn, what is in breast milk that makes it better?" The nurse's best response is that it contains a. more calories. b. essential amino acids. c. important immunoglobulins. d. more calcium.

ANS: C Breast milk contains immunoglobulins that protect the newborn against infection. The calorie count of formula and breast milk is about the same. All of the essential amino acids are in both formula and breast milk. The concentrations may differ. Calcium levels are higher in formula than breast milk. This higher level can cause an excessively high renal solute load if the formula is not diluted properly.

A nurse has taught a woman and partner about measures to improve sexuality after childbirth. Which statement by the partner demonstrates a need for further teaching? a. "We will use water-soluble lubricant before intercourse." b. "We can try having sex in the morning when we are rested." c. "Breastfeeding before sex will increase vaginal lubrication." d. "My wife will be more comfortable if she is on top."

ANS: C Breastfeeding just prior to intercourse may allow uninterrupted time while the baby sleeps afterward, although it will not increase vaginal lubrication. It also decreases the chance of leaking milk. The other statements show good understanding.

6. While working with the pregnant woman in her first trimester, the nurse is aware that chorionic villus sampling (CVS) can be performed during pregnancy as early as _____ weeks. a. 4 b. 8 c. 10 d. 12

ANS: C CVS is usually performed between 10 and 13 weeks of gestation to diagnose fetal chromosomal, metabolic, or DNA abnormalities. PTS: 1 DIF: Cognitive Level: Knowledge/Remembering REF: p. 278 OBJ: Nursing Process: Assessment MSC: Client Needs: Physiologic Integrity

What is the best way for the nurse to promote and support the maternal-infant bonding process? a. Help the mother identify her positive feelings toward the newborn. b. Encourage the mother to provide all newborn care. c. Assist the family with rooming-in. d. Return the newborn to the nursery during sleep periods.

ANS: C Close and frequent interaction between mother and infant, which is facilitated by rooming-in, is important in the bonding process. This is often referred to as the mother-baby care or couplet care. Having the mother express her feelings is important, but it is not the best way to promote bonding. The mother needs time to rest and recuperate; she should not be expected to do all of the care. The mother needs to observe the infant during all stages so she will be aware of what to expect when they go home

1. A yellow crust has formed over the circumcision site. The mother calls the hotline at the local hospital, 5 days after her son was circumcised. She is very concerned. On which rationale should the nurse base her reply? a. After circumcision, the diaper should be changed frequently and fastened snugly. b. This yellow crust is an early sign of infection. c. The yellow crust should not be removed. d. Discontinue the use of petroleum jelly to the tip of the penis.

ANS: C Crust is a normal part of healing and should not be removed. The diaper should be fastened loosely to prevent rubbing or pressure on the incision site. The normal yellowish exudate that forms over the site should be differentiated from the purulent drainage of infection. The only contraindication for petroleum jelly is the use of a PlastiBell.

A postpartum woman overhears the nurse tell the obstetrics clinician that she has a positive Homans sign and asks what it means. The nurse's best response is a. "You have pitting edema in your ankles." b. "You have deep tendon reflexes rated 2+." c. "You have calf pain when I flexed your foot." d. "You have a 'fleshy' odor to your vaginal drainage."

ANS: C Discomfort in the calf with sharp dorsiflexion of the foot may indicate a deep vein thrombosis. It does not indicate edema, rate deep tendon reflexes, or describe the odor of lochia.

A nurse observes a mother on her first postpartum day sitting in bed while her newborn lies awake in the bassinet. What action by the nurse is best? a. Realize that this situation is perfectly acceptable. b. Offer to hand the baby to the woman. c. Hand the baby to the woman. d. Explain "taking in" to the woman.

ANS: C During the "taking-in" phase of maternal adaptation, in which the mother may be passive and dependent, the nurse should encourage bonding when the infant is in the quiet alert stage. This is done best by simply giving the baby to the mother. While acceptable, the nurse can still facilitate infant bonding. The woman is dependent and passive at this stage and may have difficulty making a decision so offering her the baby is not the best option. Women learn best in the taking-hold phase.

11. What is the purpose of amniocentesis for the patient hospitalized at 34 weeks with pregnancy-induced hypertension? a. Identification of abnormal fetal cells b. Detection of metabolic disorders c. Determination of fetal lung maturity d. Identification of sex of the fetus

ANS: C During the third trimester, amniocentesis is most often performed to determine fetal lung maturity. In pregnancy-induced hypertension, preterm delivery may be necessary because of changes in placental perfusion. It is not done to identify abnormal fetal cells, detect metabolic disorders, or identify the sex of the fetus. PTS: 1 DIF: Cognitive Level: Knowledge/Remembering REF: p. 279 OBJ: Nursing Process: Assessment MSC: Client Needs: Physiologic Integrity

11. A laboring woman has been given an injection of epidural anesthesia. Which assessment by the nurse takes priority? a. Urinary output b. Contraction pattern c. Maternal blood pressure d. Intravenous infusion rate

ANS: C Epidural anesthesia may produce maternal hypotension due to vasodilation so the priority assessment by the nurse is maternal blood pressure. The other assessments are important for this woman but are not directly related to the anesthetic injection. PTS: 1 DIF: Cognitive Level: Application/Applying REF: p. 372 | Table 18.2 OBJ: Nursing Process: Implementation MSC: Client Needs: Physiologic Integrity

11. While assessing the integument of a 24-hour-old newborn, the nurse notes a pink, papular rash with vesicles superimposed on the thorax, back, and abdomen. The nurse should: a. Notify the physician immediately. b. Move the newborn to an isolation nursery. c. Document the finding as erythema toxicum. d. Take the newborn's temperature and obtain a culture of one of the vesicles.

ANS: C Erythema toxicum (or erythema neonatorum) is a newborn rash that resembles flea bites. This is a normal finding that does not require notification of the physician, isolation of the newborn, or any additional interventions.

8. During a centering pregnancy group meeting, the nurse teaches patients that the fetal period is best described as one of a. Development of basic organ systems 4/14 b. Resistance of organs to damage from external agents c. Maturation of organ systems d. Development of placental oxygen-carbon dioxide exchange

ANS: C Feedback C During the fetal period, the body systems grow in size and mature in function to allow independent existence after birth.

3. The purpose of the ovum's zona pellucida is to a. Make a pathway for more than one sperm to reach the ovum. b. Allow the 46 chromosomes from each gamete to merge. c. Prevent multiple sperm from fertilizing the ovum. d. Stimulate the ovum to begin mitotic cell division.

ANS: C Feedback C Fertilization causes the zona pellucida to change its chemical composition so that multiple sperm cannot fertilize the ovum.

17. Sally comes in for her first prenatal examination. This is her first child. She asks you (the nurse), "How does my baby get air inside my uterus?" The correct response is a. "The baby's lungs work in utero to exchange oxygen and carbon dioxide." b. "The baby absorbs oxygen from your blood system." c. "The placenta provides oxygen to the baby and excretes carbon dioxide into your bloodstream." d. "The placenta delivers oxygen-rich blood through the umbilical artery to the baby's abdomen."

ANS: C Feedback C The placenta functions by supplying oxygen and excreting carbon dioxide to the maternal bloodstream.

13. Which woman will most likely have increased anxiety and tension during her labor? a. Gravida 1 who did not attend prepared childbirth classes b. Gravida 2 who refused any medication c. Gravida 2 who delivered a stillborn baby last year d. Gravida 3 who has two children younger than 3 years

ANS: C If a previous pregnancy had a poor outcome, the woman will probably be more anxious during labor and delivery. The woman is not prepared for labor and will have increased anxiety during labor. However, the woman with a poor previous outcome is more likely to experience more anxiety, and good teaching by the nurse will diminish some of the anxiety. A gravida 2 has previous experience and can anticipate what to expect. By refusing any medication, she is taking control over her situation and will have less anxiety. This gravida 3 has previous experience and is aware of what to expect. PTS: 1 DIF: Cognitive Level: Comprehension/Understanding REF: p. 356 OBJ: Nursing Process: Assessment MSC: Client Needs: Psychosocial Integrity

10. With regard to lab tests and diagnostic tests in the hospital after birth, nurses should be aware that a. all states test for phenylketonuria (PKU), hypothyroidism, cystic fibrosis, and sickle cell diseases. b. federal law prohibits newborn genetic testing without parental consent. c. if genetic screening is done before the infant is 24 hours old, it should be repeated at age 1 to 2 weeks. d. hearing screening is now mandated by federal law.

ANS: C If testing is done prior to 24 hours of age, genetic screening should be repeated when the infant is 1 to 2 weeks old. States all test for PKU and hypothyroidism, but other genetic defects are not universally covered. Federal law mandates newborn genetic screening; however, parents can decline testing. A waiver should be signed and a notation made in the infant's medical record. Federal law does not mandate screening for hearing problems; however, the majority of states have enacted legislation mandating newborn hearing screening. In the United States the majority (95%) of infants is screened for hearing loss prior to discharge from the hospital.

15. The laboring woman who imagines her body opening to let the baby out is using a mental technique called a. dissociation. b. effleurage. c. imagery. d. distraction.

ANS: C Imagery is a technique of visualizing images that will assist the woman in coping with labor. Dissociation helps the woman learn to relax all muscles except those that are working. Effleurage is self-massage. Distraction can be used in the early latent phase by having the woman involved in another activity. PTS: 1 DIF: Cognitive Level: Knowledge/Remembering REF: p. 359 OBJ: Nursing Process: Assessment MSC: Client Needs: Psychosocial Integrity

What will the nurse note when assessing an infant with asymmetric intrauterine growth restriction? a. One side of the body appears slightly smaller than the other. b. All body parts appear proportionate. c. The head seems large compared with the rest of the body. d. The extremities are disproportionate to the trunk.

ANS: C In asymmetric intrauterine growth retardation, the head is normal in size but appears large because the infant's body is long and thin due to lack of subcutaneous fat.

8. The nurse administers vitamin K to the newborn for what reason? a. Most mothers have a diet deficient in vitamin K, which results in the infant's being deficient. b. Vitamin K prevents the synthesis of prothrombin in the liver and must be given by injection. c. Bacteria that synthesize vitamin K are not present in the newborn's intestinal tract. d. The supply of vitamin K is inadequate for at least 3 to 4 months, and the newborn must be supplemented.

ANS: C In order to promote clotting, vitamin K is necessary. However, the bacteria that synthesize vitamin K are not present in the newborn's intestinal tract, so the nurse administers it via injection. The maternal diet has no bearing on the amount of vitamin K found in the newborn. It is not involved in the synthesis of prothrombin. By day 8, normal newborns are able to produce their own vitamin K.

Postpartal overdistention of the bladder and urinary retention can lead to which complication? a. Postpartum hemorrhage and eclampsia b. Fever and increased blood pressure c. Postpartum hemorrhage and urinary tract infection d. Urinary tract infection and uterine rupture

ANS: C Incomplete emptying and overdistention of the bladder can lead to urinary tract infection. Overdistention of the bladder displaces the uterus and prevents contraction of the uterine muscle. There is no correlation between bladder distention and eclampsia, blood pressure, or fever. The risk of uterine rupture decreases after the birth.

Which nursing diagnosis would be considered a priority for a newborn infant who is receiving phototherapy in an isolette? a. Hypothermia because of phototherapy treatment b. Impaired skin integrity related to diarrhea as a result of phototherapy c. Fluid volume deficit related to phototherapy treatment d. Knowledge deficit (parents) related to initiation of medical therapy

ANS: C Infants who undergo phototherapy as a result of the medical diagnosis of hyperbilirubinemia are at risk for hyperthermia, not hypothermia. Although impaired skin integrity can occur, the priority nursing diagnosis focuses on the physiologic effects of fluid volume deficit. The infant is losing fluid via insensible losses, increased output (in the form of diarrhea), and limited intake. Lack of knowledge is a pertinent nursing diagnosis for parents but physiologic needs take precedence.

Which data would alert the nurse caring for an SGA infant that additional calories may be needed? a. The latest hematocrit was 53%. b. The infant's weight gain is 40 g/day. c. Three successive temperature measurements were 97°, 96°, and 97° F. d. The infant is taking 120 ml/kg every 24 hours.

ANS: C Low body temperature indicates that additional calories are needed to maintain body temperature.

If the patient's white blood cell (WBC) count is 25,000/mm3 on her second postpartum day, the nurse should a. tell the physician immediately. b. have the laboratory draw blood for reanalysis. c. recognize that this is an acceptable range at this point. d. begin antibiotic therapy immediately.

ANS: C Marked leukocytosis occurs with WBC counts increasing to as high as 30,000/mm3 during labor and the immediate postpartum period. The WBC falls to normal within 6 days postpartum. No action is necessary.

37. A meconium stool can be differentiated from a transitional stool in the newborn because the meconium stool is: a. Seen at age 3 days. b. The residue of a milk curd. c. Passed in the first 12 hours of life. d. Lighter in color and looser in consistency.

ANS: C Meconium stool is usually passed in the first 12 hours of life, and 99% of newborns have their first stool within 48 hours. If meconium is not passed by 48 hours, obstruction is suspected. Meconium stool is the first stool of the newborn and is made up of matter remaining in the intestines during intrauterine life. Meconium is dark and sticky.

A pregnant patient asks when the dark line on her abdomen (linea nigra) will go away. The nurse knows the pigmentation will decrease after delivery because of a. increased estrogen. b. increased progesterone. c. decreased melanocyte-stimulating hormone. d. decreased human placental lactogen.

ANS: C Melanocyte-stimulating hormone increases during pregnancy and is responsible for changes in skin pigmentation; the amount decreases after delivery. The linea nigra will eventually fade away for most women. Estrogen and progesterone levels decrease after delivery. Human placental lactogen production continues to aid in lactation. However, it does not affect pigmentation.

24. A new mother wants to be sure that she is meeting her daughter's needs while feeding her commercially prepared infant formula. The nurse determines that the mother meets her child's needs when she a. adds rice cereal to her formula at 2 weeks of age to ensure adequate nutrition. b. warms the bottles using a microwave oven. c. burps her infant during and after the feeding as needed. d. refrigerates any leftover formula for the next feeding.

ANS: C Most infants swallow air when fed from a bottle and should be given a chance to burp several times during a feeding and after the feeding. Solid food should not be introduced to the infant for at least 4 to 6 months after birth. A microwave should never be used to warm any food to be given to an infant. The heat is not distributed evenly, which may pose a risk of burning the infant. Any formula left in the bottle after the feeding should be discarded, because the infant's saliva has mixed with it.

17. A woman is scheduled for an ultrasound and is asking the nurse questions about this test. Which statement by the nurse regarding ultrasonography during pregnancy is most accurate? a. Ultrasonography uses infrared technology to create an image. b. Ultrasonography is only utilized as an adjunct to more invasive tests. c. Ultrasonography is not harmful to the fetus. d. Ultrasonography is not a component of biophysical profile testing.

ANS: C Most women look forward to the results of this test, which causes no harm to the fetus. Ultrasonography uses sound waves to create an image. As an adjunct to more invasive tests, ultrasonography can provide visual guidance for increased safety. It can be done as a standalone test. Ultrasonography is a component of biophysical profile testing. PTS: 1 DIF: Cognitive Level: Knowledge/Remembering REF: p. 274 OBJ: Integrated Process: Teaching-Learning MSC: Client Needs: Health Promotion and Maintenance

Overstimulation may cause increased oxygen use in a preterm infant. Which nursing intervention helps to avoid this problem? a. Group all care activities together to provide long periods of rest. b. While giving your report to the next nurse, stand in front of the incubator and talk softly about how the infant responds to stimulation. c. Teach the parents signs of overstimulation, such as turning the face away or stiffening and extending the extremities and fingers. d. Keep charts on top of the incubator so the nurses can write on them there.

ANS: C Parents should be taught these signs of overstimulation so they will learn to adapt their care to the needs of their infant.

A nurse is observing a 38-week gestation newborn in the nursery. Data reveals periods of apnea lasting approximately 10 seconds followed by a period of rapid respirations. The infant's color and heart rate remain unchanged. The nurse suspects that the infant: a. Is exhibiting signs of RDS. b. Requires tactile stimulation around the clock to ensure that apneic periods do not progress further. c. Is experiencing periodic breathing episodes and will require continuous monitoring while in the nursery unit. d. Requires the use of CPAP to promote airway expansion.

ANS: C Periodic breathing can occur in term or preterm infants; it consists of periods of breathing cessation (5 to 10 seconds) followed by a period of increased respirations (10 to 15 breaths/min). It is not associated with any color or heart rate changes. Infants who exhibit this pattern should continue to be observed. There is no clinical evidence that the infant is exhibiting signs of respiratory distress syndrome (RDS). There is no indication that a pattern of tactile stimulation should be initiated. Continuous positive airway pressure (CPAP) and tactile stimulation would be indicated if the infant were to have apneic spells.

24. What marks on a baby's skin may indicate an underlying problem that requires notification of a physician? a. Mongolian spots on the back b. Telangiectatic nevi on the nose or nape of the neck c. Petechiae scattered over the infant's body d. Erythema toxicum anywhere on the body

ANS: C Petechiae (bruises) scattered over the infant's body should be reported to the pediatrician because they may indicate underlying problems. Mongolian spots are bluish-black spots that resemble bruises but fade gradually over months and have no clinical significance. Telangiectatic nevi (stork bites, angel kisses) fade by the second year and have no clinical significance. Erythema toxicum is an appalling-looking rash, but it has no clinical significance and requires no treatment.

32. A first-time dad is concerned that his 3-day-old daughter's skin looks "yellow." In the nurse's explanation of physiologic jaundice, what fact should be included? a. Physiologic jaundice occurs during the first 24 hours of life. b. Physiologic jaundice is caused by blood incompatibilities between the mother and infant blood types. c. The bilirubin levels of physiologic jaundice peak between the second and fourth days of life. d. This condition is also known as "breast milk jaundice."

ANS: C Physiologic jaundice becomes visible when the serum bilirubin reaches a level of 5 mg/dL or greater, which occurs when the baby is approximately 3 days old. This finding is within normal limits for the newborn. Pathologic jaundice occurs during the first 24 hours of life. Pathologic jaundice is caused by blood incompatibilities, causing excessive destruction of erythrocytes, and must be investigated. Breast milk jaundice occurs in one third of breastfed infants at 2 weeks and is caused by an insufficient intake of fluids.

Childbirth preparation can be considered successful if the outcome is described as follows: a. Labor and delivery were pain-free. b. The woman's partner participated eagerly. c. The woman rehearsed labor and practiced skills to master pain. d. Only nonpharmacologic methods for pain control were used.

ANS: C Preparation allows the woman to rehearse for labor and to learn new skills to cope with the pain of labor and the expected behavioral changes. Childbirth preparation does not guarantee a pain-free labor. A woman should be prepared for pain and anesthesia/analgesia realistically. The partner's role and participation level should be established by the couple. Women will not always achieve their desired level of pain control by using nonpharmacologic methods alone. PTS: 1 DIF: Cognitive Level: Evaluation/Evaluating REF: p. 356 OBJ: Nursing Process: Evaluation MSC: Client Needs: Psychosocial Integrity

Which of the following is a characteristic of a postterm infant who weighs 7 lb, 12 oz? a. A hematocrit level of 55%. b. Soft and supple skin. c. Lack of subcutaneous fat. d. An abundance of vernix caseosa.

ANS: C Rationale: A. The nails are usually long. B. The skin is wrinkled, cracked, and peeling. C. This postterm infant actually lost weight in utero, which is seen as loss of subcutaneous fat. D. There is no vernix caseosa.

5. Which type of formula is not diluted before being administered to an infant? a. Powdered b. Concentrated c. Ready-to-use d. Modified cow's milk

ANS: C Ready-to-use formula can be poured directly from the can into baby's bottle and is good (but expensive) when a proper water supply is not available. Powdered and concentrated formulas should be well mixed to dissolve the powder and make it uniform. Cow's milk is more difficult for the infant to digest and is not recommended, even if it is diluted.

Decreased surfactant production in the preterm lung is a problem because surfactant: a. Causes increased permeability of the alveoli. b. Provides transportation for oxygen to enter the blood supply. c. Keeps the alveoli open during expiration. d. Dilates the bronchioles, decreasing airway resistance.

ANS: C Surfactant prevents the alveoli from collapsing each time the infant exhales, thus reducing the work of breathing.

3. While assessing the newborn, the nurse should be aware that the average expected apical pulse range of a full-term, quiet, alert newborn is: a. 80 to 100 beats/min. c. 120 to 160 beats/min. b. 100 to 120 beats/min. d. 150 to 180 beats/min.

ANS: C The average infant heart rate while awake is 120 to 160 beats/min. The newborn's heart rate may be about 85 to 100 beats/min while sleeping. The infant's heart rate typically is a bit higher when alert but quiet. A heart rate of 150 to 180 beats/min is typical when the infant cries.

35. While assessing the newborn, the nurse should be aware that the average expected apical pulse range of a full-term, quiet, alert newborn is: a. 80 to 100 beats/min. c. 120 to 160 beats/min. b. 100 to 120 beats/min. d. 150 to 180 beats/min.

ANS: C The average infant heart rate while awake is 120 to 160 beats/min. The newborn's heart rate may be about 85 to 100 beats/min while sleeping. The infant's heart rate typically is a bit higher when alert but quiet. A heart rate of 150 to 180 beats/min is typical when the infant cries.

36. In administering vitamin K to the infant shortly after birth, the nurse understands that vitamin K is: a. Important in the production of red blood cells. b. Necessary in the production of platelets. c. Not initially synthesized because of a sterile bowel at birth. d. Responsible for the breakdown of bilirubin and prevention of jaundice.

ANS: C The bowel is initially sterile in the newborn, and vitamin K cannot be synthesized until food is introduced into the bowel. Vitamin K is necessary to activate blood clotting factors. The platelet count in term newborns is near adult levels. Vitamin K is necessary to activate prothrombin and other clotting factors.

3. When instructing parents on the correct use of a bulb syringe it is important include what information? a. Avoid suctioning the nares. b. Insert the compressed bulb into the center of the mouth. c. Suction the mouth first. d. Remove the bulb syringe from the crib when finished.

ANS: C The mouth should be suctioned first to prevent the infant from inhaling pharyngeal secretions by gasping as the nares are suctioned. The nasal passages should be suctioned one nostril at a time. The mouth should always be suctioned first. After compression of the bulb it should be inserted into one side of the mouth. If it is inserted into the center of the mouth, the gag reflex is likely to be initiated. The bulb syringe should remain in the crib so that it is easily accessible if needed again.

17. With regard to the newborn's developing cardiovascular system, nurses should be aware that: a. The heart rate of a crying infant may rise to 120 beats/min. b. Heart murmurs heard after the first few hours are cause for concern. c. The point of maximal impulse (PMI) often is visible on the chest wall. d. Persistent bradycardia may indicate respiratory distress syndrome (RDS).

ANS: C The newborn's thin chest wall often allows the PMI to be seen. The normal heart rate for infants who are not sleeping is 120 to 160 beats/min. However, a crying infant temporarily could have a heart rate of 180 beats/min. Heart murmurs during the first few days of life have no pathologic significance; an irregular heart rate past the first few hours should be evaluated further. Persistent tachycardia may indicate RDS; bradycardia may be a sign of congenital heart blockage.

14. A pregnant woman states "This test isn't my idea, but my husband insists." Which response by the nurse is most appropriate? a. "Don't worry. Everything will be fine." b. "Why don't you want to have this test?" c. "You're concerned about having this test?" d. "It's your decision."

ANS: C The nurse should clarify the statement and assist the patient in exploring her feelings about the test. Stating that everything will be fine is giving false reassurance and belittles the woman's concerns. "Why" questions usually put people on the defensive and are not therapeutic. Of course having the test is the woman's decision, but this closed statement does not encourage the woman to express her feelings. PTS: 1 DIF: Cognitive Level: Application/Applying REF: p. 286 OBJ: Integrated Process: Communication and Documentation MSC: Client Needs: Psychosocial Integrity

8. What is the first step in assisting the breastfeeding mother? a. Provide instruction on the composition of breast milk. b. Discuss the hormonal changes that trigger the milk ejection reflex. c. Assess the woman's knowledge of and feelings toward breastfeeding. d. Help her obtain a comfortable position and place the infant to the breast.

ANS: C The nurse should first assess the woman's knowledge and feedings toward breastfeeding to determine her teaching needs. Assessment should occur before instruction on positions and placing the infant to the breast. Education on hormonal changes and composition of breast milk also comes after assessment.

7. The nurse's role in diagnostic testing is to provide a. advice to the couple. b. assistance with decision making. c. information about the tests. d. reassurance about fetal safety.

ANS: C The nurse should provide the couple with all necessary information about a procedure so that the couple can make an informed decision. The nurse's role is to inform, not to advise the couple. Decision making should always lie with the couple involved. Ensuring fetal safety is not possible with all of the diagnostic testing. To offer this is to give false reassurance to the parents. PTS: 1 DIF: Cognitive Level: Knowledge/Remembering REF: p. 286 OBJ: Nursing Process: Implementation MSC: Client Needs: Physiologic Integrity

12. Which statement is true about the physiologic effects of pain in labor? a. It usually results in a more rapid labor. b. It is considered to be a normal occurrence. c. It may result in decreased placental perfusion. d. It has no effect on the outcome of labor.

ANS: C When experiencing excessive pain, the woman may react with a stress response that diverts blood flow from the uterus and the fetus. Excessive pain may prolong the labor due to increased anxiety in the woman. Pain is considered normal for labor, however; this statement does not explain the physiologic effects. Pain may affect the outcome of the labor depending on the cause and the effect on the woman. PTS: 1 DIF: Cognitive Level: Knowledge/Remembering REF: p. 354 OBJ: Nursing Process: Assessment MSC: Client Needs: Physiologic Integrity

9. The student nurse asks why gloves are needed when handling a newborn because the newborn "hasn't been exposed to anything." What response by the nurse is best? a. It is part of standard precautions. b. It is hospital policy. c. Amniotic fluid and maternal blood pose risks to us. d. We are protecting the infant from our bacteria.

ANS: C With the possibility of transmission of viruses such as HBV and HIV through maternal blood and amniotic fluid, the newborn must be considered a potential contamination source until proved otherwise. As part of standard precautions, nurses should wear gloves when handling the newborn until blood and amniotic fluid are removed by bathing. While this may be policy and is part of standard precautions, simply stating these facts does not convey any detailed information. The nurses are not protecting the infant from themselves.

The nurse explains to the nursing student that one mechanism for the diaphoresis and diuresis experienced during the early postpartum period is which of the following? a. Elevated temperature caused by postpartum infection b. Increased basal metabolic rate after giving birth c. Loss of increased blood volume associated with pregnancy d. Increased venous pressure in the lower extremities

ANS: C Within 12 hours of birth, women begin to lose the excess tissue fluid that has accumulated during pregnancy. One mechanism for reducing these retained fluids is the profuse diaphoresis that often occurs, especially at night, for the first 2 or 3 days after childbirth. Postpartal diuresis is another mechanism by which the body rids itself of excess fluid. An elevated temperature causes chills and may cause dehydration, not diaphoresis and diuresis. Diaphoresis and diuresis are not caused by an increase in the basal metabolic rate. Postpartal diuresis may be caused by the removal of increased venous pressure in the lower extremities

14. When the nurse is in the process of health teaching it is very important that he or she consider the family's cultural beliefs regarding child care. One of these beliefs includes that a. Arab women are anxious to breastfeed while still in the hospital. b. it is important to complement Asian parents about their new baby. c. women from India tie a black thread around the infant's waist. d. in the Korean culture the patient's mother is the primary caregiver of the infant.

ANS: C Women from India may tie a black thread around the infant's wrist, ankle, or waist to ward off evil spirits. This thread should not be removed by the nurse. Arab women are hesitant to breastfeed in the birth facility and wish to wait until they are home and their milk comes in. Asian parents may be uneasy when caregivers are too complementary about the baby or casually touch the infant's head. In the Korean culture, the husband's mother is the primary caregiver for the infant and the mother during the early weeks.

Which meal would provide the most absorbable iron? a. Toasted cheese sandwich, celery sticks, tomato slices, and a grape drink b. Oatmeal, whole wheat toast, jelly, and low-fat milk c. Black bean soup, wheat crackers, ambrosia (orange sections, coconut, and pecans), and prunes d. Red beans and rice, cornbread, mixed greens, and decaffeinated tea

ANS: C Black bean soup, wheat crackers, orange sections, and prunes Foods rich in iron include liver, meats, whole grain or enriched breads and cereals, deep green leafy vegetables, legumes, and dried fruits. The foods in this group are all good sources of iron. Additionally, the vitamin C in ambrosia (orange sections) aids absorption. Although there is protein in the cheese sandwich, absorbable iron is not provided. Oatmeal, toast with jelly, and dairy products are poor sources of iron. Tea does not contain iron. Although the legumes have some iron, this is not the optimal choice.

A pregnant woman's diet consists almost entirely of whole grain breads and cereals, fruits, and vegetables. The nurse would be most concerned about this woman's intake of: a. Calcium b. Protein c. Vitamin B12 d. Folic acid

ANS: C Vitamin B12 This diet is consistent with that followed by a strict vegetarian (vegan). Vegans consume only plant products. Because vitamin B12 is found in foods of animal origin, this diet is deficient in vitamin B12. Depending on the woman's food choices, this diet may be adequate in calcium. Protein needs can be sufficiently met by a vegetarian diet. The nurse should be more concerned with the woman's intake of vitamin B12 due to her dietary restrictions. Folic acid needs can be met by enriched bread products.

2. The nurse is caring for a laboring patient who develops a fever after she has had her epidural initiated. What actions by the nurse are appropriate? (Select all that apply.) a. Palpate the woman's bladder distention. b. Assess the woman's blood pressure. c. Observe the woman for shivering. d. Check the skin for color and warmth. e. Prepare to assist with a blood patch.

ANS: C, D Heat dissipation is reduced as a result of decreased hyperventilation, sweating, and activity after the onset of pain relief. Vasodilation redistributes heat from the core to the periphery of the body, where it is lost to the environment. Assessing the skin will demonstrate findings consistent with vasodilation. Shivering often occurs with sympathetic blockade accompanied by a dissociation between warm and cold sensations. In essence, the body believes that the temperature is lower than it actually is and turns up the "thermostat." Bladder distention is an anticipated effect of having an epidural. A woman's bladder fills quickly because of the large quantity of IV solution, yet her sensation to void is reduced. Maternal hypotension is an expected side effect of epidural initiation. The nurse should assess the bladder and blood pressure, but these actions are not related to the fever. A blood patch procedure is not warranted for this patient. PTS: 1 DIF: Cognitive Level: Application REF: p. 364 OBJ: Nursing Process: Implementation MSC: Client Needs: Physiologic Integrity

2. The baseline fetal heart rate (FHR) is the average rate during a 10-minute segment. Changes in FHR are categorized as periodic or episodic. These patterns include both accelerations and decelerations. The labor nurse is evaluating the patient's most recent 10-minute segment on the monitor strip and notes a late deceleration. This is likely to be caused by which physiologic alteration? Select all that apply. a. Spontaneous fetal movement b. Compression of the fetal head c. Placental abruption d. Cord around the baby's neck e. Maternal supine hypotension

ANS: C, E Feedback Correct Late decelerations are almost always caused by uteroplacental insufficiency. Insufficiency is caused by: uterine tachysystole, maternal hypotension, epidural or spinal anesthesia, IUGR, intraamniotic infection, or placental abruption. Incorrect Spontaneous fetal movement, vaginal examination, fetal scalp stimulation, fetal reaction to external sounds, uterine contractions, fundal pressure and abdominal palpation are all likely to cause accelerations of the FHR. Early decelerations are most often the result of fetal head compression and may be caused by uterine contractions, fundal pressure, vaginal examination and placement of an internal electrode. A variable deceleration is likely caused by umbilical cord compression. This may happen when the cord is around the baby's neck, arm, leg or other body part, a short cord, a knot in the cord or a prolapsed cord.

While assessing her patient, what does the nurse interpret as a positive sign of pregnancy? a. Fetal movement felt by the woman b. Amenorrhea c. Breast changes d. Visualization of fetus by ultrasound

ANS: D Feedback A Fetal movement is a presumptive sign of pregnancy. B Amenorrhea is a presumptive sign of pregnancy. C Breast changes are a presumptive sign of pregnancy. D The only positive signs of pregnancy are auscultation of fetal heart tones, visualization of the fetus by ultrasound, and fetal movement felt by the examiner.

While you are assessing the vital signs of a pregnant woman in her third trimester, the patient complains of feeling faint, dizzy, and agitated. Which nursing intervention is appropriate? a. Have the patient stand up and retake her blood pressure. b. Have the patient sit down and hold her arm in a dependent position. c. Have the patient lie supine for 5 minutes and recheck her blood pressure on both arms. d. Have the patient turn to her left side and recheck her blood pressure in 5 minutes.

ANS: D Feedback A Pressures are significantly higher when the patient is standing. This option causes an increase in systolic and diastolic pressures. B The arm should be supported at the same level of the heart. C The supine position may cause occlusion of the vena cava and descending aorta, creating hypotension. D Blood pressure is affected by positions during pregnancy. The supine position may cause occlusion of the vena cava and descending aorta. Turning the pregnant woman to a lateral recumbent position alleviates pressure on the blood vessels and quickly corrects supine hypotension.

A pregnant woman has come to the emergency department with complaints of nasal congestion and epistaxis. What is the correct interpretation of these symptoms by the practitioner? a. These conditions are abnormal. Refer the patient to an ear, nose, and throat specialist. b. Nasal stuffiness and nosebleeds are caused by a decrease in progesterone. c. Estrogen relaxes the smooth muscles in the respiratory tract, so congestion and epistaxis are within normal limits. d. Estrogen causes increased blood supply to the mucous membranes and can result in congestion and nosebleeds.

ANS: D Feedback A The patient should be reassured that these symptoms are within normal limits. No referral is needed at this time. B Progesterone is responsible for the heightened awareness of the need to breathe in pregnancy. Progesterone levels increase during pregnancy. C Progesterone affects relaxation of the smooth muscles in the respiratory tract. D As capillaries become engorged, the upper respiratory tract is affected by the subsequent edema and hyperemia, which causes these conditions, seen commonly during pregnancy.

The maternal task that begins in the first trimester and continues throughout the neonatal period is called a. Seeking safe passage for herself and her baby b. Securing acceptance of the baby by others c. Learning to give of herself d. Developing attachment with the baby

ANS: D Feedback A This is a task that ends with delivery. During this task the woman seeks health care and cultural practices. B This process continues throughout pregnancy as the woman reworks relationships. C This task occurs during pregnancy as the woman allows her body to give space to the fetus. She continues with giving to others in the form of food or presents. D Developing attachment (strong ties of affection) to the unborn baby begins in early pregnancy when the woman accepts that she is pregnant. By the second trimester, the baby becomes real and feelings of love and attachment surge.

A pregnant couple has formulated a birth plan and is reviewing it with the nurse at an expectant parent's class. Which aspect of their birth plan would be considered unrealistic and require further discussion with the nurse? a. "My husband and I have agreed that my sister will be my coach since he becomes anxious with regard to medical procedures and blood. He will be nearby and check on me every so often to make sure everything is OK." b. "We plan to use the techniques taught in the Lamaze classes to reduce the pain experienced during labor." c. "We want the labor and birth to take place in a birthing room. My husband will come in the minute the baby is born." d. "We do not want the fetal monitor used during labor, since it will interfere with movement and doing effleurage."

ANS: D Feedback A This is an acceptable request for a laboring woman. B Using breathing techniques to alleviate pain is a realistic part of a birth plan. C Not all fathers are able to be present during the birth; however, this couple has made a realistic plan that works for their specific situation. D Since monitoring is essential to assess fetal well-being, it is not a factor that can be determined by the couple. The nurse should fully explain its importance. The option for intermittent electronic monitoring could be explored if this is a low risk pregnancy and as long as labor is progressing normally. The birth plan is a tool with which parents can explore their childbirth options; however, the plan must be viewed as tentative.

A step in maternal role attainment that relates to the woman giving up certain aspects of her previous life is termed a. Looking for a fit b. Roleplaying c. Fantasy d. Grief work

ANS: D Feedback A This is when the woman observes the behaviors of mothers and compares them with her own expectations. B Roleplaying involves searching for opportunities to provide care for infants in the presence of another person. C Fantasies allow the woman to try on a variety of behaviors. This usually deals with how the child will look and the characteristics of the child. D The woman experiences sadness as she realizes that she must give up certain aspects of her previous self and that she can never go back.

Which assessment finding could indicate hemorrhage in the postpartum patient? a. Firm fundus at the midline b. Saturation of two perineal pads in 4 hours c. Elevated blood pressure d. Elevated pulse rate

ANS: D Feedback A A firm fundus indicates that the uterus is contracting and compressing the open blood vessels at the placental site. B Saturation of one pad within the first hour is the maximum normal amount of lochial flow. Two pads within 4 hours is within normal limits. C If the blood volume were diminishing, the blood pressure would decrease. D An increasing pulse rate is an early sign of excessive blood loss.

When preparing a woman for a cesarean birth, the nurse's care should include a. Injection of narcotic preoperative medications b. Full perineal shave preparation c. Straight catheterization to empty the bladder d. Administration of an oral antacid

ANS: D Feedback A A narcotic at this point would put the fetus at high risk for respiratory distress. B Perineal preparation is not necessary for a cesarean section. Some agencies will do an abdominal prep just before the surgery. C The catheterization should be indwelling in order to keep the bladder small during the surgery. D General anesthesia may be needed unexpectedly for cesarean birth. An oral antacid neutralizes gastric acid and reduces potential lung injury if the woman vomits and aspirates gastric contents during anesthesia.

What finding should the nurse recognize as being associated with fetal compromise? a. Active fetal movements b. Contractions lasting 90 seconds c. FHR in the 140s d. Meconium-stained amniotic fluid

ANS: D Feedback A Active fetal movement is an expected occurrence. B The fetus should be able to tolerate contractions lasting 90 seconds if the resting phase is sufficient to allow for a return of adequate blood flow. C Expected FHR range is from 120 to 160. D When fetal oxygen is compromised, relaxation of the rectal sphincter allows passage of meconium into the amniotic fluid.

17. Which action should be initiated to limit hypovolemic shock when uterine inversion occurs? a. Administer oxygen at 31 L/min by nasal cannula. b. Administer an oxytocic drug by intravenous push. c. Monitor fetal heart rate every 5 minutes. d. Restore circulating blood volume by increasing the intravenous infusion rate.

ANS: D Feedback A Administering oxygen will not prevent hypovolemic shock. B Oxytocin drugs should not be given until the uterus is repositioned. C A uterine inversion occurs during the third stage of labor. D Intravenous fluids are necessary to replace the lost blood volume that occurs in uterine inversion. PTS: 1 DIF: Cognitive Level: Application REF: p. 662 OBJ: Nursing Process: Implementation MSC: Client Needs: Physiologic Integrity

The standard of care for obstetrics dictates that an internal version might be used to manipulate the a. Fetus from a breech to a cephalic presentation before labor begins b. Fetus from a transverse lie to a longitudinal lie before cesarean birth c. Second twin from an oblique lie to a transverse lie before labor begins d. Second twin from a transverse lie to a breech presentation during vaginal birth

ANS: D Feedback A For internal version to occur, the cervix needs to be completely dilated. B For internal version to occur, the cervix needs to be dilated. C Internal version is done to turn the second twin after the first twin is born. D Internal version is used only during vaginal birth to manipulate the second twin into a presentation that allows it to be born vaginally.

18. In which situation is a dilation and curettage (D&C) indicated? a. Complete abortion at 8 weeks b. Incomplete abortion at 16 weeks c. Threatened abortion at 6 weeks d. Incomplete abortion at 10 weeks

ANS: D Feedback A If all the products of conception have been passed (complete abortion), a D&C is not used. B D&C is used to remove the products of conception from the uterus and can be done safely until week 14 of gestation. C If the pregnancy is still viable (threatened abortion), a D&C is not used. D D&C is used to remove the products of conception from the uterus and can be used safely until week 14 of gestation. PTS: 1 DIF: Cognitive Level: Comprehension REF: p. 578 OBJ: Nursing Process: Assessment MSC: Client Needs: Physiologic Integrity

30. A woman with preeclampsia has a seizure. The nurse's primary duty during the seizure is to a. Insert an oral airway. b. Suction the mouth to prevent aspiration. c. Administer oxygen by mask. d. Stay with the patient and call for help.

ANS: D Feedback A Insertion of an oral airway during seizure activity is no longer the standard of care. The nurse should attempt to keep the airway patent by turning the patient's head to the side to prevent aspiration. B Once the seizure has ended, it may be necessary to suction the patient's mouth. C Oxygen would be administered after the convulsion has ended. D If a patient becomes eclamptic, the nurse should stay with her and call for help. Nursing actions during a convulsion are directed towards ensuring a patent airway and patient safety. PTS: 1 DIF: Cognitive Level: Application REF: p. 597 OBJ: Nursing Process: Implementation MSC: Client Needs: Physiologic Integrity

14. With regard to the care management of preterm labor, nurses should be aware that a. Because all women must be considered at risk for preterm labor and prediction is so hit-and-miss, teaching pregnant women the symptoms probably causes more harm through false alarms. b. Braxton Hicks contractions often signal the onset of preterm labor. c. Because preterm labor is likely to be the start of an extended labor, a woman with symptoms can wait several hours before contacting the primary caregiver. d. The diagnosis of preterm labor is based on gestational age, uterine activity, and progressive cervical change.

ANS: D Feedback A It is essential that nurses teach women how to detect the early symptoms of preterm labor. B Braxton Hicks contractions resemble preterm labor contractions, but they are not true labor. C Waiting too long to see a health care provider could result in essential medications' failing to be administered. Preterm labor is not necessarily long-term labor. D Gestational age of 20 to 37 weeks, uterine contractions, and a thinning cervix are all indications of preterm labor. PTS: 1 DIF: Cognitive Level: Comprehension REF: pp. 646-647 OBJ: Nursing Process: Planning MSC: Client Needs: Safe and Effective Care Environment

The nurse practicing in a labor setting knows that the woman most at risk for a uterine rupture is a gravida a. 3 who has had two low-segment transverse cesarean births b. 2 who had a low-segment vertical incision for delivery of a 10-pound infant c. 5 who had two vaginal births and two cesarean births d. 4 who has had all cesarean births

ANS: D Feedback A Low-segment transverse cesarean scars do not predispose her to uterine rupture. B Low-segment incisions do not raise the risk of uterine ruptures. C This woman is not a high-risk candidate. D The risk of uterine rupture increases as the number of prior uterine incisions increases. More than 2 previous cesarean births places the woman at increased risk for uterine rupture.

22. What routine nursing assessment is contraindicated in the patient admitted with suspected placenta previa? a. Monitoring FHR and maternal vital signs b. Observing vaginal bleeding or leakage of amniotic fluid c. Determining frequency, duration, and intensity of contractions d. Determining cervical dilation and effacement

ANS: D Feedback A Monitoring FHR and maternal vital signs is a necessary part of the assessment for this woman. B Monitoring for bleeding and rupture of membranes is not contraindicated with this woman. C Monitoring contractions is not contraindicated with this woman. D Vaginal examination of the cervix may result in perforation of the placenta and subsequent hemorrhage. PTS: 1 DIF: Cognitive Level: Analysis REF: p. 584 OBJ: Nursing Process: Assessment MSC: Client Needs: Physiologic Integrity

16. A woman who had two previous cesarean births is in active labor, when she suddenly complains of pain between her scapulae. The nurse's priority action is to a. Reposition the woman with her hips slightly elevated. b. Observe for abnormally high uterine resting tone. c. Decrease the rate of nonadditive intravenous fluid. d. Notify the physician promptly and prepare the woman for surgery.

ANS: D Feedback A Repositioning the woman with her hips slightly elevated is the treatment for a prolapsed cord. That position in this scenario would cause respiratory difficulties. B Observing for high uterine resting tones should have been done before the sudden pain. High uterine resting tones put the woman at high risk for uterine rupture. C The woman is now at high risk for shock. Nonadditive intravenous fluids should be increased. D Pain between the scapulae may occur when the uterus ruptures, because blood accumulates under the diaphragm. This is an emergency that requires medical intervention. PTS: 1 DIF: Cognitive Level: Application REF: p. 661 OBJ: Nursing Process: Implementation MSC: Client Needs: Physiologic Integrity

Which comfort measure should the nurse use to assist the laboring woman to relax? a. Keep the room lights lit so that the patient and her coach can see everything. b. Offer warm, wet cloths to use on the patient's face and neck. c. Palpate her filling bladder every 15 minutes. d. Recommend frequent position changes.

ANS: D Feedback A Soft, indirect lighting is more soothing than irritating bright lights. B Women in labor become hot and perspire. Cool cloths are much better C A full bladder intensifies labor pain. The bladder should be emptied every 2 hours. D Frequent maternal position changes reduce the discomfort from constant pressure and promote fetal descent.

To adequately care for patients, the nurse understands that labor contractions facilitate cervical dilation by a. Contracting the lower uterine segment b. Enlarging the internal size of the uterus c. Promoting blood flow to the cervix d. Pulling the cervix over the fetus and amniotic sac

ANS: D Feedback A The contractions are stronger at the fundus. B The internal size becomes smaller with the contractions; this helps to push the fetus down. C Blood flow decreases to the uterus during a contraction. D Effective uterine contractions pull the cervix upward at the same time that the fetus and amniotic sac are pushed downward.

11. Which clinical sign is not included in the classic symptoms of preeclampsia? a. Hypertension b. Edema c. Proteinuria d. Glycosuria

ANS: D Feedback A The first indication of preeclampsia is usually an increase in the maternal blood pressure. B The first sign noted by the pregnant woman is a rapid weight gain and edema of the hands and face. C Proteinuria usually develops later than the edema and hypertension. D Spilling glucose into the urine is not one of the three classic symptoms of preeclampsia. PTS: 1 DIF: Cognitive Level: Knowledge REF: p. 592 OBJ: Nursing Process: Assessment MSC: Client Needs: Physiologic Integrity

19. The fetus in a breech presentation is often born by cesarean delivery because a. The buttocks are much larger than the head. b. Postpartum hemorrhage is more likely if the woman delivers vaginally. c. Internal rotation cannot occur if the fetus is breech. d. Compression of the umbilical cord is more likely.

ANS: D Feedback A The head is the largest part of a fetus. B There is no relationship between breech presentation and postpartum hemorrhage. C Internal rotation can occur with a breech. D After the fetal legs and trunk emerge from the woman's vagina, the umbilical cord can be compressed between the maternal pelvis and the fetal head if a delay occurs in the birth of the head. PTS: 1 DIF: Cognitive Level: Comprehension REF: p. 640 OBJ: Nursing Process: Assessment MSC: Client Needs: Physiologic Integrity

26. A 32-year-old primigravida is admitted with a diagnosis of ectopic pregnancy. Nursing care is based on the knowledge that a. Bed rest and analgesics are the recommended treatment. b. She will be unable to conceive in the future. c. A D&C will be performed to remove the products of conception. d. Hemorrhage is the major concern.

ANS: D Feedback A The recommended treatment is to remove the pregnancy before hemorrhaging. B If the tube must be removed, her fertility will decrease but she will not be infertile. C A D&C is done on the inside of the uterine cavity. The ectopic is located within the tubes. D Severe bleeding occurs if the fallopian tube ruptures. PTS: 1 DIF: Cognitive Level: Comprehension REF: p. 580 OBJ: Nursing Process: Planning MSC: Client Needs: Physiologic Integrity

A pregnant woman is at 38 weeks of gestation. She wants to know if any signs indicate "labor is getting closer to starting." The nurse informs the woman that which of the following is a sign that labor may begin soon? a. Weight gain of 1.5 to 2 kg (3 to 4 lb) b. Increase in fundal height c. Urinary retention d. Surge of energy

ANS: D Feedback A The woman may lose 0.5 to 1.5 kg, the result of water loss caused by electrolyte shifts, which in turn are caused by changes in the estrogen and progesterone levels. B When the fetus descends into the true pelvis (called lightening), the fundal height may decrease. C Urinary frequency may return before labor. D Women speak of having a burst of energy before labor.

A primigravida at 39 weeks of gestation is observed for 2 hours in the intrapartum unit. The fetal heart rate has been normal. Contractions are 5 to 9 minutes apart, 20 to 30 seconds in duration, and of mild intensity. Cervical dilation is 1 to 2 cm and uneffaced (unchanged from admission). Membranes are intact. The nurse should expect the woman to be a. Admitted and prepared for a cesarean birth b. Admitted for extended observation c. Discharged home with a sedative d. Discharged home to await the onset of true labor

ANS: D Feedback A These are all indications of false labor without fetal distress. There is no indication that a cesarean birth is indicated. B These are all indications of false labor; there is no indication that further assessment or observations are indicated. C The patient will probably be discharged, but there is no indication that a sedative is needed. D The situation describes a woman with normal assessments who is probably in false labor and will probably not deliver rapidly once true labor begins.

The priority nursing intervention after an amniotomy is to a. Assess the color of the amniotic fluid. b. Change the patient's gown. c. Estimate the amount of amniotic fluid. d. Assess the fetal heart rate.

ANS: D Feedback A This is important, but not the top priority. B This is important for patient comfort, but it is not the top priority. C This is not a top priority for this patient. D The fetal heart rate must be assessed immediately after the rupture of the membranes to determine whether cord prolapse or compression has occurred.

10. A laboring patient in the latent phase is experiencing uncoordinated, irregular contractions of low intensity. How should the nurse respond to complaints of constant cramping pain? a. "You are only 2 cm dilated, so you should rest and save your energy for when the contractions get stronger." b. "You must breathe more slowly and deeply so there is greater oxygen supply for your uterus. That will decrease the pain." c. "Let me take off the monitor belts and help you get into a more comfortable position." d. "I have notified the doctor that you are having a lot of discomfort. Let me rub your back and see if that helps."

ANS: D Feedback A This statement is belittling the patient's complaints. Support and comfort are necessary. B Breathing will not decrease the pain. C It is important to get her into a more comfortable position, but fetal monitoring should continue. D Intervention is needed to manage the dysfunctional pattern. Offering support and comfort is important to help the patient cope with the situation. PTS: 1 DIF: Cognitive Level: Application REF: p. 641 OBJ: Nursing Process: Implementation MSC: Client Needs: Health Promotion and Maintenance

15. Which nursing action must be initiated first when evidence of prolapsed cord is found? a. Notify the physician. b. Apply a scalp electrode. c. Prepare the mother for an emergency cesarean delivery. d. Reposition the mother with her hips higher than her head.

ANS: D Feedback A Trying to relieve pressure on the cord should be the first priority. B Trying to relieve pressure on the cord should take priority over increasing fetal monitoring techniques. C Emergency cesarean delivery may be necessary if relief of the cord is not accomplished. D The priority is to relieve pressure on the cord. Changing the maternal position will shift the position of the fetus so that the cord is not compressed. PTS: 1 DIF: Cognitive Level: Application REF: p. 659 OBJ: Nursing Process: Implementation MSC: Client Needs: Physiologic Integrity

Which actions by the nurse may prevent infections in the labor and delivery area? a. Vaginal examinations every hour while the woman is in active labor b. Use of clean techniques for all procedures c. Cleaning secretions from the vaginal area by using back-to-front motion d. Keeping underpads and linens as dry as possible

ANS: D Feedback A Vaginal examinations should be limited to decrease transmission of vaginal organisms into the uterine cavity. B Use an aseptic technique if membranes are not ruptured; use a sterile technique if membranes are ruptured. C Vaginal drainage should be removed with a front-to-back motion to decrease fecal contamination. D Bacterial growth prefers a moist, warm environment. PTS: 1 DIF: Cognitive Level: Application REF: p. 643 OBJ: Nursing Process: Implementation MSC: Client Needs: Safe and Effective Care Environment

5. A placenta previa in which the placental edge just reaches the internal os is called a. Total b. Partial c. Complete d. Marginal

ANS: D Feedback A With a total placenta previa the placenta completely covers the os. B With a partial previa the lower border of the placenta is within 3 cm of the internal cervical os, but does not completely cover the os. C A complete previa is termed total. The placenta completely covers the internal cervical os. D A placenta previa that does not cover any part of the cervix is termed marginal. PTS: 1 DIF: Cognitive Level: Knowledge REF: p. 583 OBJ: Nursing Process: Assessment MSC: Client Needs: Physiologic Integrity

21. A nurse might be called on to stimulate the fetal scalp a. As part of fetal scalp blood sampling b. In response to tocolysis c. In preparation for fetal oxygen saturation monitoring d. To elicit an acceleration in the FHR

ANS: D Feedback A Fetal scalp blood sampling involves swabbing the scalp with disinfectant before a sample is collected. The nurse would stimulate the fetal scalp to elicit an acceleration of the FHR. B Tocolysis is relaxation of the uterus. The nurse would stimulate the fetal scalp to elicit an acceleration of the FHR. C Fetal oxygen saturation monitoring involves the insertion of a sensor. The nurse would stimulate the fetal scalp to elicit an acceleration of the FHR. D The scalp can be stimulated using digital pressure during a vaginal examination.

16. Increasing the infusion rate of nonadditive intravenous fluids can increase fetal oxygenation primarily by a. Maintaining normal maternal temperature b. Preventing normal maternal hypoglycemia c. Increasing the oxygen-carrying capacity of the maternal blood d. Expanding maternal blood volume

ANS: D Feedback A Increasing fluid volume may alter the maternal temperature only if she is dehydrated. B Most intravenous fluids for laboring women are isotonic and do not add extra glucose. C Oxygen-carrying capacity is increased by adding more red blood cells. D Filling the mother's vascular system makes more blood available to perfuse the placenta and may correct hypotension.

11. When the deceleration pattern of the fetal heart rate mirrors the uterine contraction, which nursing action is indicated? a. Administer oxygen by nasal cannula. b. Reposition the woman. c. Apply a fetal scalp electrode. d. Record this reassuring pattern.

ANS: D Feedback A This is an early deceleration; it is reassuring. B This is an early deceleration; it is reassuring. C This is an early deceleration; it is reassuring. D The periodic pattern described is early deceleration that is not associated with fetal compromise and requires no intervention.

12. When the mother's membranes rupture during active labor, the fetal heart rate should be observed for the occurrence of which periodic pattern? a. Increase in baseline variability b. Nonperiodic accelerations c. Early decelerations d. Variable decelerations

ANS: D Feedback A This is not an expected occurrence after the rupture of membranes. B Accelerations are considered reassuring; they are not a concern after rupture of membranes. C Early declarations are considered reassuring; they are not a concern after rupture of membranes. D When the membranes rupture, amniotic fluid may carry the umbilical cord to a position where it will be compressed between the maternal pelvis and the fetal presenting part, resulting in a variable deceleration pattern.

19. When using IA for FHR, nurses should be aware that a. They can be expected to cover only two or three patients when IA is the primary method of fetal assessment. b. The best course is to use the descriptive terms associated with EFM when documenting results. c. If the heartbeat cannot be found immediately, a shift must be made to electronic monitoring. d. Ultrasound can be used to find the fetal heartbeat and reassure the mother if initial difficulty was a factor.

ANS: D Feedback A When used as the primary method of fetal assessment, auscultation requires a nurse-to-patient ratio of one to one. B Documentation should use only terms that can be numerically defined; the usual visual descriptions of EFM are inappropriate. C Locating fetal heartbeats often takes time. Mothers can be reassured verbally and by the ultrasound pictures if that device is used to help locate the heartbeat. D Locating fetal heartbeats often takes time. Mothers can be reassured verbally and by the ultrasound pictures if that device is used to help locate the heartbeat.

Which vitamins or minerals can lead to congenital malformations of the fetus if taken in excess by the mother? a. Zinc b. Vitamin D c. Folic acid d. Vitamin A

ANS: D If taken in excess, vitamin A causes a number of problems. An analog of vitamin A appears in prescribed acne medications, which must not be taken during pregnancy. Zinc is vital to good maternity and fetal health and is highly unlikely to be consumed in excess. Vitamin D is vital to good maternity and fetal health and is highly unlikely to be consumed in excess. Folic acid is vital to good maternity and fetal health and is highly unlikely to be consumed in excess.

Nutrition is one of the most significant factors influencing the outcome of a pregnancy. It is an alterable and important preventive measure for various potential problems, such as low birth weight and prematurity. While completing the physical assessment of the pregnant client, the nurse can evaluate the client's nutritional status by observing a number of physical signs. Which sign would indicate that the client has unmet nutritional needs? a. Normal heart rate, rhythm, and blood pressure b. Bright, clear, shiny eyes c. Alert, responsive, and good endurance d. Edema, tender calves, and tingling

ANS: D The physiologic changes of pregnancy may complicate the interpretation of physical findings. Lower extremity edema often occurs when caloric and protein deficiencies are present; however, it may also be a common physical finding during the third trimester. It is essential that the nurse complete a thorough health history and physical assessment, and request further laboratory testing if indicated. The malnourished pregnant client may display rapid heart rate, abnormal rhythm, enlarged heart, and elevated blood pressure. A client receiving adequate nutrition will have bright, shiny eyes with no sores and moist, pink membranes. Pale or red membranes, dryness, infection, dull appearance of the cornea, or blue sclerae are signs of poor nutrition. This client is well nourished. If she presented as listless, cachectic, easily fatigued, and tired this would be an indication of poor nutritional status.

8. The most dangerous effect on the fetus of a mother who smokes cigarettes while pregnant is a. Genetic changes and anomalies b. Extensive central nervous system damage c. Fetal addiction to the substance inhaled d. Intrauterine growth restriction

ANS: D Feedback A Cigarettes normally will not cause genetic changes. B Cigarettes normally will not cause extensive central nervous system damage. C Addiction is not a normal concern with the neonate. D The major consequences of smoking tobacco during pregnancy are low-birth-weight infants, prematurity, and increased perinatal loss.

10. As a powerful central nervous system stimulant, which of these substances can lead to miscarriage, preterm labor, placental separation (abruption), and stillbirth? a. Heroin b. Alcohol c. PCP d. Cocaine

ANS: D Feedback A Heroin is an opiate. Its use in pregnancy is associated with preeclampsia, intrauterine growth restriction, miscarriage, premature rupture of membranes, infections, breech presentation, and preterm labor. B The most serious effect of alcohol use in pregnancy is FAS. C The major concerns regarding PCP use in pregnant women are its association with polydrug abuse and the neurobehavioral effects on the neonate. D Cocaine is a powerful CNS stimulant. Effects on pregnancy associated with cocaine use include abruptio placentae, preterm labor, precipitous birth, and stillbirth.

13. An appropriate nursing measure when a baby has an unexpected anomaly is to a. Remove the baby from the delivery area immediately. b. Tell the parents that the baby has to go to the nursery immediately. c. Inform the parents immediately that something is wrong. d. Explain the defect and show the baby to the parents as soon as possible.

ANS: D Feedback A The parents should be able to touch and hold the baby as soon as possible. B This would raise anxiety levels of the parents; they should be told about the defect and allowed to see the baby. C They should be told immediately, but they should be told about the defect and be allowed to see the infant. D Parents experience less anxiety when they are told about the defect as early as possible and are allowed to touch and hold the baby.

12. A woman has delivered twins. The first twin was stillborn, and the second is in the intensive care nursery and is recovering quickly from respiratory distress. The woman is crying softly and says, "I wish my baby could have lived." What is the most therapeutic response? a. "Don't be sad. At least you have one healthy baby." b. "How soon do you plan to have another baby?" c. "I have a friend who lost a twin and she's doing just fine now." d. "I am so sorry about your loss. Would you like to talk about it?"

ANS: D Feedback A This is denying the loss of the other infant. B This is denying the loss of the infant and her grief and belittling her feelings. C This is belittling her feelings. D The nurse should recognize the woman's grief and its significance.

5. An African-American woman noticed some bruises on her newborn girl's buttocks. She asks the nurse who spanked her daughter. The nurse explains that these marks are called: a. Lanugo. c. Nevus flammeus. b. Vascular nevi. d. Mongolian spots.

ANS: D A Mongolian spot is a bluish black area of pigmentation that may appear over any part of the exterior surface of the body. It is more commonly noted on the back and buttocks and most frequently is seen on infants whose ethnic origins are Mediterranean, Latin American, Asian, or African. Lanugo is the fine, downy hair seen on a term newborn. A vascular nevus, commonly called a strawberry mark, is a type of capillary hemangioma. A nevus flammeus, commonly called a port-wine stain, is most frequently found on the face.

28. During life in utero, oxygenation of the fetus occurs through transplacental gas exchange. When birth occurs, four factors combine to stimulate the respiratory center in the medulla. The initiation of respiration then follows. Which is not one of these essential factors? a. Chemical c. Thermal b. Mechanical d. Psychologic

ANS: D A psychologic factor is not one of the essential factors in the initiation of breathing; the fourth factor is sensory. The sensory factors include handling by the provider, drying by the nurse, lights, smells, and sounds. Chemical factors are essential for the initiation of breathing. During labor, decreased levels of oxygen and increased levels of carbon dioxide seem to have a cumulative effect that is involved in the initiation of breathing. Clamping of the cord may also contribute to the start of respirations. Prostaglandins are known to inhibit breathing, and clamping of the cord results in a drop in the level of prostaglandins. Mechanical factors also are necessary to initiate respirations. As the infant passes through the birth canal, the chest is compressed. With birth the chest is relaxed, which allows for negative intrathoracic pressure that encourages air to flow into the lungs. The profound change in temperature between intrauterine and extrauterine life stimulates receptors in the skin to communicate with the receptors in the medulla. This also contributes to the initiation of breathing.

10. A postpartum woman telephones about her 4-day-old infant. She is not scheduled for a weight check until the infant is 10 days old, and she is worried about whether breastfeeding is going well. Effective breastfeeding is indicated by the newborn who a. sleeps for 6 hours at a time between feedings. b. has at least one breast milk stool every 24 hours. c. gains 1 to 2 ounces per week. d. has at least six to eight wet diapers per day.

ANS: D After day 4, when the mother's milk comes in, the infant should have six to eight wet diapers every 24 hours. Typically infants sleep 2 to 4 hours between feedings, depending on whether they are being fed on a 2- to 3-hour schedule or cluster-fed. The infant should have a minimum of three bowel movements in a 24-hour period. The mother will not know what her child weighs until the appointment and so the nurse needs to provide her with other indicators of successful breastfeeding.

20. One of the greatest risks to the mother during administration of general anesthesia is a. respiratory depression. b. uterine relaxation. c. inadequate muscle relaxation. d. aspiration of stomach contents.

ANS: D Aspiration of acidic gastric contents and possible airway obstruction is a potentially fatal complication of general anesthesia. Respirations can be altered during general anesthesia, and the anesthesiologist will take precautions to maintain proper oxygenation. Uterine relaxation can occur with some anesthesia, but this can be monitored and prevented. Inadequate muscle relaxation can be altered. This is not the greatest risk for the mother. PTS: 1 DIF: Cognitive Level: Knowledge/Remembering REF: p. 367 OBJ: Nursing Process: Assessment MSC: Client Needs: Physiologic Integrity

4. In providing and teaching cord care, what is an important principle? a. Cord care is done only to control bleeding. b. Alcohol is the only agent used for cord care. c. It takes a minimum of 24 days for the cord to separate. d. The process of keeping the cord dry will decrease bacterial growth.

ANS: D Bacterial growth increases in a moist environment, so keeping the umbilical cord dry impedes bacterial growth. Cord care is to prevent infection and add in the drying of the cord. No agents are necessary to facilitate drying of the cord. The cord will fall off within 10 to 14 days.

The nurse caring for the postpartum woman understands that breast engorgement is caused by a. overproduction of colostrum. b. accumulation of milk in the lactiferous ducts and glands. c. hyperplasia of mammary tissue. d. congestion of veins and lymphatics.

ANS: D Breast engorgement is caused by the temporary congestion of veins and lymphatics, not overproduction of colostrum, accumulation of milk, or hyperplasia.

22. Which statement describing physiologic jaundice is incorrect? a. Neonatal jaundice is common, but kernicterus is rare. b. The appearance of jaundice during the first 24 hours or beyond day 7 indicates a pathologic process. c. Because jaundice may not appear before discharge, parents need instruction on how to assess it and when to call for medical help. d. Breastfed babies have a lower incidence of jaundice.

ANS: D Breastfeeding is associated with an increased incidence of jaundice. Neonatal jaundice occurs in 60% of newborns; the complication called kernicterus is rare. Jaundice in the first 24 hours or that persists past day 7 is cause for medical concern. Parents need to know how to assess for jaundice in their newborn.

10. The major advantage of chorionic villus sampling (CVS) over amniocentesis is that it a. is not an invasive procedure. b. does not require hospitalization. c. has less risk of spontaneous abortion. d. is performed earlier in pregnancy.

ANS: D CVS is performed between 10 and 13 weeks of gestation, providing earlier results than amniocentesis, which is normally done during the second and third trimesters, although it can be done as early as 11 weeks if needed. The woman does not need hospitalization for this invasive procedure, and the risk of spontaneous abortion is about the same for both procedures. PTS: 1 DIF: Cognitive Level: Knowledge/Remembering REF: p. 279 OBJ: Nursing Process: Assessment MSC: Client Needs: Physiologic Integrity

To determine a preterm infant's readiness for nipple feeding, the nurse should assess the: a. Skin turgor. b. Bowel sounds. c. Current weight. d. Respiratory rate

ANS: D Coordination of suck, swallow, and breathing is a common task for preterm infants. The infant must have a respiratory rate less than 60 breaths/min before nipple feeding can be implemented; skin turgor, bowel sounds, and current weight are not indications for nipple feeding.

6. When teaching parents about mandatory newborn screening, it is important for the nurse to explain that the main purpose is to a. keep the state records updated. b. allow accurate statistical information. c. document the number of births. d. recognize and treat newborn disorders early.

ANS: D Early treatment of disorders will prevent morbidity associated with inborn errors of metabolism or other genetic conditions. Keeping and updating records are not the reasons for the testing.

26. Which type of cutaneous stimulation involves massage of the abdomen? a. Thermal stimulation b. Imagery c. Mental stimulation d. Effleurage

ANS: D Effleurage is massage usually performed on the abdomen during contractions. Thermal stimulation is the use of warmth to provide comfort, such as showers and baths. Imagery involves the woman creating a relaxing mental scene and dissociating herself from the painful aspects of labor. Mental stimulation occupies the woman's mind and competes with pain stimuli. PTS: 1 DIF: Cognitive Level: Knowledge/Remembering REF: p. 357 OBJ: Nursing Process: Implementation MSC: Client Needs: Physiologic Integrity

4. While teaching an early pregnancy class, the nurse explains that the morula is a a. Fertilized ovum before mitosis begins b. Flattened disc-shaped layer of cells within a fluid-filled sphere c. Double layer of cells that becomes the placenta d. Solid ball composed of the first cells formed after fertilization

ANS: D Feedback D The morula is so named because it resembles a mulberry. It is a solid ball of 12 to 16 cells that develops after fertilization.

18. The most basic information a maternity nurse should have concerning conception is a. Ova are considered fertile 48 to 72 hours after ovulation. b. Sperm remain viable in the woman's reproductive system for an average of 12 to 24 hours. c. Conception is achieved when a sperm successfully penetrates the membrane surrounding the ovum. d. Implantation in the endometrium occurs 6 to 10 days after conception.

ANS: D Feedback D After implantation, the endometrium is called the decidua.

13. When explaining twin conception, the nurse points out that dizygotic twins develop from a. A single fertilized ovum and are always of the same sex b. A single fertilized ovum and may be the same sex or different sexes c. Two fertilized ova and are the same sex d. Two fertilized ova and may be the same sex or different sexes

ANS: D Feedback D Dizygotic twins are two different zygotes, each conceived from a single ovum and a single sperm. They may be both male, both female, or one male and one female.

6. Some of the embryo's intestines remain within the umbilical cord during the embryonic period because the a. Umbilical cord is much larger at this time than it will be at the end of pregnancy. b. Intestines begin their development within the umbilical cord. c. Nutrient content of the blood is higher in this location. d. Abdomen is too small to contain all the organs while they are developing.

ANS: D Feedback D The abdominal contents grow more rapidly than the abdominal cavity, so part of their development takes place in the umbilical cord. By 10 weeks, the abdomen is large enough to contain them.

15. Between 6 and 10 days after conception, the trophoblast secretes enzymes that enable it to burrow into the endometrium until the entire blastocyst is covered. This is termed implantation. Tiny projections then develop out of the trophoblast and extend into the endometrium. These projections are referred to as a. Decidua basalis b. Decidua capsularis c. Decidua vera d. Chorionic villi

ANS: D Feedback D These villi are vascular processes that obtain oxygen and nutrients from the maternal bloodstream and dispose of carbon dioxide and waste products into the maternal blood.

Which maternal event is abnormal in the early postpartum period? a. Diuresis and diaphoresis b. Flatulence and constipation c. Extreme hunger and thirst d. Lochial color changes from rubra to alba

ANS: D For the first 3 days after childbirth, lochia is mostly red and is termed rubra. Lochia serosa follows, and then at about 11 days, the discharge becomes clear, colorless, or white. The body rids itself of increased plasma volume after birth. Urine output of 3000 mL/day is common for the first few days after delivery and is facilitated by hormonal changes in the mother. Bowel tone remains sluggish for days after birth, leading to flatulence and constipation. The new mother is hungry and thirsty because of energy used in labor and thirsty because of fluid restrictions during labor.

Which condition seen in the postpartum period is likely to require careful medical assessment? a. Varicosities of the legs b. Carpal tunnel syndrome c. Periodic numbness and tingling of the fingers d. Headaches

ANS: D Headaches in the postpartum period can have a number of causes, some of which deserve medical attention. Varicosities are common. Carpal tunnel syndrome is relieved in childbirth when the compression on the median nerve is lessened. Periodic numbness of the fingers usually disappears after birth unless carrying the baby aggravates the condition.

19. An important aspect about storage of breast milk is that it a. can be frozen for up to 2 months. b. should be stored only in glass bottles. c. can be thawed and refrozen. d. can be kept refrigerated for 48 hours.

ANS: D If used within 48 hours after being refrigerated, breast milk will maintain its full nutritional value.

5. When providing labor support, the nurse knows that which fetal position might cause the laboring woman more back discomfort? a. Right occiput anterior b. Left occiput anterior c. Right occiput transverse d. Left occiput posterior

ANS: D In the left occiput posterior position, each contraction pushes the fetal head against the mother's sacrum, which results in intense back discomfort. The other fetal positions do not cause more back discomfort. PTS: 1 DIF: Cognitive Level: Knowledge/Remembering REF: p. 358 OBJ: Nursing Process: Assessment MSC: Client Needs: Health Promotion and Maintenance

A postpartum woman is unable to empty her bladder. What intervention would the nurse try last? a. Pouring water from a squeeze bottle over the woman's perineum b. Providing hot tea c. Asking the physician to prescribe analgesics d. Inserting a sterile catheter

ANS: D Invasive procedures usually are the last to be tried, especially with so many other simple methods available. Pouring water over the perineum may stimulate voiding. It is easy, noninvasive, and should be tried early on. Hot tea or other fluids ad lib is an easy, noninvasive strategy that should be tried early on. If the woman is anticipating pain from voiding, pain medications may be helpful. Other nonmedical means could be tried first, but medications still come before insertion of a catheter.

17. The method of anesthesia in labor considered the safest for the fetus is the a. pudendal block. b. epidural block. c. spinal (subarachnoid) block. d. local infiltration.

ANS: D Local infiltration of the perineum rarely has any adverse effects on either the mother or the fetus. The fetus can be affected by maternal side effects of the other types of anesthesia. PTS: 1 DIF: Cognitive Level: Knowledge/Remembering REF: p. 362 OBJ: Nursing Process: Assessment MSC: Client Needs: Health Promotion and Maintenance

16. Which statement regarding various biochemical assessments used during pregnancy is correct? a. Chorionic villus sampling (CVS) is becoming more popular because it provides early diagnosis. b. Screening for maternal serum alpha-fetoprotein (MSAFP) levels is recommended between 10 and 12 weeks of gestation in order to give parents time to consider options. c. Percutaneous umbilical blood sampling (PUBS) is one of the multiple marker screen tests for Down syndrome. d. MSAFP is a screening tool only; it identifies candidates for more definitive procedures.

ANS: D MSAFP is a screening tool, not a diagnostic tool. Further diagnostic testing is indicated after an abnormal MSAFP. CVS does provide a rapid result, but it is declining in popularity because of advances in noninvasive screening techniques. MSAFP screening is recommended for all pregnant women. Screening is recommended between 15 and 20 weeks of gestation. Abnormal findings give parents time to have additional tests done. PTS: 1 DIF: Cognitive Level: Comprehension/Understanding REF: p. 277 OBJ: Integrated Process: Teaching-Learning MSC: Client Needs: Health Promotion and Maintenance

28. Which technique could the support person use when the laboring woman appears to be losing control? a. Have the nurse take over the role of support. b. Tell the woman that she is causing stress to her baby and herself. c. Wait for the contraction to end and discuss the problem with her. d. Make eye contact with the woman and breathe along with her.

ANS: D Making eye contact and breathing along with the laboring woman to help pace her breathing will assist her in remaining calm. The woman already has a trusting relationship with the support person so they should stay in that position if possible. Telling the woman she is stressing herself and the baby is very uncaring and will not be helpful. A woman who has lost control will not be able to engage in a productive discussion. PTS: 1 DIF: Cognitive Level: Comprehension/Understanding REF: p. 371 OBJ: Nursing Process: Implementation MSC: Client Needs: Psychosocial Integrity

A 25-year-old gravida 1 para 1 who had an emergency cesarean birth 3 days ago is scheduled for discharge. As the nurse prepares her for discharge, she begins to cry. What action should the nurse take first? a. Assess her for pain. b. Point out how lucky she is to have a healthy baby. c. Explain that she is experiencing postpartum blues. d. Allow her time to express her feelings.

ANS: D Many women experience transient postpartum blues and need assistance in expressing their feelings. This condition affects 70% to 80% of new mothers. The nurse should allow time for the new mother to express herself. The nurse should not assume she is in pain at this point. Pointing out how lucky she is belittles her feelings. Patient teaching can be done later.

3. How can the nurse help the mother who is breastfeeding and has engorged breasts? a. Suggest that she switch to bottled formula just for today. b. Assist her in removing her bra, making her more comfortable. c. Apply heat to her breasts between feeding and cold to the breasts just before feedings. d. Instruct and assist the mother to massage her breasts.

ANS: D Massage of the breasts causes release of oxytocin and increases the speed of milk release. Engorgement is more likely to increase if breastfeeding is delayed or infrequent. A well-fitting bra should be worn both day and night to support the breasts. Cold applications are used between feedings to reduce edema and pain. Heat is applied just before feedings to increase vasodilation.

26. The Baby Friendly Hospital Initiative was founded to encourage institutions to offer optimal levels of care for lactating mothers. Which is of the following is inconsistent with the nurse's knowledge about the "Ten Steps to Successful Breastfeeding for Hospitals"? a. Give newborns no food or drink other than breast milk. b. Have a written breastfeeding policy that is communicated to all staff. c. Help mothers initiate breastfeeding within one half-hour of birth. d. Give artificial pacifiers as necessary.

ANS: D No artificial pacifiers should be given to breastfeeding infants. The other statements are consistent with the "Ten Steps to Successful Breastfeeding for Hospitals."

14. Which method of pain management does the nurse plan for a gravida 3 para 2 admitted at 8-cm cervical dilation? a. Epidural anesthesia b. Narcotics c. Spinal block d. Breathing and relaxation techniques

ANS: D Nonpharmacologic methods of pain management may be the best option for a woman in advanced labor. There is probably not enough remaining time to administer epidural anesthesia or spinal anesthesia. A narcotic given at this time may reach its peak about the time of birth and result in respiratory depression in the newborn. PTS: 1 DIF: Cognitive Level: Application/Applying REF: p. 357 OBJ: Nursing Process: Planning MSC: Client Needs: Health Promotion and Maintenance

If rubella vaccine is indicated for a postpartum patient, instructions to the patient should include a. drinking plenty of fluids to prevent fever. b. no specific instructions. c. recommending that she stop breastfeeding for 24 hours after injection. d. explaining the risks of becoming pregnant within 1 month after injection.

ANS: D Potential risks to the fetus can occur if pregnancy results within 28 days after rubella vaccine administration. Drinking fluids will not prevent a fever. Small amounts of the vaccine do cross the breast milk, but it is believed that there is no need to discontinue breastfeeding.

Which nursing action is especially important for an SGA newborn? a. Observe for respiratory distress syndrome. b. Observe for and prevent dehydration. c. Promote bonding. d. Prevent hypoglycemia with early and frequent feedings.

ANS: D Rationale: A. Respiratory distress syndrome is seen in preterm infants. B. Dehydration is a concern for all infants and not specific for SGA. C. Promoting bonding is a concern for all infants and not specific for SGA. D. SGA infants have poor glycogen stores and are subject to hypoglycemia.

Which is true about newborns classified as small for gestational age (SGA)? a. They weigh less than 2500 g. b. They are born before 38 weeks of gestation. c. Placental malfunction is the only recognized cause of this condition. d. They are below the 10th percentile on gestational growth charts.

ANS: D Rationale: A. SGA infants are defined as below the 10th percentile in growth compared with other infants of the same gestational age. It is not defined by weight. B. Infants born prior to 38 weeks are defined as preterm. C. There are many causes of SGA babies. D. SGA infants are defined as below the 10th percentile in growth compared with other infants of the same gestational age.

In comparison with the term infant, the preterm infant has: a. Few blood vessels visible though the skin. b. More subcutaneous fat. c. Well-developed flexor muscles. d. Greater surface area in proportion to weight.

ANS: D Rationale: A. This is an indication of a more mature infant. B. This is an indication of a more mature infant. C. This is an indication of a more mature infant. D. Preterm infants have greater surface area in proportion to their weight.

12. An NST in which two or more fetal heart rate (FHR) accelerations of 15 beats per minute (bpm) or more occur with fetal movement in a 20-minute period is termed a. nonreactive. b. positive. c. negative. d. reactive.

ANS: D The NST is reactive (normal) when two or more FHR accelerations of at least 15 bpm (each with a duration of at least 15 seconds) occur in a 20-minute period. A nonreactive result means that the heart rate did not accelerate during fetal movement. Positive and negative are not results given with this test. PTS: 1 DIF: Cognitive Level: Knowledge/Remembering REF: p. 281 OBJ: Nursing Process: Assessment MSC: Client Needs: Physiologic Integrity

16. When responding to the question "Will I produce enough milk for my baby as she grows and needs more milk at each feeding?" the nurse should explain that a. the breast milk will gradually become richer to supply additional calories. b. as the infant requires more milk, feedings can be supplemented with cow's milk. c. early addition of baby food will meet the infant's needs. d. the mother's milk supply will increase as the infant demands more at each feeding.

ANS: D The amount of milk produced depends on the amount of stimulation of the breast. Increased demand with more frequent and longer breastfeeding sessions results in more milk available for the infant. Mature breast milk will stay the same. The amounts will increase as the infant feeds for longer times. Supplementation will decrease the amount of stimulation of the breast and decrease the milk production. Solids should not be added until about 4 to 6 months, when the infant's immune system is more mature. This will decrease the chance of allergy formations.

4. When is the best time to determine gestational age based on biparietal diameter through ultrasound? a. First trimester only b. Second trimester only c. Any time d. Second half of pregnancy

ANS: D The biparietal diameter is used to determine gestational age during the second half of pregnancy. PTS: 1 DIF: Cognitive Level: Knowledge/Remembering REF: p. 276 OBJ: Nursing Process: Assessment MSC: Client Needs: Health Promotion and Maintenance

10. While evaluating the reflexes of a newborn, the nurse notes that with a loud noise the newborn symmetrically abducts and extends his arms, his fingers fan out and form a "C" with the thumb and forefinger, and he has a slight tremor. The nurse would document this finding as a positive: a. Tonic neck reflex. c. Babinski reflex. b. Glabellar (Myerson) reflex. d. Moro reflex.

ANS: D The characteristics displayed by the infant are associated with a positive Moro reflex. The tonic neck reflex occurs when the infant extends the leg on the side to which the infant's head simultaneously turns. The glabellar reflex is elicited by tapping on the infant's head while the eyes are open. A characteristic response is blinking for the first few taps. The Babinski reflex occurs when the sole of the foot is stroked upward along the lateral aspect of the sole and then across the ball of the foot. A positive response occurs when all the toes hyperextend, with dorsiflexion of the big toe.

30. A nursing student is helping the nursery nurses with morning vital signs. A baby born 10 hours ago by cesarean section is found to have moist lung sounds. What is the best interpretation of these data? a. The nurse should notify the pediatrician stat for this emergency situation. b. The neonate must have aspirated surfactant. c. If this baby was born vaginally, it could indicate a pneumothorax. d. The lungs of a baby delivered by cesarean section may sound moist for 24 hours after birth.

ANS: D The condition will resolve itself within a few hours. For this common condition of newborns, surfactant acts to keep the expanded alveoli partially open between respirations. In vaginal births, absorption of remaining lung fluid is accelerated by the process of labor and delivery. Remaining lung fluid will move into interstitial spaces and be absorbed by the circulatory and lymphatic systems. This is a particularly common condition for infants delivered by cesarean section. Surfactant is produced by the lungs, so aspiration is not a concern.

15. Which statement describing the first phase of the transition period is inaccurate? a. It lasts no longer than 30 minutes. b. It is marked by spontaneous tremors, crying, and head movements. c. It includes the passage of meconium. d. It may involve the infant's suddenly sleeping briefly.

ANS: D The first phase is an active phase in which the baby is alert. Decreased activity and sleep mark the second phase. The first phase is the shortest, lasting less than 30 minutes. Such exploratory behaviors include spontaneous startle reactions. In the first phase the newborn also produces saliva.

12. A nurse is responsible for teaching new parents about the hygienic care of their newborn. What information does the nurse include? a. Avoid washing the head for at least 1 week to prevent heat loss. b. Sponge bathe only until the cord has fallen off. c. Cleanse the ears and nose with cotton-tipped swabs, such as Q-tips. d. Water temperature should be at least 38° C.

ANS: D The ideal temperature of the bath water should be at least 38° C, or 100.4° F. The head can be washed. Tub baths may be initiated from birth. Ensure that the infant is fully immersed. Q-tips should not be used, because they may cause injury. A corner of a moistened washcloth should be twisted into shape so that it can be used to cleanse the ears and nose.

9. A primiparous woman wants to begin breastfeeding as soon as possible. The nurse can facilitate the infant's correct latch-on by helping the woman hold the infant a. with his arms folded together over his chest. b. curled up in a fetal position. c. with his head cupped in her hand. d. with his head and body in alignment.

ANS: D The infant's head and body should be in correct alignment with the mother and the breast during latch-on and feeding. The other positions do not facilitate nursing.

19. What infant response to cool environmental conditions is either not effective or not available to them? a. Constriction of peripheral blood vessels b. Metabolism of brown fat c. Increased respiratory rates d. Unflexing from the normal position

ANS: D The newborn's flexed position guards against heat loss because it reduces the amount of body surface exposed to the environment. The newborn's body is able to constrict the peripheral blood vessels to reduce heat loss. Burning brown fat generates heat. The respiratory rate may rise to stimulate muscular activity, which generates heat.

A new father states, "I know nothing about babies," but he seems to be interested in learning. What action by the nurse is best? a. Continue to observe his interaction with the newborn. b. Tell him when he does something wrong. c. Show no concern, as he will learn on his own. d. Include him in teaching sessions.

ANS: D The nurse must be sensitive to the father's needs and include him whenever possible. As fathers take on care new role, the nurse should praise every attempt even if his early care is awkward. It is important to note the bonding process of the mother and the father, but that does not satisfy the expressed needs of the father. He should be encouraged by pointing out the correct procedures he does. Criticizing him will discourage him. The nurse should be sure to include him in all teaching sessions.

A nurse is observing a family. The mother is holding the baby she delivered less than 24 hours ago. Her husband is watching his wife and asking questions about newborn care. The 4-year-old brother is punching his mother on the back. What action by the nurse is best? a. Report the incident to the social services department. b. Advise the parents that the toddler needs to be reprimanded. c. Report to oncoming staff that the mother is not a good disciplinarian. d. Realize that this is a normal family adjusting to family change.

ANS: D The observed behaviors are normal variations of families adjusting to change. The nurse could provide suggestions on managing the adjustments. There is no need to report this one incident. The child does not need to be reprimanded, however; when the family is receptive the nurse could provide anticipatory guidance for this situation and help them problem solve. The nurse should avoid labeling the parents.

A postpartum patient asks, "Will these stretch marks go away?" The nurse's best response is a. "They will fade and be gone by your 6-week checkup." b. "No, unfortunately they will never fade away." c. "Yes, eventually they will totally disappear." d. "They will fade to silvery lines but won't disappear completely."

ANS: D The stretch marks will fade to silvery lines but will not disappear completely.

26. One reason the brain is vulnerable to nutritional deficiencies and trauma in early infancy is the: a. Incompletely developed neuromuscular system. b. Primitive reflex system. c. Presence of various sleep-wake states. d. Cerebellum growth spurt.

ANS: D The vulnerability of the brain likely is to the result of the cerebellum growth spurt. The neuromuscular system is almost completely developed at birth. The reflex system is not relevant. The various sleep-wake states are not relevant.

A nurse has taught a woman how to do Kegel exercises. What statement by the patient shows good understanding? a. "I contract my thighs, buttocks, and abdomen." b. "I do 10 of these exercises every day." c. "I stand while practicing this new exercise routine." d. "I pretend that I am trying to stop the flow of urine midstream."

ANS: D The woman can pretend that she is attempting to stop the passing of gas, or the flow of urine midstream. This will replicate the sensation of the muscles drawing upward and inward. Each contraction should be as intense as possible without contracting the abdomen, buttocks, or thighs. Guidelines suggest that these exercises should be done 24 to 100 times per day. Positive results are shown with a minimum of 24 to 45 repetitions per day. The best position to learn Kegel exercises is to lie supine with knees bent. A secondary position is on the hands and knees.

A woman gave birth vaginally to a 9-pound, 12-ounce girl yesterday. Her primary health care provider has written orders for perineal ice packs, use of a sitz bath tid, and a stool softener. What information is most closely correlated with these orders? a. The woman is a gravida 2, para 2. b. The woman had a vacuum-assisted birth. c. The woman received epidural anesthesia. d. The woman has an episiotomy.

ANS: D These orders are typical interventions for a woman who has had an episiotomy, lacerations, and hemorrhoids. A multiparous classification is not an indication for these orders. A vacuum-assisted birth may be used in conjunction with an episiotomy, which indicates these interventions, but that is not the only situation in which an episiotomy would be used, so this is not the best answer. Use of epidural anesthesia has no correlation with these orders.

A 25-year-old multiparous woman gave birth to an infant boy 1 day ago. Today her husband brings a large container of brown seaweed soup to the hospital. When the nurse enters the room, the husband asks for help with warming the soup so that his wife can eat it. The nurse's most appropriate response is to ask the woman a. "Didn't you like your lunch?" b. "Does your doctor know that you are planning to eat that?" c. "What is that anyway?" d. "I'll warm the soup in the microwave for you."

ANS: D This statement shows cultural sensitivity to the dietary preferences of the woman and is the most appropriate response. Cultural dietary preferences must be respected. Women may request that family members bring favorite or culturally appropriate foods to the hospital. Asking if the provider knows she is eating this soup is insensitive.

What does the student learn about recent trends in multiple births? a. The rate of twin births has declined. b. The rate of higher order pregnancies has increased. c. Higher order pregnancies are now very rare. d. Twinning is the most common form of multiple pregnancy.

ANS: D Twinning is the most common form of multiple pregnancy, and the rate has been increasing, not declining. Higher order births increased for a time but have now decreased, although they are not rare.

A pregnant woman's diet history indicates that she likes the following list of foods. The nurse would encourage this woman to consume more of which food to increase her calcium intake? a. Fresh apricots b. Canned clams c. Spaghetti with meat sauce d. Canned sardines

ANS: D Sardines are rich in calcium. Fresh apricots are not high in calcium. Canned clams are not high in calcium. Spaghetti with meat sauce is not high in calcium.

A woman has come to the clinic for preconception counseling because she wants to start trying to get pregnant in 3 months. She can expect the following advice: a. "Discontinue all contraception now." b. "Lose weight so that you can gain more during pregnancy." c. "You may take any medications you have been taking regularly." d. "Make sure you include adequate folic acid in your diet."

ANS: D "Make sure that you include adequate folic acid in your diet." A healthy diet before conception is the best way to ensure that adequate nutrients are available for the developing fetus. A woman's folate or folic acid intake is of particular concern in the periconception period. Neural tube defects are more common in infants of women with a poor folic acid intake. Depending on the type of contraception used, it may not be appropriate to discontinue all contraception at this time. Advising the client to lose weight now so that she can gain more during pregnancy is not appropriate advice. Depending on the type of medications the woman is taking, it may not be appropriate for her to continue taking them regularly.

As an adjunct to inductions, a number of procedures to ripen the cervix are employed. One of these methods is the vaginal administration of preparations using prostaglandins. Before administering this medication, the nurse should be aware that this class of drug is an appropriate choice for women who have had a prior cesarean birth. Is this statement true or false?

ANS: F Prostaglandins are contraindicated in patients who have had a prior cesarean birth or other uterine surgery. A side effect of prostaglandin administration is hyperstimulation of the uterus. This may result in reduced uterine blood flow to the fetus, impaired gas exchange, and increased risk of uterine rupture. Prostaglandins should also be used with caution in women who have asthma, heart disease, glaucoma, or renal or hepatic dysfunction.

Occasionally a woman arrives at the intrapartum unit ready to give birth. Bearing down, grunting, or stating something like "the baby's coming" should direct the nurse to advise the client, "Do not push, pant, and blow until the physician arrives." Is this statement true or false?

ANS: F The nurse's priority is to prevent or reduce injury to mother and infant if delivery is imminent. The emergency delivery kit should be obtained and preparation made for immediate delivery. An abbreviated assessment should be completed in order to obtain the mother's name, that of the support partner, and her care provider. Estimated date of delivery, allergies, and prenatal care are also important information. If time allows, maternal vital signs should be done, as well as a fetal assessment. After delivery, the priority is to maintain the infant's airway and temperature.

2. An increasing number of women are now becoming pregnant relatively late in their reproductive lives. The birth rate for women ages 40 to 44 has increased to the highest level in 40 years. Which advances in maternity care these women are at no greater risk for obstetric complications. Is this statement true or false?

ANS: F The older women it is definitely at an increased risk for obstetric complications, including spontaneous abortion, gestational diabetes, cesarean births, stillbirth, preeclampsia, placenta previa, abruption, preterm delivery, and low birth weight infants.

The woman in labor should be encouraged to use the Valsalva maneuver (holding one's breath and tightening abdominal muscles) for pushing during the second stage. Is this statement true or false?

ANS: F The woman should actually be discouraged from using the Valsalva maneuver. This activity increases intrathoracic pressure, reduces venous return, and increases venous pressure. During the Valsalva maneuver, fetal hypoxia may occur. The process is reversed when the woman takes a breath.

1. The nurse has been caring for a primiparous patient who is suspected of carrying a macrosomic infant. Pushing appears to have been effective so far; however, as soon as the head is born, it retracts against the perineum much like a turtle's head drawing into its shell. In evaluating the labor progress so far, the nurse is aware that this is normal with large infants and extra pushing efforts by the mother may be necessary. Is this statement true or false?

ANS: F This is often referred to as the "turtle sign" and is an indication of shoulder dystocia. Delayed or difficult birth of the shoulders may occur if they become impacted above the maternal symphysis pubis. This complication of birth requires immediate intervention because the umbilical cord is compressed and the chest cannot expand within the vagina. Any of several methods may be employed to relieve the impacted shoulders. Shoulder dystocia is unpredictable and although more common in large infants, can occur with a baby of any weight. PTS: 1 DIF: Cognitive Level: Analysis REF: p. 638 OBJ: Nursing Process: Evaluation MSC: Client Needs: Physiologic Integrity

1. Society tends to minimize perinatal loss because of the prevailing belief that there are no barriers to getting pregnant and the expectation that once a woman is pregnant, a healthy, live infant will result. Is this statement true or false?

ANS: T Because of these perceptions, grieving parents often do not receive the support they need, and society often allows much too short a time for mothers to grieve (and even less for fathers).

Pelvic congestion during pregnancy may lead to heightened sexual interest and increased orgasmic experiences. Is this statement true or false?

ANS: T Increased vascularity, edema, and connective tissue changes during pregnancy make the tissues of the vulva and perineum more pliable. This can lead to an increased interest in sexual activity and ease of orgasm.

The birth attendant evaluates whether labor and birth are safer for the woman and her fetus than continuing the pregnancy. The Bishop Scoring System remains a popular tool to assist in this task. Is this statement true or false?

ANS: T The Bishop score uses 5 factors to determine readiness for labor (dilation, effacement, consistency, position, and fetal station). The likelihood of a vaginal birth is similar to that of spontaneous labor if the score is greater than 8.

Pregnancy is a hypercoagulable state, where the mother's blood clots more readily. Is this statement true or false?

ANS: T This is because of an increase in factors that favor coagulation and a decrease in factors that inhibit coagulation. Fibrinogen increases by 50% and factors VII, VIII, IX, and X also rise.

The pregnant woman who is over the age of 35 years has risks related to pregnancy outcome. Which statement about increased risk in this age group is true? a. Fetal death is significantly lower in women 35 years and older. b. Chronic health problems are less likely to occur in women 35 years and older. c. Amniocentesis is recommended for all women 35 years and older. d. The cesarean section rate is decreased in women 35 years of age or older.

Amniocentesis is recommended for all women 35 years and older. Rationale: Amniocentesis is recommended for all women over age 35 to provide early detection of chromosomal abnormalities, including Down syndrome. The cesarean section rate is increased, chronic medical conditions are more frequent, and the risk of fetal death is significantly higher for women over 35 years of age.

What are appropriate nursing measures to help relieve a preterm infants pain during a painful procedure? (CHOOSE ALL THAT APPLY.) A. Swaddling the infant B. Use a pacifier dipper in sucrose C. Administering a prescribed sedative D. Keeping a hand near the face to be used for sucking

Answer: A, B, & D

A preterm infant with respiratory difficulties should be placed in which position to facilitate drainage? A. Supine B. Prone C. Trendelenburg D. Fowler's

Answer: B Rationale: A. Supine position can be used later as the infant matures and has improved respiratory efforts. B. The prone position is not recommended for normal newborn infants, because it is associated with an increased incidence of sudden infant death syndrome. However, the prone position for a preterm infant can facilitate drainage of respiratory secretions and regurgitated feedings. Prone positions also increase oxygenation and lung compliance and reduce energy expenditure. C. Trendelenburg position would increase the energy expenditure by making it more difficult for the infant to breath. D. Fowler's position would not assist with the drainage of secretions.

A pregnant client complains of severe heartburn. The nurse suggests which of the following relief measures for heartburn? a. Avoid fried and highly spiced foods. b. Lie down for half an hour after eating. c. Take a teaspoon of sodium bicarbonate in water after meals. d. Eat three well-spaced meals each day.

Avoid fried and highly spiced foods. Rationale: Remedies for heartburn include eating small and more frequent meals; avoiding fried and highly spiced foods; and remaining upright for at least an hour after meals. Use of lowsodium antacids also may be suggested. The use of sodium bicarbonate should be discouraged.

ABO incompatibility also causes pathologic jaundice in the newborn. Mothers with what blood type have natural antibodies to types A and B blood? A. It does not matter what blood type the mother is B. Type O C. Type A D. Type B

B ABO incompatibility also causes pathologic jaundice. Mothers with type O blood have natural antibodies to types A and B blood. The antibodies cross the placenta and cause hemolysis of fetal red blood cells. However, the destruction is much less severe than with Rh incompatibility and causes milder signs.

The patient has been diagnosed with hydramnios. When an amniotomy is performed, the nurse is aware that the patient is at risk for which complication? A Placenta previa B Abruptio placentae C Infection D Fetal hypoxia

B Abruptio placentae may occur after an amniotomy if the uterus is distended. Hydramnios will distend the uterus. Placenta previa is not a risk factor associated with hydramnios and/or amniotomy. Infection is a risk factor associated with amniotomy; however, having hydramnios does not increase the risk factor. Fetal hypoxia may occur if abruptio placentae occurs, but it is not a risk factor associated with amniotomy.

A woman is expecting her first baby in 7 months. During the nurse's assessment Anna continues to ask questions about changes in her body. The nurse can recommend which type of class to assist the woman with her questions? A. Preconception class B. Early pregnancy class C. Childbirth preparation class D. Parenting class

B Early pregnancy class An early pregnancy class focuses on the first two trimesters. They cover information on adapting to pregnancy, dealing with discomforts, and understanding what to expect. Preconception class is for couples thinking about having a baby. They are designed to help them prepare to have a healthy pregnancy. Childbirth preparation class focuses on preparation for labor and delivery. Parenting classes focus on care of the newborn.

The nurse is monitoring the feedings of the infant with hyperbilirubinemia. The purpose of ensuring that the infant receives feedings every 2 to 3 hours, whether by breast or bottle is to A. prevent hyperglycemia. B. provide fluids and protein. C. decrease gastrointestinal motility. D. prevent rapid emptying of the bilirubin from the bowel.

B Frequent feedings prevent hypoglycemia, provide protein to maintain the albumin level in the blood, and promote gastrointestinal motility and prompt removal of bilirubin in the stools.

In order to monitor for one of the side effects of oxytocin, it is important for the nurse to note the patient's A temperature. B intake and output. C respiratory rate. D deep tendon reflexes.

B Prolonged administration may cause fluid retention. Recording intake and output identifies fluid retention, which precedes water intoxication. Infection, respiratory depression, and alterations in deep tendon reflexes are not a side effect of oxytocin use.

The goal of treatment of the infant with phenylketonuria (PKU) is to: a. Cure mental retardation. b. Prevent central nervous system (CNS) damage, which leads to mental retardation. c. Prevent gastrointestinal symptoms. d. Cure the urinary tract infection

B CNS damage can occur as a result of toxic levels of phenylalanine. No known cure exists for mental retardation. Digestive problems are a clinical manifestation of PKU. PKU does not involve any urinary problems.

To care adequately for infants at risk for neonatal bacterial infection, nurses should be aware that: a. Congenital infection progresses more slowly than does nosocomial infection. b. Nosocomial infection can be prevented by effective handwashing; early-onset infections cannot. c. Infections occur with about the same frequency in boy and girl infants, although female mortality is higher. d. The clinical sign of a rapid, high fever makes infection easier to diagnose.

B Handwashing is an effective preventive measure for late-onset (nosocomial) infections because these infections come from the environment around the infant. Early-onset, or congenital, infections are caused by the normal flora at the maternal vaginal tract and progress more rapidly than do nosocomial (late-onset) infections. Infection occurs about twice as often in boys and results in higher mortality. Clinical signs of neonatal infection are nonspecific and are similar to those of noninfectious problems, thus making diagnosis difficult.

With regard to small for gestational age (SGA) infants and intrauterine growth restrictions (IUGR), nurses should be aware that: a. In the first trimester diseases or abnormalities result in asymmetric IUGR. b. Infants with asymmetric IUGR have the potential for normal growth and development. c. In asymmetric IUGR weight is slightly more than SGA, whereas length and head circumference are somewhat less than SGA. d. Symmetric IUGR occurs in the later stages of pregnancy.

B IUGR is either symmetric or asymmetric. The symmetric form occurs in the first trimester; SGA infants have reduced brain capacity. The asymmetric form occurs in the later stages of pregnancy. Weight is less than the 10th percentile; head circumference is greater than the 10th percentile. Infants with asymmetric IUGR have the potential for normal growth and development

To provide optimal care of infants born to mothers who are substance abusers, nurses should be aware that: a. Infants born to addicted mothers are also addicted. b. Mothers who abuse one substance likely will use or abuse another, thus compounding the infant's difficulties. c. The NICU Network Neurobehavioral Scale (NNNS) is designed to assess the damage the mother has done to herself. d. No laboratory procedures are available that can identify the intrauterine drug exposure of the infant.

B Multiple substance use (even just alcohol and tobacco) makes it difficult to assess the problems of the exposed infant, particularly with regard to withdrawal manifestations. Infants of substance-abusing mothers may have some of the physiologic signs but are not addicted in the behavioral sense. "Drug-exposed newborn" is a more accurate description than "addict." The NNNS is designed to assess the neurologic, behavioral, and stress/abstinence function of the neonate. Newborn urine, hair, or meconium sampling may be used to identify an infant's intrauterine drug exposure.

Necrotizing enterocolitis (NEC) is an inflammatory disease of the gastrointestinal mucosa. The signs of NEC are nonspecific. Some generalized signs include: a. Hypertonia, tachycardia, and metabolic alkalosis. b. Abdominal distention, temperature instability, and grossly bloody stools. c. Hypertension, absence of apnea, and ruddy skin color. d. Scaphoid abdomen, no residual with feedings, and increased urinary output.

B Some generalized signs of NEC include decreased activity, hypotonia, pallor, recurrent apnea and bradycardia, decreased oxygen saturation values, respiratory distress, metabolic acidosis, oliguria, hypotension, decreased perfusion, temperature instability, cyanosis, abdominal distention, residual gastric aspirates, vomiting, grossly bloody stools, abdominal tenderness, and erythema of the abdominal wall. The infant may display hypotonia, bradycardia, and metabolic acidosis.

A newborn was admitted to the neonatal intensive care unit after being delivered at 29 weeks of gestation to a 28-year-old multiparous, married, Caucasian woman whose pregnancy was uncomplicated until premature rupture of membranes and preterm birth. The newborn's parents arrive for their first visit after the birth. The parents walk toward the bedside but remain approximately 5 feet away from the bed. The nurse's most appropriate action would be to: a. Wait quietly at the newborn's bedside until the parents come closer. b. Go to the parents, introduce himself or herself, and gently encourage the parents to come meet their infant; explain the equipment first, and then focus on the newborn. c. Leave the parents at the bedside while they are visiting so they can have some privacy. d. Tell the parents only about the newborn's physical condition, and caution them to avoid touching their baby.

B The nurse is instrumental in the initial interactions with the infant. The nurse can help the parents "see" the infant, rather than focus on the equipment. The importance and purpose of the apparatus that surrounds their infant also should be explained to them. Parents often need encouragement and recognition from the nurse to acknowledge the reality of the infant's condition. Parents need to see and touch their infant as soon as possible to acknowledge the reality of the birth and the infant's appearance and condition. Encouragement from the nurse is instrumental in this process. Telling the parents only about the newborn's physical condition and cautioning them to avoid touching their baby is an inappropriate action.

Near the end of the first week of life, an infant who has not been treated for any infection develops a copper-colored, maculopapular rash on the palms and around the mouth and anus. The newborn is showing signs of: a. Gonorrhea. b. Congenital syphilis. c. Herpes simplex virus infection. d. Human immunodeficiency virus.

B The rash is indicative of congenital syphilis. The lesions may extend over the trunk and extremities.

The nurse is teaching a group of prenatal clients about care of the breasts during pregnancy. Which self-care measure might be effective for women with inverted nipples? a. A bra with wide straps. b. A bra with tucks that allow expansion for breast tissue. c. Breast shields. d. A bra that supports the nipple between shoulder and elbow.

Breast shields. Rationale: Breast shields designed to correct inverted nipples are effective for some women, but others gain no benefit from them. A well-fitting supportive bra that: (1) has wide, non-stretch straps; (2) has tucks that allow for expansion of breast tissue; and (3) supports the nipple line between the elbow and shoulder will not remedy inverted nipples, but will provide excellent support

After seeing the physician, the woman is confused about her upcoming induction. She states to the nurse, "The doctor said I would need a gel inserted prior to going into labor. What does that mean?" The nurse's response should be based on knowledge that A a lubricating gel is inserted prior to induction to facilitate the insertion of the electronic monitoring devices. B a gel is inserted prior to induction to stimulate the rupture of the membranes. C a gel is inserted prior to induction to ripen the cervix. a lubricating gel is inserted so that it will not need to be D reapplied prior to each vaginal exam during labor.

C

What factor is a contraindication for induction of labor? A Post term dates B Maternal hypertension C Previous cesarean section with a classic incision D Fetal death

C A classic incision for a cesarean section is a contraindication for induction of labor. Post term dates, maternal hypertension, and fetal death are indications for induction of labor.

Transitory tachypnea of the newborn (TTN) is thought to occur as a result of A. a lack of surfactant. B. hypoinflation of the lungs. C. delayed absorption of fetal lung fluid. D. a slow vaginal delivery associated with meconium-stained fluid.

C Delayed absorption of fetal lung fluid is thought to be the reason for TTN. A lack of surfactant is seen in preterm infants. Risk factors include cesarean birth without labor, precipitous delivery, male gender, perinatal asphyxia, and maternal diabetes or asthma. A slow vaginal delivery with meconium-stained fluid is not a risk factor for TTN.

A woman who is 7 months' pregnant states, "I'm worried that something will happen to my baby." The nurse's best response is A. "There is nothing to worry about." B. "The doctor is taking good care of you and your baby." C. "Tell me about your concerns." D. "Your baby is doing fine."

C Encouraging the woman to discuss her feelings is the best approach. The nurse should not disregard or belittle the woman's feelings.

After the use of forceps during labor, the nurse should assess the woman for signs of A bladder distention. B uterine atony. C vaginal lacerations. D deep vein thrombosis.

C Maternal risks include laceration or hematoma of the vagina, perineum, or periurethral area. Bladder distention, uterine atony, and deep vein thrombosis are not effects of forceps.

Signs of newborn drug exposure in utero usually begin how soon after birth? A. 2 hours B. 12 hours C. 24 to 72 hours D. I week later

C Signs develop during the first 24 to 72 hours after birth, but may not occur for up to 4 weeks, depending on the specific drug and the time of the mother's last use.

A woman tells the nurse she is 16 weeks' pregnant. During the assessment, the nurse measures the fundus of the uterus to be at the umbilicus. The nurse correctly interprets the comparison of the dates with the measurements to be A. not comparable. B. congruent. C. incongruent. D. irrelevant.

C The fundus should be at the umbilicus by 20 weeks. At 16 weeks, it is normally midway between the symphysis pubis and the umbilicus. The two sets of data do not match, and more assessment is necessary. From 16 to 18 weeks until 36 weeks, the fundal height, measured in centimeters, is approximately equal to the gestational age of the fetus in weeks.

During prenatal teaching it is important for the nurse to inform the patient about danger signs in pregnancy. Which sign need to be reported immediately to the health care provider? A. Clear mucous vaginal discharge B. Frequent urination C. Vaginal bleeding D. Backache that occurs after standing for a long period

C Vaginal bleeding during pregnancy needs to be reported immediately. It may be an indication of several complications of pregnancy, such as placenta previa or abruptio placenta. Mucous discharge may increase during pregnancy and is considered normal. Frequent urination is common during the first trimester and later in the third trimester. Backaches are the most common complaint during the third trimester.

Necrotizing enterocolitis (NEC) is an acute inflammatory disease of the gastrointestinal mucosa that can progress to perforation of the bowel. Approximately 2% to 5% of premature infants succumb to this fatal disease. Care is supportive; however, known interventions may decrease the risk of NEC. To develop an optimal plan of care for this infant, the nurse must understand which intervention has the greatest effect on lowering the risk of NEC: a. Early enteral feedings b. Exchange transfusion c. Breastfeeding d. Prophylactic probiotics

C A decrease in the incidence of NEC is directly correlated with exclusive breastfeeding. Breast milk enhances maturation of the gastrointestinal tract and contains immune factors that contribute to a lower incidence or severity of NEC, Crohn's disease, and celiac illness. The neonatal intensive care unit nurse can be very supportive of the mother in terms of providing her with equipment to pump breast milk, ensuring privacy, and encouraging skin-to-skin contact with the infant. Early enteral feedings of formula or hyperosmolar feedings are a risk factor known to contribute to the development of NEC. The mother should be encouraged to pump or feed breast milk exclusively. Exchange transfusion may be necessary; however, it is a known risk factor for the development of NEC. Although still early, a study in 2005 found that the introduction of prophylactic probiotics appeared to enhance the normal flora of the bowel and therefore decrease the severity of NEC when it did occur. This treatment modality is not as widespread as encouraging breastfeeding; however, it is another strategy that the care providers of these extremely fragile infants may have at their disposal.

As related to central nervous system injuries that could occur to the infant during labor and birth, nurses should be aware that: a. Intracranial hemorrhage (ICH) as a result of birth trauma is more likely to occur in the preterm, low-birth-weight infant. b. Subarachnoid hemorrhage (the most common form of ICH) occurs in term infants as a result of hypoxia. c. In many infants signs of hemorrhage in a full-term infant are absent and are diagnosed only through laboratory tests. d. Spinal cord injuries almost always result from forceps-assisted deliveries.

C Abnormalities in lumbar punctures or red blood cell counts, for instance, or in visuals on computed tomography scan may reveal a hemorrhage. ICH as a result of birth trauma is more likely to occur in the full-term, large infant. Subarachnoid hemorrhage in term infants is a result of trauma; in preterm infants it is a result of hypoxia. Spinal cord injuries are almost always from breech births; they are rare today because cesarean birth often is used for breech presentation.

An infant is to receive gastrostomy feedings. What intervention should the nurse institute to prevent bloating, gastrointestinal reflux into the esophagus, vomiting, and respiratory compromise? a. Rapid bolusing of the entire amount in 15 minutes b. Warm cloths to the abdomen for the first 10 minutes c. Slow, small, warm bolus feedings over 30 minutes d. Cold, medium bolus feedings over 20 minutes

C Feedings by gravity are done slowly over 20- to 30-minute periods to prevent adverse reactions. Rapid bolusing of the entire amount in 15 minutes would most likely lead to the adverse reactions listed. Temperature stability in the newborn is critical. Warm cloths to the abdomen for the first 10 minutes would not be appropriate because it is not a thermoregulated environment. Additionally, abdominal warming is not indicated with feedings of any kind. Small feedings at room temperature are recommended to prevent adverse reactions.

The most common cause of pathologic hyperbilirubinemia is: a. Hepatic disease. b. Postmaturity. c. Hemolytic disorders in the newborn. d. Congenital heart defect.

C Hemolytic disorders in the newborn are the most common cause of pathologic jaundice. Hepatic damage may be a cause of pathologic hyperbilirubinemia, but it is not the most common cause. Prematurity would be a potential cause of pathologic hyperbilirubinemia in neonates, but it is not the most common cause. Congenital heart defect is not a common cause of pathologic hyperbilirubinemia in neonates.

As related to the eventual discharge of the high risk newborn or transfer to a different facility, nurses and families should be aware that: a. Infants will stay in the neonatal intensive care unit (NICU) until they are ready to go home. b. Once discharged to home, the high risk infant should be treated like any healthy term newborn. c. Parents of high risk infants need special support and detailed contact information. d. If a high risk infant and mother need transfer to a specialized regional center, it is better to wait until after birth and the infant is stabilized.

C High risk infants can cause profound parental stress and emotional turmoil. Parents need support, special teaching, and quick access to various resources available to help them care for their baby. Parents and their high risk infant should spend a night or two in a predischarge room, where care for the infant is provided away from the NICU. Just because high risk infants are discharged does not mean that they are normal, healthy babies. Follow-up by specialized practitioners is essential. Ideally, the mother and baby are transported with the fetus in utero; this reduces neonatal morbidity and mortality.

Infants of mothers with diabetes (IDMs) are at higher risk for developing: a. Anemia. c. Respiratory distress syndrome. b. Hyponatremia. d. Sepsis.

C IDMs are at risk for macrosomia, birth injury, perinatal asphyxia, respiratory distress syndrome, hypoglycemia, hypocalcemia, hypomagnesemia, cardiomyopathy, hyperbilirubinemia, and polycythemia. They are not at risk for anemia, hyponatremia, or sepsis.

The nurse practicing in the perinatal setting should promote kangaroo care regardless of an infant's gestational age. This intervention: a. Is adopted from classical British nursing traditions. b. Helps infants with motor and central nervous system impairment. c. Helps infants to interact directly with their parents and enhances their temperature regulation. d. Gets infants ready for breastfeeding.

C Kangaroo care is skin-to-skin holding in which the infant, dressed only in a diaper, is placed directly on the parent's bare chest and then covered. The procedure helps infants interact with their parents and regulates their temperature, among other developmental benefits.

Providing care for the neonate born to a mother who abuses substances can present a challenge for the health care team. Nursing care for this infant requires a multisystem approach. The first step in the provision of this care is: a. Pharmacologic treatment. b. Reduction of environmental stimuli. c. Neonatal abstinence syndrome scoring. d. Adequate nutrition and maintenance of fluid and electrolyte balance.

C Neonatal abstinence syndrome (NAS) is the term used to describe the cohort of symptoms associated with drug withdrawal in the neonate. The Neonatal Abstinence Scoring System evaluates central nervous system (CNS), metabolic, vasomotor, respiratory, and gastrointestinal disturbances. This evaluation tool enables the care team to develop an appropriate plan of care. The infant is scored throughout the length of stay, and the treatment plan is adjusted accordingly. Pharmacologic treatment is based on the severity of withdrawal symptoms. Symptoms are determined by using a standard assessment tool. Medications of choice are morphine, phenobarbital, diazepam, or diluted tincture of opium. Swaddling, holding, and reducing environmental stimuli are essential in providing care to the infant who is experiencing withdrawal. These nursing interventions are appropriate for the infant who displays CNS disturbances. Poor feeding is one of the gastrointestinal symptoms common to this client population. Fluid and electrolyte balance must be maintained and adequate nutrition provided. These infants often have a poor suck reflex and may need to be fed via gavage.

For clinical purposes, preterm and post-term infants are defined as: a. Preterm before 34 weeks if appropriate for gestational age (AGA) and before 37 weeks if small for gestational age (SGA). b. Post-term after 40 weeks if large for gestational age (LGA) and beyond 42 weeks if AGA. c. Preterm before 37 weeks, and post-term beyond 42 weeks, no matter the size for gestational age at birth. d. Preterm, SGA before 38 to 40 weeks, and post-term, LGA beyond 40 to 42 weeks.

C Preterm and post-term are strictly measures of time—before 37 weeks and beyond 42 weeks, respectively—regardless of size for gestational age.

In caring for the preterm infant, what complication is thought to be a result of high arterial blood oxygen level? a. Necrotizing enterocolitis (NEC) b. Bronchopulmonary dysplasia (BPD) c. Retinopathy of prematurity (ROP) d. Intraventricular hemorrhage (IVH)

C ROP is thought to occur as a result of high levels of oxygen in the blood. NEC is caused by the interference of blood supply to the intestinal mucosa. Necrotic lesions occur at that site. BPD is caused by the use of positive pressure ventilation against the immature lung tissue. IVH results from rupture of the fragile blood vessels in the ventricles of the brain. It is most often associated with hypoxic injury, increased blood pressure, and fluctuating cerebral blood flow.

In caring for the mother who has abused (or is abusing) alcohol and for her infant, nurses should be aware that: a. The pattern of growth restriction of the fetus begun in prenatal life is halted after birth, and normal growth takes over. b. Two thirds of newborns with fetal alcohol syndrome (FAS) are boys. c. Alcohol-related neurodevelopmental disorders not sufficient to meet FAS criteria (learning disabilities, speech and language problems) are often not detected until the child goes to school. d. Both the distinctive facial features of the FAS infant and the diminished mental capacities tend toward normal over time.

C Some learning problems do not become evident until the child is at school. The pattern of growth restriction persists after birth. Two thirds of newborns with FAS are girls. Although the distinctive facial features of the FAS infant tend to become less evident, the mental capacities never become normal.

An infant is being discharged from the neonatal intensive care unit after 70 days of hospitalization. The infant was born at 30 weeks of gestation with several conditions associated with prematurity, including respiratory distress syndrome, mild bronchopulmonary dysplasia, and retinopathy of prematurity requiring surgical treatment. During discharge teaching the infant's mother asks the nurse whether her baby will meet developmental milestones on time, as did her son who was born at term. The nurse's most appropriate response is: a. "Your baby will develop exactly like your first child did." b. "Your baby does not appear to have any problems at the present time." c. "Your baby will need to be corrected for prematurity. Your baby is currently 40 weeks of postconceptional age and can be expected to be doing what a 40-week-old infant would be doing." d. "Your baby will need to be followed very closely."

C The age of a preterm newborn is corrected by adding the gestational age and the postnatal age. The infant's responses are evaluated accordingly against the norm expected for the corrected age of the infant. Although it is impossible to predict with complete accuracy the growth and development potential of each preterm infant, certain measurable factors predict normal growth and development. The preterm infant experiences catch-up body growth during the first 2 to 3 years of life. The growth and developmental milestones are corrected for gestational age until the child is approximately 2.5 years old. Stating that the baby does not appear to have any problems at the present time is inaccurate. Development will need to be evaluated over time.

As a result of large body surface in relation to weight, the preterm infant is at high risk for heat loss and cold stress. By understanding the four mechanisms of heat transfer (convection, conduction, radiation, and evaporation), the nurse can create an environment for the infant that prevents temperature instability. While evaluating the plan that has been implemented, the nurse knows that the infant is experiencing cold stress when he or she exhibits: a. Decreased respiratory rate. b. Bradycardia followed by an increased heart rate. c. Mottled skin with acrocyanosis. d. Increased physical activity.

C The infant has minimal to no fat stores. During times of cold stress the skin will become mottled, and acrocyanosis will develop, progressing to cyanosis. Even if the infant is being cared for on a radiant warmer or in an isolette, the nurse's role is to observe the infant frequently to prevent heat loss and respond quickly if signs and symptoms occur. The respiratory rate increases followed by periods of apnea. The infant initially tries to conserve heat and burns more calories, after which the metabolic system goes into overdrive. In the preterm infant experiencing heat loss, the heart rate initially increases, followed by periods of bradycardia. In the term infant, the natural response to heat loss is increased physical activity. However, in a term infant experiencing respiratory distress or in a preterm infant, physical activity is decreased.

A plan of care for an infant experiencing symptoms of drug withdrawal should include: a. Administering chloral hydrate for sedation. b. Feeding every 4 to 6 hours to allow extra rest. c. Swaddling the infant snugly and holding the baby tightly. d. Playing soft music during feeding.

C The infant should be wrapped snugly to reduce self-stimulation behaviors and protect the skin from abrasions. Phenobarbital or diazepam may be administered to decrease central nervous system (CNS) irritability. The infant should be fed in small, frequent amounts and burped well to diminish aspiration and maintain hydration. The infant should not be stimulated (such as with music) because this will increase activity and potentially increase CNS irritability.

A primigravida has just delivered a healthy infant girl. The nurse is about to administer erythromycin ointment in the infant's eyes when the mother asks, "What is that medicine for?" The nurse responds: a. "It is an eye ointment to help your baby see you better." b. "It is to protect your baby from contracting herpes from your vaginal tract." c. "Erythromycin is given prophylactically to prevent a gonorrheal infection." d. "This medicine will protect your baby's eyes from drying out over the next few days."

C With the prophylactic use of erythromycin, the incidence of gonococcal conjunctivitis has declined to less than 0.5%. Eye prophylaxis is administered at or shortly after birth to prevent ophthalmia neonatorum. Erythromycin has no bearing on enhancing vision, is used to prevent an infection caused by gonorrhea, not herpes, and is not used for eye lubrication.

When preparing a woman for a pelvic examination, the nurse notices that she had undergone a genital mutilation. During the examination, the nurse needs to plan for the woman to A. feel embarrassed because of the mutilation. B. be comfortable with the examination. C. be concerned that a full examination will not be possible. D. experience pain and to make her as comfortable as possible.

D Because the introitus is so small and there is scar tissue that is inelastic, the woman will experience pain with the examination. A full examination may be possible depending on the extent of the mutilation.

While the vital signs of a pregnant woman in her third trimester are being assessed, the woman, who is lying supine, complains of feeling faint, dizzy, and agitated. Which nursing intervention is appropriate? A. Have the patient stand up; retake her blood pressure. B. Have the patient sit down and hold her arm in a dependent position. C. Have the patient lie supine for 5 minutes; recheck her blood pressure on both arms. D. Have the patient turn to her left side; recheck her blood pressure in 5 minutes.

D Blood pressure is affected by positions during pregnancy. The supine position may cause occlusion of the vena cava and descending aorta. Turning the pregnant woman to a lateral recumbent position alleviates pressure on the blood vessels and quickly corrects supine hypotension. Having the patient stand up would cause an increase in systolic and diastolic pressures. Having the patient hold her arm in a dependent position will cause a false reading.

The infant of a diabetic mother is hypoglycemic. What type of feeding should be instituted first? A. Glucose water in a bottle B. D5W intravenously C. Formula via nasogastric tube D. Breastfeeding, or breast milk/formula in a bottle

D Breastfeeding or breast milk/formula by bottle should be given first to raise the blood glucose level not glucose water. Oral feedings are tried first, and intravenous lines may be initiated if the hypoglycemia continues. Formula would be administered via bottle, not by tube feeding.

Internal version might be used to manipulate the A fetus from a breech to a cephalic presentation before labor begins. B fetus from a transverse lie to a longitudinal lie before cesarean birth. C second twin from an oblique lie to a transverse lie before labor begins. D second twin from a transverse lie to cephalic during vaginal birth.

D Internal version is used during vaginal birth to manipulate the fetus into a longitudinal lie (cephalic or breech) that allows it to be born vaginally.

During a prenatal visit at 36 weeks of gestation, the nurse tested a woman's urine for glucose and protein. The results indicated a trace amount of glucose. The nurse's next action should be to A. retest the urine for accuracy. B. have the woman give another sample for retesting. C. report the results immediately to the physician so further testing can be preformed. D. consider this as a normal result for this stage of pregnancy.

D Small amounts of glucose in the urine may indicate physiologic spilling that occurs during normal pregnancy, and further testing is not necessary. Larger amounts of glucose in the urine require further testing.

Four hours after the delivery of a healthy neonate of an insulin-dependent diabetic mother, the baby appears jittery and has rapid respirations and poor muscle tone. Which nursing action has top priority? A. Start an intravenous line with D5W. B. Notify the clinician state. C. Document the event in the nurses' notes. D. Test for blood glucose level.

D These symptoms are signs of hypoglycemia in the newborn, and the nurse should first test the blood glucose level according to agency's policy, treat symptoms with standing orders protocol, and then notify the physician with the results.

On day 3 of life, a newborn continues to require 100% oxygen by nasal cannula. The parents ask whether they can hold their infant during his next gavage feeding. Given that this newborn is physiologically stable, what response would the nurse give? a. "Parents are not allowed to hold infants who depend on oxygen." b. "You may hold only your baby's hand during the feeding." c. "Feedings cause more physiologic stress, so the baby must be closely monitored. Therefore, I don't think you should hold the baby." d. "You may hold your baby during the feeding."

D "You may hold your baby during the feeding" is an accurate statement. Parental interaction via holding is encouraged during gavage feedings so that the infant will associate the feeding with positive interactions. Nasal cannula oxygen therapy allows for easier feedings and psychosocial interactions. The parent can swaddle the infant during gavage feedings to help the infant associate the feeding with positive interactions. Some parents like to do kangaroo care while gavage feeding their infant. Swaddling or kangaroo care during feedings provides positive interactions for the infant.

With regard to hemolytic diseases of the newborn, nurses should be aware that: a. Rh incompatibility matters only when an Rh-negative child is born to an Rh-positive mother. b. ABO incompatibility is more likely than Rh incompatibility to precipitate significant anemia. c. Exchange transfusions frequently are required in the treatment of hemolytic disorders. d. The indirect Coombs' test is performed on the mother before birth; the direct Coombs' test is performed on the cord blood after birth.

D An indirect Coombs' test may be performed on the mother a few times during pregnancy. Only the Rh-positive child of an Rh-negative mother is at risk. ABO incompatibility is more common than Rh incompatibility but causes less severe problems; significant anemia, for instance, is rare with ABO. Exchange transfers are needed infrequently because of the decrease in the incidence of severe hemolytic disease in newborns from Rh incompatibility.

When providing an infant with a gavage feeding, which of the following should be documented each time? a. The infant's abdominal circumference after the feeding b. The infant's heart rate and respirations c. The infant's suck and swallow coordination d. The infant's response to the feeding

D Documentation of a gavage feeding should include the size of the feeding tube, the amount and quality of the residual from the previous feeding, the type and quantity of the fluid instilled, and the infant's response to the procedure. Abdominal circumference is not measured after a gavage feeding. Vital signs may be obtained before feeding. However, the infant's response to the feeding is more important. Some older infants may be learning to suck, but the important factor to document would be the infant's response to the feeding (including attempts to suck).

What bacterial infection is definitely decreasing because of effective drug treatment? a. Escherichia coli infection b. Candidiasis c. Tuberculosis d. Group B streptococcal infection

D Penicillin has significantly decreased the incidence of group B streptococcal infection. E. coli may be increasing, perhaps because of the increasing use of ampicillin (resulting in a more virulent E. coli resistant to the drug). Tuberculosis is increasing in the United States and Canada. Candidiasis is a fairly benign fungal infection.

HIV may be perinatally transmitted: a. Only in the third trimester from the maternal circulation. b. From the use of unsterile instruments. c. Only through the ingestion of amniotic fluid. d. Through the ingestion of breast milk from an infected mother.

D Postnatal transmission of HIV through breastfeeding may occur. Transmission of HIV from the mother to the infant may occur transplacentally at various gestational ages. This is highly unlikely because most health care facilities must meet sterility standards for all instrumentation. Transmission of HIV may occur during birth from blood or secretions.

Human immunodeficiency virus (HIV) may be perinatally transmitted: a. Only in the third trimester from the maternal circulation. b. By a needlestick injury at birth from unsterile instruments. c. Only through the ingestion of amniotic fluid. d. Through the ingestion of breast milk from an infected mother.

D Postnatal transmission of HIV through breastfeeding may occur. Transmission of HIV from the mother to the infant may occur transplacentally at various gestational ages. Transmission close to or at the time of birth is thought to account for 50% to 80% of cases.

When assessing the preterm infant the nurse understands that compared with the term infant, the preterm infant has: a. Few blood vessels visible through the skin. b. More subcutaneous fat. c. Well-developed flexor muscles. d. Greater surface area in proportion to weight.

D Preterm infants have greater surface area in proportion to their weight. More subcutaneous fat and well-developed muscles are indications of a more mature infant.

While completing a newborn assessment, the nurse should be aware that the most common birth injury is: a. To the soft tissues. b. Caused by forceps gripping the head on delivery. c. Fracture of the humerus and femur. d. Fracture of the clavicle.

D The most common birth injury is fracture of the clavicle (collarbone). It usually heals without treatment, although the arm and shoulder may be immobilized for comfort.

Because of the premature infant's decreased immune functioning, what nursing diagnosis should the nurse include in a plan of care for a premature infant? a. Delayed growth and development b. Ineffective infant feeding pattern c. Ineffective thermoregulation d. Risk for infection

D The nurse needs to understand that decreased immune functioning increases the risk for infection. Growth and development, thermoregulation, and feeding may be affected, although only indirectly.

A pregnant woman at 37 weeks of gestation has had ruptured membranes for 26 hours. A cesarean section is performed for failure to progress. The fetal heart rate (FHR) before birth is 180 beats/min with limited variability. At birth the newborn has Apgar scores of 6 and 7 at 1 and 5 minutes and is noted to be pale and tachypneic. On the basis of the maternal history, the cause of this newborn's distress is most likely to be: a. Hypoglycemia. b. Respiratory distress syndrome. c. Phrenic nerve injury. d. Sepsis.

D The prolonged rupture of membranes and the tachypnea (before and after birth) both suggest sepsis. An FHR of 180 beats/min is also indicative. This infant is at high risk for sepsis.

A macrosomic infant is born after a difficult forceps-assisted delivery. After stabilization the infant is weighed, and the birth weight is 4550 g (9 pounds, 6 ounces). The nurse's most appropriate action is to: a. Leave the infant in the room with the mother. b. Take the infant immediately to the nursery. c. Perform a gestational age assessment to determine whether the infant is large for gestational age. d. Monitor blood glucose levels frequently and observe closely for signs of hypoglycemia.

D This infant is macrosomic (more than 4000 g) and is at high risk for hypoglycemia. Blood glucose levels should be monitored frequently, and the infant should be observed closely for signs of hypoglycemia. Observation may occur in the nursery or in the mother's room, depending on the condition of the fetus. Regardless of gestational age, this infant is macrosomic.

Premature infants who exhibit 5 to 10 seconds of respiratory pauses followed by 10 to 15 seconds of compensatory rapid respiration are: a. Suffering from sleep or wakeful apnea. b. Experiencing severe swings in blood pressure. c. Trying to maintain a neutral thermal environment. d. Breathing in a respiratory pattern common to premature infants.

D This pattern is called periodic breathing and is common to premature infants. It may still require nursing intervention of oxygen and/or ventilation. Apnea is a cessation of respirations for 20 seconds or longer. It should not be confused with periodic breathing.

A second-trimester client in the prenatal clinic complains of ankle edema. Which intervention should the nurse suggest? a. Elevate legs when sitting. b. Stretching exercises before bedtime. c. Practice plantar flexion when standing. d. Wear ankle socks daily.

Elevate legs when sitting. Rationale: Elevating the legs when sitting will assist circulation and therefore reduce edema. Dorsiflexion and avoiding any restrictive bands around the ankles are appropriate to prevent ankle edema. Stretching exercises before bedtime help relieve leg cramps.

A prenatal client at 10 weeks' gestation is complaining of urinary frequency. Which self-care strategy should the nurse teach? a. Empty bladder every 4 hours. b. Decrease the amount of fluid intake. c. Empty the bladder when the urge is felt. d. Empty bladder every hour.

Empty the bladder when the urge is felt. Rationale: Fluid intake should be maintained at 2,000 mL, not decreased. The bladder should be emptied every 2 hours, not every hour (too frequent) or every 4 hours (not frequent enough). The bladder should be emptied when the urge is felt, not delayed.

Which minerals and vitamins usually are recommended to supplement a pregnant woman's diet? a. Fat-soluble vitamins A and D b. Water-soluble vitamins C and B6 c. Iron and folate d. Calcium and zinc

Iron and folate Iron generally should be supplemented, and folic acid supplements often are needed because folate is so important. Fat-soluble vitamins should be supplemented as a medical prescription, as vitamin D might be for lactose-intolerant women. Water-soluble vitamin C sometimes is consumed in excess naturally; vitamin B6 is prescribed only if the woman has a very poor diet. Zinc sometimes is supplemented. Most women get enough calcium.

A nurse is teaching a group of prenatal clients about hazards in the workplace during pregnancy. The nurse correctly teaches that pregnant women who have jobs requiring long periods of standing have higher incidences of: a. Preterm birth. b. Placenta previa. c. Prolapsed cord. d. Abruptio placentae

Preterm birth. Rationale: Preterm birth is an occupational hazard for women who work standing up for prolonged periods, as there is more uterine stimulation while standing than while sitting or lying down. Prolapsed cord, placenta previa, and abruptio placentae are not related to prolonged standing

The labor nurse is admitting a patient in active labor with a history of genital herpes. On assessment, the patient reports a recent outbreak, and the nurse verifies lesions on the perineum. What is the nurse's next action? a. Ask the patient when she last had anything to eat or drink. b. Take a culture of the lesions to verify the involved organism. c. Ask the patient if she has had unprotected sex since her outbreak. d. Use electronic fetal surveillance to determine a baseline fetal heart rate.

a. Ask the patient when she last had anything to eat or drink.

An infant of a diabetic mom arrives in the nursery unit for observation. The infant is term at 38 weeks' gestation and weighs 10 pounds. The maternal hemoglobin A1c level is noted at 10%. Which findings would the nurse suspect as being present? (Select all that apply.) a. Fetus is jittery, temperature is decreased b. Nasal flaring and retractions c. Slight jaundice noted on blanching of nose d. Calcium level of 10 mg/dL

a. Fetus is jittery, temperature is decreased b. Nasal flaring and retractions c. Slight jaundice noted on blanching of nose

A pregnant patient with acquired immunodeficiency syndrome (AIDS) is reviewing infant care instructions with the prenatal nurse. Which patient statement indicates to the nurse that the teaching was effective? a. "I will bathe my baby twice a day." b. "I will use premixed formula to feed my baby." c. "I will use gloves to change my baby's diapers." d. "I will use alcohol wipes six times a day on the baby's cord until it falls off."

b. "I will use premixed formula to feed my baby."

Which postpartum client requires further assessment? a. G4 P4 who has had four saturated pads during the last 12 hours b. G1 P1 with Class II heart disease who complains of frequent coughing c. G2 P2 with gestational diabetes whose fasting blood sugar level is 100 mg/dL d. G3 P2 postcesarean client who has active herpes lesions on the labia

b. G1 P1 with Class II heart disease who complains of frequent coughing


Conjuntos de estudio relacionados

Unit 13 vocabulary: World War II

View Set

Passpoint Questions that are hard, weird, or specific

View Set

Coursera (Practice Quiz: Code Reviews)

View Set

CA LIFE STATE PRACTICE EXAM PT 2(FROM EXAM FX ONLINE TEST)

View Set

Chapter 35: The Child with Neuromuscular or Muscular Dysfunction

View Set

Science Chapter 13 - Directed Reading

View Set